SOLVED Recalls April 2018 and Previous

You might also like

Download as pdf or txt
Download as pdf or txt
You are on page 1of 256

Some

contro ques given initially ,solved recently

Post op incarcerated hernia 1st day aggitation stem fever sob O2 sat 88% xray given after o2 what
initial step.xray had patches! What initial (xray had patches here)
IV antibiotics
Heparin
Thrombolysis
Droperidol


CIN stage II, upper border of lesion not seen? What to do next?
A – Large Loop Excision of the Transformation Zone of the cervix
B – Radiotherapy
C- Chemotherapy
D- cold knife conization/biopsy
E- hysterectomy




Middle age man, pain in buttock and thigh during 100 m walk on ground, can walk 20 m uphill,
femoral pulse not palpable, dorsalis pedis is palpable, ABI 0.3. Best way to diagnose?
A. Arterial Doppler
B. Digital subtraction arteriography
c. ct angiogram (for intervention)
d. arteriography


Lady of zuclophenthixol depot for psych issues bcz she’s non compliant with oral meds. Tried
olzapine n resperidone previously but non compliant. Now with ridigidty n tremor what will u do?
Switch to quietiapine
Resperidal consta
Resperidone
Clozapine

A baby delivered by normal vaginal delivery, is well after birth. On the 4th day, the baby is found
collapsed in the cot, breathless and floppy. On examination there are no murmurs. Possible cause
could be--
a) TOF
b) PDA
c) TGA with VSD
d) Pulmonary stenosis
e) L. Ventricular Hypertrophy hypoplastic left heart
60 yrs old male 10 yrs history of hypertension,feeling dizzy ,and light headache from this morning
when he was doing exercise.smoke 20 cigarettes per day,drinks 2-3 glass of wine most of the night
.BP 130/80 ,Sitting 120/75 on lying.currently on ibesertan-thiazide 150/12.5 mg ECG was given(svt)
cause of his dizziness.
A.Dehydration
B.Ischemic heart diseases
C.Hypertension
D.alcholic cardiomyopathy
E. MI



55 yr old lady with insulin dependent dm , has had right lwg amputation 5yrs previously ,now blood
pressurw 175/90 normal regular pulse bmi32 ldl 2.8 triglycerides 4.5 hba1c 8.5 fasting blood glucose
was 9.5 .which of the following is important to keep hwr left leg from amputation?
A) meticulous foot care
B)control bp
C) tight glycemic control
E)reduce ldl

35 year old lady comes to you regarding screening for breast cancer screening. She says her paternal
aunt was diagnosed with breast and ovarian cancer at the age of 60 years which was diagnosed as
being associated with BRCA1.Which of the following is the most appropriate advice?
A-Genetic counselling assessment
B-refer for BRCA 1 screening
C-screen now
D-tell her to look at her breast
E-annual mammography

22. What is the emergency case:
hepatic injury, free fluid in the peritoneum, intra vascular splenic injury

Teenage boy comes to you. His exam is near. He looks so anxious, sleeping less and saying that meat
is poisonous. What is happening to him?
A. Hypochondriasis
B. Delusion

27. farmer present with depressive symptoms and angry due to drought. Spend money recklessly in
women and accusing government for the loss due to drought. He said he is just exhausted and not
depressed. Which kind of mechanism is he demonstrating? (exact option) I confused this one.
a) Denial
b) Reaction formation
c) Projection
d) Displacement
e) Rationalisation

A 68 years old man with COPD was brought to your surgery from nursing home by
ambulance. On his way to hospital he received O2, 10 L/m by mask. He is still unarousable
and his ABG most likely--
a) Ph 7.29 PaCO2 65 PaO2 85
b) Ph 7.15 PaCO2 50 PaO2 68
c) Ph 7.25 PaCO2 25 PaO2 100



Lawyer who is a perfectionist, recently fall in performance after miscarriage, affect sleeping, crying,
can’t concentrate her work and problem with husband. don’t want to take medicine coz it might be
habit forming. what is suitable management at this stage?
a) CBT
b) SSRI
c) Interpersonal therapy
d) Diazepam


There is a surgeon always shouting to the staff or others. It’s happened several times. The staffs get
upset and some crying until left the operation room during operation. You are intern, what you
should do:
talk to surgeon,
talk to anaesthetist,
raise the problem at the intern meeting,
report to director of clinical training

Teenage boy comes to you. His exam is near. He looks so anxious, sleeping less and saying that meat
is poisonous. What is happening to him?
A. Hypochondriasis
B. Delusion

CIN stage II, upper border of lesion not seen? What to do next?
A – Large Loop Excision of the Transformation Zone of the cervix
B – Radiotherapy
C- Chemotherapy
D- cold knife conization/biopsy
E- hysterectomy

5 year old lady comes to you regarding screening for breast cancer screening. She says her paternal
aunt was diagnosed with breast and ovarian cancer at the age of 60 years which was diagnosed as
being associated with BRCA1.Which of the following is the most appropriate advice?
A-Genetic counselling assessment
B-refer for BRCA 1 screening
C-screen now
D-tell her to look at her breast
E-annual mammography

File ques given below


1)Gbs stem.how to monitor respiration.
FEV
FVC
Monitor chest expansion (ref jm )

2) Post op incarcerated hernia 1st day aggitation stem fever sob O2 sat 88% xray given after o2 what
initial step.xray had patches! What initial
IV antibiotics
Heparin
Thrombolysis
Droperidol

Chest pain 12 hrs pain reduced 4/10 on ecg st depression o2 given nxt
Morphine
GTN
Heparin ( oxford 114 )
Thrombolysis
Wpw hr 280 nxt
Verapamil
Amiodarone ( wpw : antiarrythmic amiodaron 1st line : other drug increases av nodal delay and increase
renentry to accessory pathway)
Cardiovert

Central chest pain incresing on inspiration..has fever ecg given cudnt make it out exactly what it was!
Guess pericarditis
What next aft O2
Heparin
Nsaid (anti-inflammatory )
Cath lab
Thrombolysis

Old lady fever agitation admitted uti given trimethoprium.was taking carbamazapin 4wks..PTU since yrs n
antihtnives.labs now shows Na 120 what next
Stop carbamazapine give hypertonic saline
Stop trimethoprium give N/S
Stop ptu give saline
Stop carbamazapine n restrict fluids

Carmazepine causes bloking of Na channel>hyponatraemia (CBZ induced SIADH )>urinary retension and
LUTS

Temporal lobe epilepsy child n lady in shopping center both old n same stems.

Xray given pt with hematuria and shortness of breath since 2months serum ca high rest all normal what
next
Ace levels (dx :sarcoidosis )
Ct chest
Antigbm
No choice of biopsy

Hypercalcemia >calceuria >nephrocalcinosis>nephrolithiasis …(hamaturia )

(there is similarity with good pasture syndrome - pulmonary renal syndrome )

scenrio of a schizophrenic patient walking naked in streets.which of the following is LEAST effective for
this patient.
A.olanzapine
b.quietiapine
c.amisulpride
d.clozapine

Encoparesis ( uncontrol defecaton )7yrs girl fighting with brother wen he call him smelly ?
Normal developmnt
Delayed developmnt
ODD
Regression
Depression
Pic of sle rash face … supposing.arthritis mouth ulcers trmnt
Methotrexate
Hydroxychloroquine
Cyclosporin

Man with tirdness lethargy cough dyspnea


Labs shows
Hb 104
Tlc 40
Plt 90
Blast cells 60%
Diagnosis ?

Acute leukemia
Bone marrow infiltration by CA lung
CML
CLL

Psoriasis rash on whole legs since 6 mons trtmnt


Uv therapy
Calcipotriol
Predsinolone

Postop cataract 5th day got up with pain n blurring of vision red eye pic laterly i cudnt apreciate exactly
ant eye had PUS or not
Hypopyon
Glaucoma
Scleritis

MR pt going fo colonscopy what prophylaxis need


Diffrnt antibiotics in 4 options
No prophylaxis required

Pt allergic to penicilline staph aureus on culture


Ceftriaxone
Cefazoline
Cefalaxine
vancomycin

Xray given plumber resp symptoms n signs mix.fev n fvc values given frgot what read the topic.asked
next
CT
Pleural biopsy
Trial of steroids

1. 3 synto questions
2.#april2018
Man with nocturia n urinary frequency n dribbling. mass above the pubic symphysis DRE
showed enlarged prostate with palpable median sulcus palpable.what is the most
appropriate next step?
A-trans rectal ultrasound
B-PSA
C-s. Creatinine
D-CT
E-urine cytology
3. Sarcoidosis- skin biopsy and ACE
4. rheumatic arthritis inv- throat swab and esr
5. Viral pneumonia trip to Asia
6. 5.old man stopped taking drugs ,now he has edema up to knee , HR 106 HTN with
clear lungs
what next ?
a.fursmide
b.spironolactone
c.digoxin
d.BB
E.recommence ACE ,BB,other drugs but not diuretics
7. Feeling of inferiority
8. Radical prostatectomy
9. Child with urinary ketones- hba1c
10. CPAP
11. Empyema
12 restless leg syndrome- iron studies
13. Hutchinson's freckle- local excision
14. Mitral stenosis_ pulm htn x-ray
15. 1.Arab woman who doesn’t speak English with 16. weeks pregnancy, was referred by
the mid-wife for which she suspects her mental condition. The woman seems irritable,
worried(or anxious) about the people around and she’s uncomfortable when her husband
is not around. She is also irritated with her 2 children. What condition in her history will be
present to lead you to diagnose this patient? (looks like prodromal symptoms)
1. Panic attacks
2. Paranoid personality disorder
3. Family member with schizophrenia
4. History of trauma
17. Lateral epicondylitis- brace below elbow
18. Methotrexate induced hepatitis
19. Uremic encephalopathy- reason for confusion
20. Osteoporosis - zolendroic acid
21 osteoporosis with menopausal symptoms
22. Menopausal symptoms - dvt with hysterectomy history in the past- transdermal
estrogen
23. Bag and mask ventilation with light meconium stained liquor
24. Ankylosing spondylitis managed with physiotherapy failed what's next step- naproxen
2. 51 yrs old man presents to your office for back pain. Diagnosed with ankylosing
spondylitis 20 yrs ago and has mild back pain all the time. has received no medical
treatment, and was on exercise program on n off. patient suffers from peptic ulcer has
been treated several times for it. which one of the ffw is the most appropriate mx
3. a) paracetamol
4. b) naproxen
5. c) methotrexate
6. d) sulfasalazin
7. e) infliximab
25. 35. Female 42 years of age presents with mild hypertension for which she was
started on ramipril. After 2 months she presents with B.P 170/100 , nocturia and Blood
chemistry showing increase in urea and creatinine well above baseline. Which of the
following will point towards her disease?
There was Presence of 3 cysts in left kidney and 2 cysts in the right- pkd (2ndy HTN due
to pckd )
26. Chronic purulent ear discharge - Cipro ear drops
27. Glaucoma - long term management- iridotomy
28. Acute Gout with ckd - Prednisone
29. Abdominal pain with af -Serum lactate
30. Sigmoid carcinoma - one with CT and one without
8. A policeman is coming for his collegue that beaten his friend during an argument,he is
intoxicated for alchol what in history will be important to say personality sdisorder
History of fight with colleague two weeks ago History of animal crulity during childhood
Alchol and drug abuse

old male with heart faluire ,HTN,diabetes ,he is on clozapine for three weeks now
complaining of shortness of breath and palpitation ,HR,Bl.p all normal what next :
a.clozapine level
b.echo
c.FBC
D.irrilevant
4.Old age woman on multiple drugs digoxin ACEI BB ,she stopped all drugs 3 weeks ago
as
she was in a trip and ran out of drugs ,she is complaining of confusion ,and occasional
postural dizziness (the exact words ) ..lab results show :
Creatinine 118 (normal up to 90 )
Non fasting blood glucose : 4
Bl p. sitting 168/70 ,standing 130 /60
What is the cause :
a.dehydration .
b.renal impairment
c.hypoglycemia

7.7 weeks child with high fever ,vomiting increased until it is now after every meal ,no
other
sighns, urine exam revealed 5*10*9 cells what next :
Oral cipro
Oral amoxi
IV gentamycin
IV cephalosporin
74 yrs old male has metastatic prostate cancer ,he is taking morphine for pain relieve and
self-prescribed cannabis but recently they arenot working and he is complaining of
insomnia and agitation ,what will be better for him:
a.sleep hygiene b.motivational therapy c.supportive psychotherapy d.interpersonal
therapy e.visual photo therapy (something like that )
pt comes.with history of headache (no word of mild or severe just headache) with
retrorbital pain and ptosis. He has history of previous recurrent headaches also. Now
gives history of neck pain but NO STIFFNESS. neurological exam is normal...what 2 do
A) Do urgenf CT
B) give him trial of 100 % Oxygen
Bilateral pedal oedema in lady
A patient comes with progressive dyspnea, facial puffiness, pedal edema and ascites. He
has bilateral basal crackles. His chest xray shows bilateral diffuse infiltrates. Urine
analysis protein 3+, rbc 3+. What is the diagnosis?
a. Nephritic syndrome
b. Post streptococcal GN
c. IGA nephropathy
d. Anti glomerular basement disease.
e. Heart failure
Day 7 post renal transplant , no urine output . Planned for dialysis , what is the cause ?
a.Acute rejection
b.Obstruction
c.Acute tubular necrosis
d.Donors graft thrombosis

During labour oxytocin was applied d t weak contractions but the fetal heart rate dropped to 70
and took few min. To go back to normal ,the membrane ruptured and the baby’s head is molded
,what next
Stop oxytocin
Oxygen mask to the mother
IV dextrose 50%to the mother
CS
Ventouse

1) Patient with mitral stenosis. Where to listen and what kind of murmur is it?
A. Diastolic, carotid arteries
B. Systolic, 2nd intracostal space on the left
C. Systolic, axillary line
D. Diastolic, lower
2) left sternum border

2) 23 y.o. Medical student, yellow scleras, high total bilirubin, 4/5 is unconjugated.
A. Gylberts syndrome ref jm 649
B. Cholecystitis
C. Cholelithiasis
D. Cholangitis
student posted interesting case on his facebook profile from the hospital. What to do?
A. Inform uni student supervisor
B. Inform hospital administrator
C. Ask him to take it of
D. Contact facebook

) Patient wrote on his Facebook page that you are a bad practitioner and that no one should go with you.
What to do?
A. Contact patient and ask him to take it of
B. Talk to hospital manager
C. Do nothing

n there is a option called contact medical defense, because they will provide further information whether
to just let it go or to trace the pt and discuss the issue and ask him to take it down,

patient cant fall asleep until he goes through his whole day in his head before going to sleep to make sure
hi didnt do anything bad at work. He is aware its not good what he is doing but cant stop. What is the
treatment?
A) Tricyclic antidepressants
B) SSRI ( OCD )
C) Haloperidol
D) Clozapine

Case about haemothorax. What to do?


A) Thoracentesis
B) Drain tube
C) 100% O2
D) Send home and reschedule exam for next day

Patient with flank pain that radiates into the groin. Abd CT found 2cm stone in upper pole of the kidney.
What to do?
A. extracorporeal lithotripsy
B. Wait to pass it by himself
C. Alfa blockator and wait
D. Shock wave lithotripsy

2y.o. Boy vomits after feeding, cries, lost 200g in previous week. Blood analysis shows metabolic alkalosis.
What is the investigation?
A) CT
B) Barium enema
C) Ultrasonography
D) Endoscopy

) 5y boy is brought in by his parents. They state that he’a been staring at the wall and rolling his eyes,
moving his hand and twitching his neck. Boy is confused after the episode that lasts 1-2 mins. What is the
therapy?
A) Ethosuximide
B) Valproate
C) Carbamezepine
D) Phenytoin

Complex partial seizure =temporal lobe epilepsy …but from jm no clear feature

Pregnant lady comes at 16th week of gestation. What to do?


A) Amniocentesis
B) MSST
C) Do nothing and see her at her regular 20 week visit
D) OGTT
10week pregnant lady comes first time to antenatal check up. You find out that she smokes
marijuana, drinks 10 drinks every weekend, uses heroin and occasionally amphetamines and ecstasy.
Which one of these is most harmful( teratogenic) for the baby?

Patient arrives with jaundice, pain in the epigastrium. They gave blood analysis and electrolytes. And
there was more text about symptoms but I cant remember all. What is it?
A) Hepatocellular ca
B) Pancreas pseudocyst
C) volvulusManage


Newborn apgar score 8 in 1st and 5th minute. Both because of cyanosis. Given 100% oxigen with no
improvement. Diagnosis?
A) Respiratory distress syndrome
B) Aspiration of meconium
C) Transposition of the great vessels (from HB )

55 y lady gave up her university studies to pursue career in alternative medicine. Doesnt have close
friends, fights with neighbours but enjoys company of individuals to whom she reads future
from the cards. What is the diagnosis?
A) Shizotypal disorder
B) Borderline personality disorder
C) Dependant
D) Bipolar disorder

Hellow all, first of all i apologize i couldn’t able to remember that much
questions.Trying to recall some of the questions those made me confused.
Exam was not so good, not so bad, got around 40% recalls .. Please keep me in
your prayers.
1.young female 7 weeks amenorrhoea, how to confirm?- b hcg

(4 wks )

2.pt was going to stent operation. After 4/5 days got a swelling in the right
groin 6 cm painfil,pulsatile,what to do?-antithrombin inj to the mass –
Compress by prob

3.boy wearing girlfriend’s underwear. What is this?- female entrapted in a


man’s body - Transvestism

4. 80 years old women in a seperated ward,nurse says she found masturbating


herself frequently..what to do?- a)involve social worker ,b)tell her it is not
good, c)send her in a 4 patient room in ward,d)tell nurse pt needs privacy

5.4 year old boy wants to b his fav cartoon character,momns concern he talks
to his imaginary friends,he sleeps with his fav panda, dry for the day but
sometimes got wet his bed at night.What it this?- a)tell mom this is normal in a
4 years old b)autism c)aspergus

6. African circumcission lady antenatal checkup..a)c/s b)basic antenatal care


c)surgery for repair the circumcision

7.4 years old boy come to chamber doesnot want to go to school. When asked
his name he says tt.....tt..... tom , when asked do u have any friend, he says, i...
d....dddd....do........ he smiles and waves hand when he left your room..what is
this condition? A)autism b) stuttering c)aspergus d) tourette

8)UDCA ques

40 yr fit n healthy woman came to OPD with incrasing jaundice, itchy skin n
dark color urine for a week otherwise no symptoms. On exam: she is obviously
jaundice and scratch marks present. 4 weeks ago, she has URTI and treated
with Amoxil-calvulonic acid, no other history of hospitalization and no
medication hisrory. Lab value ALP 410 BIL 80 AST 150 ALT 180 GGT increased
USG: biliary USG IS NORMAL. what is your management for the patient?
Activated charcoal Observation N acetyl cystein Prednisolone Ursodeoxycholic
acid (UDCA)

9)ECG of RBBB, pt was on antihypertensive,diabetes and now had drowsinees


for last 5/6 days,..what to do next? A) atropine b)pacemaker c)cease
antihypertensive

M pattern in V1V2

10) pt was on warfarin and other medication as well. Gave a Ct.. what was it?a)
cerbral hematoma b) cerebral infarct

11)pt had surgery,after 12 hours got chest pain. What to do? A)chest
physiotherapy b) echo c) ecg
12)2 ECG 1st one atrial fibrillation that was when pt came to u, after a while did
another ecg,that shows ventricullar fibrillation ecg..what to do? A)
defribrillator B)cardioversion

13)pt had injury to tibia, 5 cm clean, what to do next? A)clean the wound b)
give tetanus toxoid(no thing mentioned about tetanus history or anything)

14)pt had injury to tibia,6 cm lacerated wound,what tod o next? A)wound


debribement a) give tetanus toxoid (nothing mention about tetanus history)

15.pt has muscle weakness,both upper limb and lower limb,hand small muslce
weakness as well..what tod o next? A)EMG b) MRI

16.an 80 years old man killes his wife she was 70 years old they married for
almost 50 years, you found multiple bruises in her body.what was the cause of
her death? A) she has metastatic cancer b) the old man had over valued idea c)
she had extra marritial affair.

17.a man had anti parkinson drug for 9 years. Recently prescribed for
haloperiodol. And got abnormal movement. What to do next. A) decrease dose
of anti parkinson drug. B) increase antiparkinson drug c) decrease haloperidol
d) increase haloperidol

18.man has chest pain. ECG done found MI,you gave morphine, o2 .what
next?a) echo b) i/v nitro glycerine c)heparin 19.man has all the enzyme
increased . AST(sgpt)< ALT(sgot) Bilirubin, what is the cause. A) hep a b) hep b
c) hep c d)CMVe)EBV

20. 9 years old girl had enuresis . what to do next? A) DMSA b) cystoscopy and
urodyanamics c)nothing

21.18 years old lady never had period. For last 3 months she got pain in the
lower abdomen for 3 days and become iiritable before that. GP examine her,
all development normal.what is cause of this?a) hidden ovulation
b)imperforated hymenc) pituitary disorder

22.lady with retroverted uterus, on examintaion the uterus is floating. Want to


concieve . what to suggest?a) do surgery for uterus b)ICSI c) wait for 12
months.

23.women has 4 days history of constipation progressing, no history of any


surgery before,xray findings written, ask cause.. a)ca colon b)adhesion c)stool
impaction no sigmoid in option

24. xray given of ceacal volvulas.


25.16 years old girl live with mom , wants HPV vaccine. What to do? A) give her
b) ask mother c)tell u will get it after sexually active ( vaccine wants –give
vaccine,screening –counciling)

26.young guy came to u and says that he heard on TV they are telling him to
join army as becuase they need him to protect people in IRAQ. What is
important history to reach you diagnosis. A)tv is telecasting about himself b)his
previous school drop history

27.male came with scrotal swelling .you diagnose this as varicocele. How u will
find on exam? A)the mass is around the testis and postive transillumintaion
test b)mass is above the testis and negative tranillumination test c) the mass is
soft and can get above the swelling

28.old man had back pain, sometimes it wakes him up from sleep. No injury
history.what is the cause. A)mechanical back pain b)lumber disc prolapse
29.40 years man had unilateral pain in ankle joint and swelling. Looks dark in
colour. What u be associated with his condition. A) gout b) haemochromatosis
b) arthritis

Pigmented villonodular synovitis

A 35 year old man acutely complained of Right knee pain and swelling right
after playing golf. Arthrocentesis done revealing 25 mL of brown pigmented
fluid aspirated. What is the diagnosis? A. Rheumatoid arthritis B. Gout C.
Osteoarthritis D. Pigmented villonodular synovitis E. Hemochromatosis
Achondrosis

30. 60 years old lady had ankle swelling and pain, what will find aftr aspiration
of fluid from the swelling. A) clear crystal fluid b) hazzy fluid


31.65 years old lady has DM got an ulcer in the 3 rd metatarsal 1 cm yellow
colour fluid coming out. What to do? A) give antibiotic
(Collect the swab if asked next )

b)immobilisation of the limb c)amputation of the finger.

32. an old pt came with abdominal pain. He got peritonitis,the condition was
he needs urgent laparotomy otherwise he will die.He doesnt want to do any
operation, his wife wants to do that. What to do next? A)access his
competency to make wish b) listen to his wife c) do surgery

33. you found one intern is giving wrong dose of treatment in ward,it’s
happening for last couple of weeks. What you will do. A)tell to the intern to
concern about that b) report to the intern supervisor or someone like that c)
tell nurse to help the intern

34.an Uni student come to the psychiatry registrer about his friend telling his
friend has some suicidal thought. What the registrer will do?a) tell the intern
to take his friend to GP b) tell him to contact with university social welfare
people(someone like that) c)access his friend

35.lady has active herpes lession in gentalia. What to do toprotect his husband.
A) treat his husband b) use condom c) treat only herself d)avoid sexual activity

36.lady on carbimazole for thyroid, now going to be pregnant.what to do? A)


continue carbimazole b) stop carbimazole c)decrease the dose

( best propyl thyo uracil and carbimazol can be given from 2nd trimester)
Sample Questions

Melanoma eye picture à refer for plastic surgery

Jaundice started on 6 hours of age à Haemolytic disease of newborn

1. Man presents with one alopecia, patch on scalp and rash in hands and legs. He
recently travelled asia alone and also live in Darwin for 3 months. What is initial
investigation for his condition?
a. Syphilis serology[jm-
b. Thyroid function test
c. Serum zinc level
d. Malaria blood film
e. Don’t remember now and it is blood test too
2. Female 74 years, H/O of productive cough with shortness of breathing, little sputum
for 2 years, progressive dyspnoea with no orthopnoea, H/O cardiac stent for 4 years,
chest lung examination (hyper resonant), come to emergency, 02 saturations 80%
with no distress, fine basal crepitation, diagnosis (Lateral and PA X-ray given, look
like COPD)
a) asthma
b) COPD
c) left ventricular failure

3. Old man, CXR given with opacity in hilar area +??? (I confused between lungs cancer
and acute on chronic bronchitis). He is a smoker and has a history of chronic cough
+. Now, he has 2 weeks history of rusty sputum Diagnosis?

a. lung cancer

b. Pulmonary Fibrosis

c. COPD

d. Acute on chronic bronchitis

e. Heart failure

4. Picture showing red patches of rashes in legs à psoriasis??? Treatment


a) prednisolone (didn’t remember it was given oral prednisolone or just
prednisolone)
b) calcipotriol cream jm 1320
c) antifungal

5. 15 years old girl, came with (fever, sore throat, swollen neck glands) Otherwise no
medical illness, not on any medications, no travel history. Asking (something like)
what symptom will actually tell you the definite diagnosis? (exact option)
a. Diffuse pharyngitis
b. Temperature 38.5
c. Swelling of ankle
d. generalized maculopapura rush
e. splenomegaly

6. Forearm pic of child showing just two small skin lesions given – Impedigo?? Asking
school exclusion??
a. Immediately
b. After lesions dry
c. After lesions resolve
d. After 2 day antibiotics
e. After 7 day home
7. X-ray showing distended bowel I think small bowel obstruction? patient vomiting,
abdominal distension present for 3 days??? (no acute presentation and no pain
mention in my exam question) Rectal exam normal. Patient had AF. Next step?
A. Endoscopy or colonoscopy (forgot)
B. microlax enema
C. ct scan
D. air enema
E. nasogastric tube insertion

8. an infant who was delivered prematurely at 28 weeks with birthweight of 1100


grams. After delivery he was given surfactant and intubated and kept in a ventilator
for 3 days, now the baby present with spastic paresis in his lower limbs. Cause?
a. prematurity
b. respiratory distress syndrome
c. intrapartum hypoxia
d. low birth weight jm 1046
e. intrapartum infection

9. Testicular swelling likely hydrocele??? scenario, testes is soft and asking what
investigation to do?
a. Ultrasound
b. AFP and beta HCG

10. Confused question with lab results – Urea and creatinine increased others normal
range. Patient come with dizziness or confused? à I choose haemodialysis (other
options contain oral supplement like iron, folate)

11. Patient come with sudden painless vision loss. Fundoscopy describe as CRAO???
Scenario said vision loss in right eye improved after some time. Cause?
a. CRAO
b. Carotid Disease
c. Retinal detachment
d. CRVO

12. Picture of red eye with scenario said acute painful red eye, no discharge. Which
eyedrop is used for this condition. (exact option)
a. Timolol
b. Prednisolone
c. Chloramphenicol
d. Atropine
e. saline
13. Female patient with RA she takes ibuprofen and methotrexate to control her disease
for 5 years. Recently patient complains of some symptoms (jaundice), and her labs
are given.
protein - 9 (6-8) slight increased
albumin- 5 (3.5-5.5) normal
ALT - >100 (7-56) Liver enzymes >100 in my exam question
AST - >100 (10-40)
GGT - >100 (0-30)
What is the cause of her symptoms?
A- Methotrexate induced hepatitis
B- ibuprofen induced hepatitis
C- Autoimmune hepatitis
D- Viral hepatitis

14. Alcoholic patient present with foul smelling cough, high fever with rigor. X-ray was
given and it showed opacity in middle zone with air fluid level, scenario said staph
aureus and after giving appropriate antibiotic, what next?
(no option of aspiration in my question)
a) transpleural drainage
b) underwater sealed chest tube with pleural drainage
c) CT chest

15. The same red eye pic but this time scenario said Redness of eye with oral and
genital ulcer:
A. topical chloramphenicol
B. oral chlorampenicol
C. topical steroid
D. oral prednisolone ( bechet disease )

16. 16 years old Girl came to clinic, Father diagnosed Huntington disease. She wants to
test without parents wish. What should you do?
a) Arrange Genetic counselling her alone
b) arrange counselling whole family
c) order genetic test for her
d) tell her to come back 18 years of age
e) tell her to get permission from parents

17. Lady with 24 hours of muscle weakness and tingling sensation +, weakness more in
lower limbs, no other sensory loss, no other symptoms mentioned
a) acute inflammatory polymyopathy
b) multiple myeloma
c) myastinia gravis

18. Baby 6 weeks age, persistant vomiting (non bilious) since 2 weeks and constipation,
abdominal examination can’t be done perfect as baby was crying, but seems normal.
what to do?
a) add food thickener
b) USG abdomen
c) oral rehydration therapy
d) reassure it is normal in this age ( pyloric stenosis )

19. Woman known with polymyalgia rheumatica and has prednisone 10 mg daily,
antihypertensive medication for HTN and metformin for DM, she feels dizzy and
unwell as she went overseas and didn’t take medication at all. Labs- Glu 4.0, Na 125,
K 5.0 diagnosis??
a) Adrenal insufficiency
b) indapamide
c) metformin
d) lisinopril
(Adrenal insufficiency is a condition in which the adrenal glands do not produce
adequate amounts of steroid hormones, primarily cortisol; but may also include impaired
production of aldosterone (a mineralocorticoid), which
regulates sodium conservation, potassium secretion, and water retention.[1][2] Craving for
salt or salty foods due to the urinary losses of sodium is common.)
20. Patient after surgery present with agitation, tremor, restless. Previous medication
contain oxycodone and others drugs. HT + and history of drinking one glass of wine
everyday before. Current medication list mention and didn’t contain oxycodone.
a) Benzodiazepam withdrawal
b) Alcohol withdrawal

21. women with 2 years after Filshie clip sterilization. Regular cycle with menorrhagia +,
At this cycle, bleeding for 10 days with few clots. Pap smear normal 12 months ago.
Initial investigation? (exact option)
a) pap smear
b) Full blood count
c) ultrasound
d) Thyroid function test
e) Endometrial Biopsy

22. Child present with fever, neck stiffness, anterior frontanalae full, lymphocytes
5400cumm (n <5), glucose is reduced, no organism on gram stain, After giving
antibiotic and (fluid?), What will need to give? (exact option)

A. Intravenous phenytoin

B. Rectal diazepam

C. Intramuscular morphine
D. Oral paracetamol

E. Intravenous dexamethasone

23. Indian university student, 2 months dysuria, hematuria, frequency. All urine test
normal except RBCs and WBC ++ in urine. Urine culture (-). Dx?
a) Cystoscopy
b) Renal biopsy
c) Urogram ( dx : cystitis , after msu we go for IVU )
d) Repeat urinalysis
e) Chlamydia PCR

24. Man with lack of interest in normal life, insomnia + and is having visual hallucination
and voices in his head +, he is saying my mother died last month but it is not the
case and mother died year ago, what is most likely diagnosis?
A. Personality Disorder
B. Depression with unresolved grief
C. Major depression with psychosis
D. Schizophrenia

25. 75 years old man present with severe retrosternal chest pain. pain is preceded by
vomiting. with BP 100/70 mmHg, dull on percussion and reduced breath sound at
base of the lung. he has HTN DM asking most diagnostic test?

1. non-contrast CT
2. electrocardiogram
3. chest X-ray
4. Gastrograffin swallow

. Patient got operated for appendicectomy. There was carcinoid incidentally detected which
was resected along with appendicectomy. Persistentdiarrhea present ever since preoperative
period. No other symptoms of carcinoid. Wat is ur initial treatment of choice.

A. Octreotide
B. Prednisolone
C. Loperamide
D. Cholestyramine
it is a case of persistent diarrhea, it may be because of carcinoid too. But there was clearly
no other symptoms n signs of carcinoid. 1 St choice of drug in a case of persistent diarrhea is
loperamide. If it persists after that we may try octreotide which is especially used for
management of carcinoid symptoms.

(indications for SANDOSTATIN/octreotide )


Severe diarrhea and flushing due to metastatic carcinoid tumors. Profuse watery
diarrhea due to vasoactive intestinal peptide-secreting tumors (VIPomas
26. Hand x-ray showing RA features. Diagnostic investigation asked ?
a. anti CCP
b. ANA
c. Anti Ds DNA
d. RF

27. farmer present with depressive symptoms and angry due to drought. Spend money
recklessly in women and accusing government for the loss due to drought. He said
he is just exhausted and not depressed. Which kind of mechanism is he
demonstrating? (exact option) I confused this one.
a) Denial
b) Reaction formation
c) Projection
d) Displacement
e) Rationalisation

(Repression is the withdrawal from consciousness of an unwanted idea, affect, or desire by


pushing it down, or repressing it, into the unconscious part of the mind. An example may be
found in a case of hysterical amnesia, in which the victim has performed or witnessed some
disturbing act and then completely forgotten the act itself and the circumstances surrounding it.

2. Reaction formation is the fixation in consciousness of an idea, affect, or desire that is opposite
to a feared unconscious impulse. A mother who bears an unwanted child, for example, may react
to her feelings of guilt for not wanting the child by becoming extremely solicitous and
overprotective to convince both the child and herself that she is a good mother.

3. Projection is a form of defense in which unwanted feelings are displaced onto another person,
where they then appear as a threat from the external world. A common form of projection occurs
when an individual, threatened by his own angry feelings, accuses another of harbouring hostile
thoughts.

4. Regression is a return to earlier stages of development and abandoned forms of gratification
belonging to them, prompted by dangers or conflicts arising at one of the later stages. A young
wife, for example, might retreat to the security of her parents’ home after her first quarrel with
her husband.

5. Sublimation is the diversion or deflection of instinctual drives, usually sexual ones, into
noninstinctual channels. Psychoanalytic theory holds that the energy invested in sexual impulses
can be shifted to the pursuit of more acceptable and even socially valuable achievements, such as
artistic or scientific endeavours.

6. Denial is the conscious refusal to perceive that painful facts exist. In denying latent feelings of
homosexuality or hostility, or mental defects in one’s child, an individual can escape intolerable
thoughts, feelings, or events.

7. Rationalization is the substitution of a safe and reasonable explanation for the true (but
threatening) cause of behaviour. )
28. Concern women come to you asking for antibiotic for her son who has fever, her son
visited child clinic which was visited by a girl documented to be meningococcal
meningitis, the girl visited the clinic from Monday to Wednesday, but her son visited
on Friday. Hospital gave all the contacts of girl rifampicin as prophylaxis, what to do?

a) treat her son

b) giver her son rifampicin

c) ask her to get her son ER for review

d) provide information about symptoms to mother

e) nothing need to give like reassure

29. Middle aged man who is hypertensive and just treated for his helicobacter pylori with
triple therapy. Now comes with urine protein and hematuria with 3 cysts in right and 2
cysts in left renal cysts seen on USG. Diagnosis?
a) Polycystic kidney disease
b) IgA Nephropathy
c) acute tubulonephritis
d) Acute tubular necrosis
e) Acute glomerulonephritis

30. Patient after pacemaker insertion presents with difficulty in breathing and chest pain.
faint heart sounds +, spo2 96%, bp-90/60mmhg, JVP-6 with no ECG given.
Investigation of choice needed.
A. Chest X-ray
B. CTPA
C. ECG
D. ECHOCARDIOGRAM
E. SERUM TROPONIN

31. Diabetic patient well controlled with mild renal impairment (given Creatinine level but
I forgot). Which of the following risk will have in baby?
A. IUGR
B. Macrosomia
C. Renal agenesis
D. Intrauterine fetal demise
E. Low Birth weight

32. Old age, History of appendicetomy & cholecystectomy presented with 3 weeks
history of abdominal distension on ascending colon, transverse colon, descending
colon, rectum is empty. There is mild tenderness of the abdomen and loud
borborygmi. What is the diagnosis? (CT was given) CT unclear for me à I choose CA
sigmoid according to scenario.
A- Sigmoid volvulus
B- Adhesive IO
C- CA Sigmoid
D- Fecal impaction
E- Caecal vovulus
33. The same scenario without CT. option same.

34. Insufficient milk production, reason?

a) insufficient fluid intake


b) not putting for sufficient frequently
c) not putting for long time during feeding
d) exhausted
e) poor positioning

35. Old female with depression for 6 wk after dead of her partner ,sleeps badly, she lives
alone now , she had the same symptoms in the past when she lost her child , what is
the prompt Rx?
A)Olanzapine
B) ECT
C) midazolam
D) venlafaxine

E) temazepam

36. A woman after removal of central venous line lady developed facial swelling and
swelling around the neck. What’s the most appropriate investigation?
a. CT chest
b. CT neck angiogram
c. Chest x-ray
d. Neck USG Doppler
e. Echo
SVC syndrome.obstruction due to hematoma
.neck vessels injury during removal of CVP line
initial test usg duplex..best is CT neck

37. 35 year old lady comes to you regarding screening for breast cancer screening. She says
her paternal aunt was diagnosed with breast and ovarian cancer at the age of 60 years
which was diagnosed as being associated with BRCA1.Which of the following is the most
appropriate advice?

A-Genetic counselling assessment


B-refer for BRCA 1 screening
C-screen now
D-tell her to look at her breast
E-annual mammography

38. 39 weeks pregnant lady came with labour pain…on iv fluids with syntocinon is running
5 unit in 500 ml of Ringer latate.with irregular uterine contractions. Ctg was done
which shows heart rate of 140, which dropped to 70 and came back to 140 in
2min.what is the most appropriate next step.
A. give oxygen to mother via face mask
B. stop syntocinon
C. C section
D. fetal scalp sampling
E. Titrate to increase dose of syntocinon

39. Lady full term with adequate contraction was monitored on CTG. Everything was
progessing normally. Suddenly there was a deceleration of FHR dropping to 70bpm
for 4 mins. O2 is given to mother, what is the next most appropriate step?
a. Stop syntocinon
b. Continue synto at an increased dose
c. Immediate cesarean
d. Ventouse delivery

40. Pregnant women came as her friends told her take precautions in her usual diet.
what she should not eat??
a. tinned salmon
b. coffee
c. soft cheese
d. fresh veg
e. Iodine

41. patient has acute gout with h/o chronic renal failure and previous h/0 gout. what
should be given-
a. allopurinol
b. NSAID
c. colchicine
d. ibuprofen
e. prednisolone

42. Gout scenario with chronic renal failure and ccf, now presents with acute symptoms,
what to give? (no prednisolone in option)
a) paracetamol
b) codeine
c) naproxen
d) allopurinol (. Main side effects are a rash with an incidence of 2% and
hypersensitivity manifesting as renal failure, hepatitis, exfoliative dermatitis and
Stevens–Johnson syndrome -appropriate doses of allopurinol for patients with
reduced renal function.

Others drug have side effect on heart …)


e) colchicines
43. An 18 year old girl presents with epistaxis, malaise and tiredness for months. no
other symptoms . Hb is 8.5, microcytic hypochromic picture. INR is 1.5. Calcium is
1.9. What is nex most appropriate to reach diagnosis
a. Hb electrophoresis
b. serum electrolytes
c. Iron studies
d. anti glidian antibods
e. stool culture

44. 60yrs old male,10 yrs history of hypertension,feeling dizzy and light headache from
this morning when he was doing exercise.smoke 20 cigarettes per day.drinks 2-3
glass of wine most of the night.BP 138/80 sitting 120/75 on lying.Currently on
ibesertan-thiazide 150/12.5 Mg and ASA 100 mg. ECG was given (SVT???) HR
around 150/min. Cause of his dizziness.
A) Dehydration
b) ischemic heart disease
C) hypertension
D) alcoholic cardiomyopathy

45. Child presented with UTI, on USG the right kidney is smaller than the left. What is
the best investigation to assess renal function?
a) DMSA
b) DTPA
c) Urine culture
d) CT scan

46. A man aged 68 years. his psa is 3.8ng now. last 2 year( sure) it was 1.5.he did 12
biopsies this time and one showed a focus of adenocarcinoma and gleason score 4.
What is the management for this man?
A) radical prostatectomy
B) external beam radiotherapy
C) orchidectomy
D) continued surveillance

47. Farmer came with left swelling in a groin. U did FNAC and it shows squamous cells.
Where is the lesion?
A. Left leg
B. Anus
C. Rectum
D. testis
E. penis

48. A mother come with her 8 months old child complaint of continuous bleeding after
fall from coffee table. On examination there is bleeding frenulum and some old
bruise in forehead and leg. There was no petechiae, lymphadenopathy,
hepatomegaly. What is the cause?
A. ITP
B. Non accidental injury
C. VWD
D. Hemophilia A
E. Hemophilia B

49. child come to your clinic. he play for a few min with each toy. Hyperactive, can
concentrate only a few minutes. What is dx?
A. ADHD
B. Autism
C. Asperger
D. Oppositional deficient disorder

50. Study on Aspirin effects on preventing MI on 100 people. If those who took aspirin of
the 100 ppl only 1 person had a MI. Of those who didn't take aspirin 2 ppl had MI.
What is the increase in relative risk in people not taking aspirin?
a. 1%
b. 10%
c. 100%
d. 200%

51. scenario with female patient always angry, and can’t control herself, unstable
relationships. Self-harm was there, treatment was asked
a. anger management
b. dialectal behavioral therapy
c. Psychoanalytical therapy
d. Interpersonal therapy

52. Rural doctor wants to make study about the relationship between admission to
hospital with rota virus infection and birth weight, what’s appropriate?
a) RCT
b) Case control
c) Cross sectional
d) Cohort
e) Case series

53. 40 years old lady with back pain at L4 level with severe shooting pain to leg to toe
next, no neurological deficit ,best ?
A. Observation
B. X-ray lumbosacral
C. CT lumbosacral
D. MRI lumbosacral
54. An old man with acute onset pain in the lower back with urinary symptoms.
significant weight loss. On DRE, irregular prostate +. What is your appropriate
investigation to lead you to diagnosis?
a. chest x-ray
b. CT spine
c. TRUS
d. bladder scan
e. PSA
(no option for MRI)

55. A child 6 years old with history of asthma presented with upper respiratory tract
infection, urine examination showed very high serum glucose and ketone body of
2+.
What is the most appropriate test to follow up this child?
A. HbA1C ***
B. Serum creatinine and electrolyte
C. FBS
D. OGTT
E. ABGA

56. 8 years old with type 1 diabetes. On small and intermediate acting insulin. every
morning high glucose levels. what to do?
a. check blood glucose levels at 2:00-3:00 am for 3 days
b. check early morning insulin levels
c. increase the evening intermediate insulin
d. give insulin before breakfast
e. give another dose before sleep

57. Scenario of acute closure angle glaucoma in old age, haloes around light, red sclera,
IOP 25 mmHg, asked long term management?
A. Trabeculectomy
B. Peripheral iridotomy
C. Acetazolamide
D. Pilocarpine

58. Child (3-4 year) with fever 39C and history of cough unwell for 3 days. I think he has
wheeze but minimal lung signs on exam. CXR given. Asked which investigation what
u do for further assisting in diagnosis? (similar with this picture)
a. pneumococcal PCR
b. pleural aspirate
c. blood culture

d. FBC

59. Middle age man, pain in buttock and thigh during 100 m walk on ground, can walk
20 m uphill, femoral pulse not palpable, dorsalis pedis is palpable, ABI 0.3. Best way
to diagnose?
A. Arterial Doppler
B. Digital subtraction arteriography
C. CT angiogram
D. arteriography

60. A man with history of limb claudication on 100 meters relieved by rest, on
examinations there was absent left femoral pulse and absent dorsalis pedis pulse,
ABI was done and it was 0.25. What is the most appropriate test leading you to the
diagnosis?
a. Arteriography
b. Ct angiography
c. compression Doppler ultrasound
d. MR angiogram
e. X-ray

61. old patient who drinks about 10-12 standard drinks per day and also smoker,
underwent some surgery. on 3rd post op day he developed agitation ,O2 saturation
88%.no fever. what next ?
a) blood alcohol
b) blood glucose
c) CTPA
d) XRAY CHEST
e) urine culture
62. 72 year old man living alone is brought to ED by daughter. He’s so confused and
gave incomprehensible answers to questions and couldn’t follow any commands.
There’s no other abnormality on examination. CT was given large dark area is in left
temporo- occipital region with shifting of left lateral ventricles plus surrounding
oedema. Asked diagnosis
a. Cerebral tumour
b. Cerebral infarction
c. Cerebral haemorrhage
d. Cerebral tuberculosis
e. Subarachnoid haemorrhage

63. A 42yo woman who smokes 20 cigarettes/d presents with complains of heavy
bleeding and

prolonged menstrual period. What is the most appropriate treatment for her?

a. Tranexemic acid

b. COCP

c. Mefenemic acid

d. IUCD

e. Norethisterone

64. young girl Came with menorrhagia. Absent from work because of menses pain &
heavy flow.
From history & exam, cannot find cause. You ordered several lab investigations but
she insists to give medication. Her cycles were regular
A- OCP
B- Oestrogen patches
C- Mefenamic acid during period
D- tranexamic acid during period

65. 45 yr old lady smoker and htn uses ocps for many years now comes re prescription
,she says doesn't want to come off because it has controlled her menorrhagia and
also says doesn't want to concieve. What to do now
A. Tell her stop smoking then can give ocps
B. Use implanon
C. Use copper iud
D. Use progestogen only pill
E. mirena

66. Woman with a history of hysterectomy and DVT. Now complaining of hot flushes and
insomnia. What will you give her?
A. Oral low dose estradiol
B. Transdermal estradiol
C. Progesterone only pills
D. HRT
E. Clonidine

67. 35 year old pregnant woman presented at 15 weeks gestation asking you for a
diagnostic screening of down, what is the diagnostic test you should do for this
woman?
A-nuchal translucency ultrasound
B-maternal serum and ultrasound
C-maternal serum
D-amniocentesis
E-chorionic villous sampling

68. mother with hepatitis C, what will reduce vertical transmission?

A. Monitor fetus with scalp clipping should be avoided

B. Caesarean section is indicated

C. Avoid breast feeding

69. To see prevalence of HCV in child born to HCV positive mother, which of the
following can be used?
A. HCV PCR in neonate
B. HCV Ab just after birth
C. HCV AB at 18 month
D. HCV PCR in mother

70. Patient with deep jaundice, moderate ascites, and confused complaining of
abdominal pain due to tense ascites. Flapping tremors and hepatic encephalopathy
feature. Liver is difficult to palpate. Complaint of high fever 38’C. What is your next
appropriate initial step?
A- abdominal paracentesis
B- Blood culture
C- USG
D- Fluid restriction
E- IV antibiotics

71. scenario where a girl came with menorrhagia ,looking pale and hypocalcemia there
lab value was INR -1.9 with no S/s.bt another thing was there was micocytic
anemia,asking for what to do?
1) hemolytic screen
2) iv calcium injection
3) blood electrophoresis
4) give vit k injection
5) give FFP ( celiac disease – menorrhagia )
72. One ECG picture (no obvious changes, a bit tall T in V2,V3). Serum potassium 5.5,
patient has no symptom. He is on Ramipril and other drugs. Asking what to do next?
a. IV glucose
b. Cease Ramipril
c. Iv Calcium gluconate
d. Resonium

73. 7 days old infant presented to you complained with poor feeding and bile stain
vomiting. His birth was at 38 weeks gestation and weight 2600g.He is breastfeeding
with no immediate post natal complications. The child has a history of passing
meconium on day 4 of birth. Now, he had mild jaundice and abdominal distension
present. What is the appropriate diagnosis?
A. Meconium Ileus
B. Hirschsprung's Disease
C. Duodenal Atresia
D. Necrotizing Enterocolitis
E. Volvulus
74. Lady comes on 10th post-partum day with pain in perineum tear. History of
perineum laceration during delivery which was sutured. Now on examination there is
a perineum wound 2 cm long 1 cm wide and 1 mm deep. Clean with no discharge.
What is next?
A. Apply local antibiotics
B. Put on oral antibiotics
C. Simply keep the wound clean
D. Suture the wound under LA
E. Suture the wound after cutting the edges

75. 3yr old boy with ear discharge for 6 weeks no fever. Asking treatment after ear toilet
is done.
a. Po Amoxil
b. Ciprofloxacin ear drop
c. Other antibiotics

76. Woman around 30-35 years of age attends for Pap smear, Her previous smears are
all normal. She doesn’t have sexual activity in last two years. What’s the reason of
testing chlamydia in her?
A. chlamydia is asymptomatic
B. chlamydia can cause infertility
C. Both partners could be asymptomatic
D. No need for screening

77. A 6 month old child came with recurrent upper respiratory infection. The
immunization history reveals that the child has received his hepatitis B vaccination at
his birth and no further immunization history at 2 and 4 month of age due to
recurrent upper respiratory infection. Now the child has fever 37.8 and clear nasal
discharge. What is your management regarding immunization?
A. Give hepatitis B immunization and institute catch up schedule
B. Give hepatitis B immunization and recommence standard schedule
C. Investigate underlying cause of URTI
D. Give hepatitis B immunization after fever subside
E. Give hepatitis B immunization at now

78. Lawyer who is a perfectionist, recently fall in performance after miscarriage, affect
sleeping, crying, can’t concentrate her work and problem with husband. don’t want
to take medicine coz it might be habit forming. what is suitable management at this
stage?
a) CBT
b) SSRI
c) Interpersonal therapy
d) Diazepam
e) Dx
79. A case of gynaecomastia pt on risperidone.
What other drug to counteract the side effects
A. Quetiapine
B. aripipazole
C. olanzapine
D. Clozapine
E. Mirtazipine

80. Man with severe depression treated with venlafaxine now presented with agitation
pressured speech euphoria and mania ask what next appropriate thing to do
a. add sodium valproate.
B. add clozapine
c. add olanzapine
d. Withdraw venlafaxine
e. add risperidone

81. Woman 51 years of age presents after menopause for the last 8 months. She
complains of irritability, moodiness, low libido and hot flushes. Which of the following
would be most appropriate to give to this woman?
a. Combined continuous estrogen and progesterone throughout the cycle
b. Continuous estrogen throughout the cycle
c. Continuous estrogen with progester one first 12 days of the cycle
d. Continuous progesterone with testosterone

82. 8yr old child brought by his mother , he complained from episode of staring suddenly
that occur along with fidgeting of right hand and movement of right arm head
twitching to right side, sometimes chewing and lip smacking each episode last for
60-90 second then the child remain dizzy and confused for 1-2 minutes after the
episode. These occurs in 3-4 days and then the child back to his normal activity and
behaviour for several weeks. what is the most likely diagnosis?
A- Temporal lobe epilepsy
B- Juvenile myoclonic epilepsy
C- Absence seizure
D. Hypsrhythmias
E. Rolandic seizures

83. Old lady 58yrs with urinary incontinence, when she makes some stress such as
cough, laughing, urine flow out. On examination she has only rectocele, no other
cystocele. told to do pelvic floor exercise and a little improved but still incontinence
and embarrassed. She is not satisfied. She had DM and well controlled with diet.
What is the next most appropriate treatment in this patient?
A. Weighted vaginal cone
B. Bladder neck suspension
C. Imipramine
D. Oxybutyline
E. Surgery for rectocele

84. Family history of father and paternal aunt died at 50 years of age. Patient complaint
of wide based gait and verbal dysfunction. What is your next management? New
question
a. Genetic counselling[dx-huntungton]
b. Lumbar puncture
c. Serum copper and ceruloplasmin
d. MRI
e. CT head

85. Old man presented to you complaining of rest tremor and bradykinesia, also taking
respiridone and he has a known history of visual hallucination and forgetfulness.
what is the most likely dx?
A. parkinson disease
B. lewy body dementia
C. alzheimer disease
D. acute delirium
E. schizophrenia

86. old age lady with polymyalgia rhematica. on oral pain killers like codeine paracetamol
with dose given. one of her friend who had end stage cancer and was on morphine
gave her morphine and she was using it. now friend has died. now lady has come for
prescription of morphine. what will u do?
a. give her prescription
b. refuse her request
c. report her drug seeking behaviour
d. increase the dose of her oral analgesics

87. 24 months old, developmental task for language?


a. 2 or more personal pronouns
b. Sentence with 4 syllables
c. Name 4 color
d. Know age
e. Know family name

88. Female worried about baby. How to prevent SIDS?


a. Sleep with baby
b. Keep baby in supine position

89. A student thinks that he has brain tumor. The final exam is near and he gets more
tired, headache and stressed due to tumor. All neuro exam and CT are normal. After
CBT, he still think he has brain tumor. What is the diagnosis?
a. Hypochondriasis
b. Somatisation disorder
c. Conversion disorder
d. Factitious disorder

90. Patient with weakness of the left upper limb, weakness of interosseous muscles and
right plantar response is equivocal, left is increased; reflexes are normal. What
investigation will you do to reach diagnosis?
A. MRI cervical spine
B. Ach receptor antibodies
C. EMG
D. CT brain

91. A young man brought to ED after brawl in bar in which he hit someone. He is
alcoholic, take multiple drugs and aggressive. What in history will you to know if he
has personality disorder?
a. Childhood sexual abuse
b. H/o cruelty to animals in adolescence
c. Drug and alcohol dependence
d. H/o of hitting partner one week back
e. H/o depression in mother

92. a man works whole day in a kitchen cabinet, he complains of pain in elbow.
Tenderness in lateral side of elbow. what the cause of this pain.
a. bursa inflammation
b. tendon rupture
c. fracture
d. lateral epicondylitis

93. Patient complaining of soreness on lateral epicondyle at the end of work,the patient
attributes it due to computer mouse clicking. On examination , tenderness on lateral
epicondyle .Asking treatment?
A-lateral epicondylectomy
B-finger immobilization splint
C-lateral epicondyle immobilization splint
D-bracing under lateral epicondyle
94. Patient is taking methotrexate, prednisolone and celecoxib for RA. Lab results show
pancytopenia with high esr (67). Management?
a. Increased prednisolone
b. HCQ
c. Folic acid
d. Folinic acid
e. Cease celecoxib

95. Picture showing rashes on hands of a man. Patient on amiodarone and many drugs.
Developed maculopapular skin rash on hands and face after 2-3 days of started on
amiodarone. Asking cause?
a. Fixed drug eruption
b. pseudoporphyrin
c. phototoxicity
d. pemphigus

96. 3.2 kg baby was delivered vaginally with no complication except for slight meconium
stained liquor, normal fetal heart rate at birth, after one minute, become limp,
cyanosis, decreased HR, what is most appropriate next step?
a. Intubate
b. Bag and mask ventilation
c. Aspiration for meconium
d. CXR
e. Na bicarbonate to infant

97. CT scan given, chest MVA with severe chest pain and difficulty breathing due to pain.
Tenderness present on both side of chest. Decreased breath sounds bilaterally and
dullness to percussion. Asked about the cause of pain. Diagnosis?
a) Fractured ribs
b) Hemothorax
c) Pneumothorax
d) Pneumomediastinum

98. A young girl who is 9 years of age with history of recurrent URTI last time was two
weeks ago and did urinalysis at that time showed blood (3+), proteinuria (1+). Now
she came after two week. Urinalysis is unchanged except for Blood (2+) which is
non-glomerular and there are no casts in the urine. Which of the following is the
next best investigation?
a. Urine culture
b. DMSA
c. Ultrasound
d. IVP
e. ASOT
99. 65 years old lady took handful of antidepressant, on 3rd day of hospitalisation she
became agitated, confused then later nurse found her staring at ceiling.
a. Acute psychosis
b. Delirium
c. Dementia
d. Catatonic

100. Young man after a quarrel had a fracture of floor of eye what is the most consistent
symptom with that?
a- Conjunctival haemorrhage
b- loss of visual activity
c- anaesthesia around the cheek
d- Cant open the mouth completely
e- Epistaxis

101. Lady with drag left foot, reflexes on left lower limb are increased, planter flexion &
dorsiflexion are 4/5, weak ankle movements, equivocal plantar reflex, upper limb and
face are normal.
a. Cerebral cortex
b. C spine
c. L5, S1 nerve root
d. Common perineal nerve
e. Brain Stem

102. Aboriginal woman, needs hospitalization but refuses to get admitted, wanted to
show her condition to a traditional healer. What do you do –
a. inform the police
b. admit her without her consent
c. involve an indigenous officer
d. arrange a meeting with the healer

103. pregnant woman at 12 weeks gestation presents with heavy bleeding had an
ultrasound Nuchal translucency at 11 weeks Abdomen rigidity, tenderness. Maternal
hypotension and tachycardia. Asking diagnosis
a. incomplete abortion
b. threatened Abortion
c. Ectopic pregnancy
d. Placenta previa
e. molar pregnancy

104. The woman who is working as baby sitter is 7 wk pregnant, the child is confirmed
to get rubella. She is concerned. What to advice?
A. Check rubella IgG
B. Vaginal USG
C. Chorionic villous sampling for rubella PCR
D. Vaccination for rubella

105. scenerio of a schizophrenic patient walking naked in streets. which of the following
is LEAST effective for this patient.

a. olanzapine

b. quietiapine

c. amisulpride

d. clozapine

106. Pt comes with haematuria CT given showing RCC asked what will increase the chance
of DVT after his operation?
a. Nicotine stain of fingers
b. Atrial fibrillation
c. Bilateral varicose veins
d. BMI
e. Spider naevi on chest

107. enlarged gall bladder with multiple stones, found dilated bile duct and stone
obstruction in common bile duct, patient developed toxic shock, what to do after
fluid resuscitation?
a. laparotomy cholecystectomy
b. laparoscopic bile duct explores
c. percutaneous bile drainage
d. endoscopic decompression

108. Young man who is previously healthy , also a marijuana user was brought to ER. He
is v active now and said he has been attack by Underground gangsters and police is
also following him. His friends said he has seen many times using amphetamine at
parties. Which one of the following is is imp risk factor?
1) Cardiotoxicity from amphetamine use
2) Gangsters are after him
3) Police
4) co morbid drug abuse

109. 72 years old Man come with low back pain. On investigating X ray, found L4-L5
compressed fracture. Many lab investigations: low Hb around 85 , mild leukocytosis ,
ESR 108, What will you do to know the cause of fracture ?
A. PSA
B. Bone marrow examination
C. MRI spine
D. Spine X ray
110. lady is going to be grandmother ,came for flu vaccine, had pneumonia.
vaccine 5 or 6 year ago, asking advice which vaccine to have to keep
baby safe
a. flu vaccine
b. pneumonia vaccine
c. Dtpa
d. Typhoid

111. Girl with excessive eating and self-induced vomiting. No BMI given. What can be
seen in her family history?
a. Obesity
b. Elite athletism

112. A 49 year old female presented with progressive back pain. She has a history of
Breast Ca and lumbar osteoarthritis. What of the following clinical features will help
you decide the investigation (MRI) to do?
a. Past history of breast Ca
b. Back pain without trauma
c. Past history of vertebral osteoarthritis
d. High blood pressure
e. Radicular Pain radiating to the buttock

113. Woman whose son is a drug addict, beats her every day. She is afraid and presents
to you. What will u do for the safety of the woman?
A. Tell her that you are obliged to inform the police
B. Go to the police
C. Tell her to go to refugee shelter
D. Advise her to go support group
E. Send the son to correctional facilities

114. Child with ear infection. Cause?


a. H. influenza
b. Streptococcus pneumoniae
c. Staphylococus aureus

115. 3 year old with croup was improving. A day before discharge developed stridor,
chest retraction, fever 39 chest auscultation normal. Cause
a. Epiglotitis
b. Bacterial tracheitis
c. Pnemonia
d. Pneumothorax
e. Rsv
116. A case of chronic liver disease presented with ascites and low albumin level. What
is the next appropriate management?
a. lactulose

b. albumin infusion

c. salt and fluid restriction

d. spironolactone and frusemide

117. A patient with rheumatoid arthritis got a flare up and doctor wants to administer
azathioprine. which screening test should be done before starting this drug?
A. Tb Gold test
B. Thiopurine methyltransferase
C. Anti-Acetylcholine inhibitors
D. Complements assay.
E. Anti-gliadin IgA

118. Man 1 week after prostatectomy, his wife came to consultation because she is
concerned her husband is acting different, more irritated, aggressive, shouting. what
in the history will help u reach diagnosis?
A. Worsening symptoms at night
B. marked interpersonal aggression
C. history of cruelty towards animal
D. history of drug use

119. 18 year old male with blood mixed with stool. stool culture negative. Upper GI
endoscopy and Colonoscopy showed nothing. Most appropriate next?

a) Ct angiogram

b) X ray abdomen

c) Capsule endoscopy

d) repeat colonoscopy

120. Similar scenario with Upper GI endoscopy and colonoscopy showed nothing. I
choose Capsule endoscopy too.

April 19 2018 SYDNEY



st
1.1 POD aft surgery for incareceratedhernia..patient irritable aggiatated has fever O2 sat
88% what next appropriate after giving O2
A. IV antibiotic
B. heparin
C. thrombolysis
D. droperidol

2.GBS diagnosed already.how to monitor respiration
A. FEV1
B. FVC
C. chest expansion

3.old lady has fever agitation brought to hosp n found to have UTI. Started on
trimethoprium.she was taking carbamazepine since 2 wks n on PTU fo thyroid since years
and some antiHTNives.on labs now Na.120 rest all normal.
A. stop trimethoprium give NS
B. stop carbamazepine n give hypertonic saline
C. stop carbamazepine n restrict fluid
D. stop PTU n give hypertonic saline
E. continue same tm

4.central chest pain increasing on inspiration.ECG given seems like acute pericarditis n
patient had fever what next after 02
A. heparin
B. nsaid
C. cath lab
D. thrombolysis

5.pt chest pain 12hrs now pain decreased 4/10 after O2 what next
A. morphine
B. GTN
C. heparin
D. thrombolysis

6.Xray given sarcoidosis SOB n hematuria 2 months.labs Ca high rest all normal.next
appropriate investigation
A. CT chest
B. ACE levels
C. urinary Ca levels
D. antigbm

7.scenrio of a schizophrenic patient walking naked in streets.which of the following
is LEAST effective for this patient.
A. olanzapine
b. quietiapine
c. amisulpride
d. clozapine

8.Encoparesis 7yrs girl fighting with brother when he call her smelly
A. Normal developmnt
B. Delayed developmnt
C. ODD
D. Regression
E. Depression

9. Man smoker presented with tiredness lethargy cough dyspnea blood stained
sputum
Labs shows
Hb 104
Tlc 40
Plt 90
Blast cells 60%
lymphos n neutros also increased
Diagnosis
A.Acute leukemia
B.Bone marrow infiltration by CA lung
C.CML
D.CLL

10.Psoriasis rash on whole legs since 6 mons.nextappropiate Treatment
A.Uv therapy
B.Calcipotriol
C.Predsinolone

11.Postop cataract 5th day got up with pain n blurring of vision red eye pic
laterlyicudntapreciate exactly ant eye had pus or not :(

A.Hypopyon
B.Glaucoma
C.Scleritis

13.Pt admitted to hospdue to gentourianry infection not sure though! History of
rash topenicillin. staph aureus on culture which is the antibiotic of choice for her
A.Ciprofloxacin
B.Cefazoline
C.Cefalaxine
D.vancomycin
E.ceftriaxone

14.You are a doctor in a town, where 6 people out of 100 are non-smoker. What are
the
chances of stroke. The chances of stroke in smoker is 50%more than non-smoker.
Now

the pharmaceutical company is introducing a medicine which reduces the


chances of
stroke up to 1/3rd in smoker population. What is the percentage of the stroke
population

will get stroke

A. 3%
B. 6%
C. 9%
D. 12%
E. 20%


15.A study wish to make a relation btw the fatigue in track car driver in high ways
& the

incidence of MVA happen. wt is the most suitable method to carry out study?

A. Cohort
B. Case control
C. RCT
D. Cross sectional
E. Case study


16.young man presented for routine check up.he is on sertraline 100mg. He
has no complaints and all his examination was fine except for blood pressure
of 160/100. What is your appropriate next step

A-take a 2nd measure of his blood pressure after 1 week

B-prescribe antihypertensive drug (may be ACEI)

C-24 hours ambulatory blood pressure monitoring

D-reduce the dose of sertaaline

E- stop sertraline
man was prescribed Selegeline for parkinsonism. He was on Sertraline 100mg per
day for depression and was well controlled on it. He developed abdominal pain and
diarrhea. His BP was high. What is the next appropriate step?
a. Stop Selegeline
b. Stop sertraline
c. Decrease the dose of sertraline
d. Decrease the dose of Selegeline
e. Stop both sertraline and selegeline
17.7 days old baby presented with jaundice since 4 days of life, his birth was
at term, not complicated and he is breast feeding, serum bilirubin level was
280 (normal <200) with 06 direct bilirubin.What is the most appropriate next
step

A-thyroid function test

B-abdominal ultrasound for liver and biliary tree

C-coombs test

D-stop breast-feeding and follow up

E-phototherapy

18.patient with type 2 diabetes on metformin 500 mg came to routine health
checkup, lab investigation was done showed as follows ,

Test result

Random blood sugar 5.6 (4.5-6.5)

Cholesterol (within normal)

HbA1c 6.9% ( normal<6.5%)

Urine dipstick trace of protein

What is your most appropriate next step in management?

A-Commence insulin

B-increase metformin

C-commence Ramipril

D-add simvastatin

E-continue same treatment jm 870 ( dx diabetic nephropathy )




19.a breast feeding woman presented complained that her baby remain
hunger and her breasts after feeding remain full, what is the relevant behind
this

A-putting the baby on the breast insufficient frequently

B-putting the baby few time on breast each feeding

C-poor positioning


20.- A woman presented with MMSE 27/30 with history of hypertension and
parkinsonism on many medications complains of seeing green cat in his
window. Asking for diagnosis.

A-Due to medication

B-.Parkinsonism

C-Lewy body dementia

D-Creutzjacob disease

21.Lady primigravida HCV +ve comes to discuss risk to fetus during delivery.
What is the most important thing to tell her about vertical transmission?

A-Vaginal delivery is contraindicated

B- She should not breastfeed her baby

C-delivery should be by caesarean section

D- Fetus will be checked for HCV at birth

E- Fetal scalp sampling should be avoided




22.History of appendectomy & cholecystectomy presented with 3 week
history of abdomen pain, distension, Bowel Sound exaggerated, CT abdomen
image given. Asking the cause

A- Sigmoid volvulous
B- Adhesive IO

C- CA Sigmoid

D- Pseudo obstruction

23.case of a patient present with foul smelling cough, high fever with rigor, x-
ray done found cavity with air fluid level,after giving antibiotic what is the
most app step

A- Trans pleural drainage

B- Water seal drainage

C- Needle aspiration

D-Blood culture NO option of CT. dx: empeyma



24.Grandmother come to you to get advice related to vaccine as her daughter


is pregnant. She has heard that there is some vaccination, which she can use
to protect newborn. Which is the vaccine?

A-diphtheria pertussis tetanus

B-influenza.

C-pneumococcal.

D-Meningococcal

25.Lady comes on 10th post-partum day with pain in perineal tear. History of
perineal laceration during delivery which was sutured. Now on examination
there is a perineal wound 2 cm long 1 cm wide and 1 mm deep. Clean with no
discharge, next?

A-apply local antibiotics

B-put on oral antibiotics


C-simply keep the wound clean

D- suture the wound under LA

E- suture the wound after cutting the edges



26.- 40 year old woman of retro orbital pain and reduced visual acuity for 3
days. What is the most appropriate investigation?

A-Temporal artery biopsy

B-CT scan

C-Visual evoked potential

D-fundoscopy

No MRI in options ( multiple sclerosis -optic neuritis )



27.An alcoholic, very agitated, rushed to the emergency department with
many complaints, claiming if he is not attended to immediately he will jump
in front of a car. What will you check first?
a) Alcohol level
b) magnesium level
c) serum electrolytes (Na+ and K+)

28.A patient has taken lots of tablets of venlafaxine. Was on depression


treatment. One year back she was given psychotherapy as she had problem
at work (shout at her coworkers). She can’t relax at home after work and
insomnia. After stabilization of the patient, what should be added?

A. Lorazepam

B. Risperidone

C. Mirtazapine

D.Haloperidol
E. olanzapine

29.Young primigraivda delivered a healthy 3250gm baby after an uneventful
labour lasting only four hours. During labour, no analgesics or epidural was
required. There was slight meconium staining. The 49.fetus developed severe
central cyanosis, limp n pulse rate of 40 breaths per minute. What is the most
important next step to resuscitate the baby?

a. bag and mask ventilation

b. Intubation and ventilation

c. Intravenous sodium bicarbonate

d. Intravenous glucose

30.That lady presented with meningitis.her Child had rash and fever few days
back resolved.ladys CSF protein 0.45, glucose 3.5, cells- Monocytes. Cause? A.
HSV B. Enterovirus C. Meningococcus

31.4days old infant found in cot cyanosed but no murmur heard, O2 saturation decreased
in spite of giving O2, asking Dx?
a) Transposition of great vessels
b) TOF
c) VSD


32. Patient taking OCPs presented with weakness of right arm tone n reflexes normal
cannot move arm actively asking cause
A. TIA
B. conversion disorder
C. stroke
D. malingering

33. Gout picture. Treatment asked. Patient was already on Allopurinol and
intermittent colchicine and urate was below normal level

A)increase allopurinol
B) Naproxen

C)Prednisolone

34.picture of abdominal aortic aneurysm abd pain and mass palpable pulse
100 bp 100/70 next appropiate step
A. FAST USG
B. aortogram
C. repair n grafting

35.papillary thyroid CA left lobe 3cm most app management
A. total thyroidectomy
B. left lobe lobectomy
C. iodine therapy
D.chemotherapy

36..14 years old guy has several episode of blank stare last for 60-90
seconds, abnormal movement of hands, lip smacking. He seems puzzed
after attacks. sometimes it happens every 2-3 days, something he has no
symptoms for several weeks. Diagnosis?


A. Absence seizure
B. Juvenile myoclonic epilepsy
C. Temporal lobe epilepsy


37.WPW HR 180 what next
A. amiodarone
B. verapamil
C. cardioversion


38.man with back pain on xray thoracic vertebrae fracture.on bone scan
uptake at fracture site only
labs anemia.CA normal. No symptoms of BPH given.what investigation ll lead
u to diagnosis
A. PSA
B. bone marrow
C. MRI
D. DEXA

If bone scan uptake more –then suspect malignancy

If uptake only one localized area –then do CT and MRI



39.vertebral fracture pic given in lady working long hours Ca.normalVitD bit
low DEXA -2.5 at hip n spine both. Most appropriate therapy
A. vitD
B. Alendronate

40.Old MAN with urinary incontinence when coming back home from
shopping center asking cause
A.detrusor instability
B. BPH
C. UTI



41.Man brought to hospital after throwing brick at windows he said last
thing he remembers is leaving work from rural farm
Dissociative fugue
Factitious disorder
Malingering


42.24 years old aboriginal male with lack of interest in normal life and is
having visual hallucination watching his ancestors he is saying my mother
died last month but it is not the case and mother died year ago, also has
sucidal thought what is most likely diagnosis?
A. Personality Disorder
B. Depression with unresolved grief
C. Depression with psychosis
D. Schizophrenia
E. major Depression

43.calculate cardiovascular risk..chart given

44.xray of anterior dislocation asked finding
A.post dislocation
B. loss of sensation inner aspect arm
C. loss of sensation outer aspect forearm
D. loss of sensation shoulders

45.mitral valve prolapse pt going for colonoscopy prophylaxis asked ans :
erythromycin

46.CT of glioma I guess had butterfly appearance with same qs previous
history of clark 1 melanoma askin diagnosis
A.glioma
B.metastatic melanoma
C.brainabcess

47.Post cataract surgery 5th day woke up with pain n redness pic given
A. hypopyon
B. glaucoma
C.iritis
D.scleritis

48.past history of dvtpt on UFH perioopertively n switch to lmw heparin
after 5 days undergone rt hip surgery develop dvt after 10 days that what to
do
In investigation only platelets are decreased
A- ffps
B- vit k
C- cease heparin and switch to other anticoagulant
D- platelets infusion

49.post op hemicolectomy pt labs Na 110 K 3.5 CL low as well.osmolality


not given casue
A. SIADH
B. Overinfusion

50. Young girl with menorrhagia regular cycle passing clots on 1st 2 days
with heavy bleeding what initial investigation
A. FBC
B. pelvic USG
C. hysteroscopy

many qs of copd n heart failure



twin stop syntoqs 2min 4min

twisted recalls same topics. read topics well. ALL THE BEST !!!

1. Young boy developed pain in the right knee .. mild progressive marked swelling .. no other
systemic symptoms as I recall Blood results given .. anemia .. low platelets .. normal WBCs What
is your diagnosis
Juvenile rheumatoid arthritis
Acute leukemia
Apalstic anemia
SLE

2. Which of the following is notconsidered an extra-articularmanifestation of Rheumatoid
Arthritis?
a) Osteoperosis
b) Peripheral neuropathy
c) Cutaneous nodules
d) Pericardial effusions
e) Hepatomegaly
3. couple came to you they r in relationship with each other from last 4 yrs now they are
planning for pregnancy what will you check in female partner
a) Rubella b) CMV c) Parvovirus d) Toxopalasmosis e) Varicalla

4. Commercial driver had seizure and was adequately treated .. You told him that he souldn't
drive ..however he came back after 3 weeks telling you that his work needs made him drive
yesterday and was no problems at all in that ..he thinks he is totally fit now and can drive
Management
1. Talk to him and tell him how dangerous is that to drive now
2. Call his boss and tell him about the disease
3. Tell him that you have to report what he did for the official driving agency for safety
4. Ask him to give you his license
5. Pregnant lady with full term baby at delivery, what CTG expected to show if
with cord prolapse presentation?
A. early decelerations
B. late decelerations
C. variable deceleration

6. 20 year old came for HPV examination ( pap smear). She is sexually active.
On examination, yellowish exudates obstructing the endocervical space. What
best next?

a) do pap smear as requested

b) tell her it is not beneficial for her age

c) treat infection and perform pap smear after 3 months


7. Which one of the following can be used for data analysis in Cross sectional?

A. Odds ratio
B. Relative risk
C. Chi square test
D. Attributable risk and Chi square test
E. odds ratio and attributable risk
8. A 16-year old female comes in for treatment of severe cystic acne with
Accutane(isotretinoin). She lives alone and is self-supporting with a job as a
waitress. She hasbeen out of her parents'house for a year and pays all her own
bills. you have just fin_ishecl informi'g her of the potentially sevcre
teratogenicitv of isotretinoin. Her acneis severe and she would still like the
isotrctinoin.What should you tell her?

a.This medication cant be taken by women of reproductive age

b. Use benzol4 peroxidc topically instead

c. l will give you the isotretir.roin if your parents accompany you

d. I will give you the isotretinoin as requested.

e. I will treat you with isotretinoin ifyou have a consent signed by your parents

9. Which of the follorving most closely represents the role of risk managernent
in the hospital

a to ensure proper ethical management ofpatients


b. To ensure proper clinical care ofpatients

c. To act as a patient advocated..

D To minimize the legal risk to the hospital from litigation

10. A young man has a syncopal attack while weight lifting. He has had
similar episodes twice before. His father died of cardiac disease. What is
the most appropriate management? a) Holter monitor b) Echocardiogram
c) Stess test d) BP in supine & lying down e) CT scan

11. All of the following are true about duodenal ulcer, except? a) Hunger
pain b) Loss of appetite c) Weight gain d) Relapses & remissions e)
Relief by antacids ( jm plus surg oxford )

12. A 68 years old man with COPD was brought to your surgery from
nursing home by ambulance. On his way to hospital he received O2, 10
L/m by mask. He is still unarousable and his ABG most likely-- a) Ph 7.29
PaCO2 65 PaO2 85 b) Ph 7.15 PaCO2 50 PaO2 68 c) Ph 7.25 PaCO2 25
PaO2 100

13. Opening snap indicates: a) Mitral valve mobility b) Atrial fibrillation


causes disappearance of the opening snap c) Replaces S3 d) Best heard
at 2nd right intercostals space

14. A 37-year-old female gives birth to her 2nd child. At the 10th day the
child becomes jaundiced. His mother tells the doctor that her first child
had jaundice after birth and then developed bilateral cataract after the
jaundice. What is the diagnosis? a) Congenital rubella b) Neonatal
hepatitis c) Diabetes Mellitus d) Syphilis e) Galactosaemia ( jm jaundice )

15. The commonest cause of hip pain in a 3-yearold child? a) Transient


synovitis b) Perthes disease c) Slipped upper femoral epiphysis d)
Arthritis e) Tuberculosis ( jm )

16. Regarding unilateral undescended testis, which of the following is


most commonly associated? a) Malignancy b) Varicocele c) Ingiunal
hernia(Indirect) d) Hydrocele e) Torsion of the testis
17. female had mastalgia for the last 1 year. Conservative treatment has
not worked. How would you manage this patient? a) Bromocriptine b)
Danazole c) Clomiphene d) OCP e) NSAID’s

18. skin condition COMMONLY responsible for persistent dandruff > a)


Irritant dermatitis > b) Seborrhoeic dermatitis > c) Tinea capitis > d)
Lichen planus > e) Pityriasis versicolor

19. Extensive psoriatic lesions on hand and legs. Treatment? > A- UV-B >
B- diathranol > C- steroids > D- erythromycin

20. Australian government appointed you to go through a community


based service and help a local doctor there. Hepatits B data given in a
community of total population of 200. 2009 2010 Antibody +ve cases 10-
15 Antigen +ve cases 40 - 55 Need to calculate the prevalence per 1000
population in the year 2010

A- 350

B- 250

C- 70 D- 50 E- 125

21. 39 weeks pregnant lady came with labour pain. Synto was given appropriately. Ctg was done
which shows heart rate of 140 which dropped to 70 and came back to 140 in 2min. asking next
immediate appropriate treatment.

A. Stop syntocinon

B. Fetal scalp sampling

C. C section

D. Titrate to increase dose of syntocinon

22 weeks pregnant lady came with labour pain..she was put in left lateral position having oxygen
mask in place.iv fluids with syntocinon is running. Ctg was done which shows heart rate of 140 which
dropped to 70 and came back to 140 in 4 min asking next immediate appropriate treatment

A. Fetal scalp sampling


B. Continuuous ctg monitoring

C. Stop syntocinon

D. C section

23. 38 year old g1p0 38 weeks coming to ER after Spontaneous rupture of membrane. After 3 hrs, no
active contractions and synto was commenced.. after 4 hrs assessement, uterine contractions good
but on continuous CTG montioring there is 60 bpm with slow recovery to normal. On exam:LOA
position, caput 1+ mould 1+? , cx dilated to 9 cm.What is your next appropriate mx?

Add 1 L of NS

Stop synto

Forceps delivery

Vaccum delivery

Urgent CS

24. Man with nocturia n urinary frequency. mass above the pubic symphysis DRE showed enlarged
prostate with palpable median sulcus palpable.what is the most appropriate next step?

A-trans rectal ultrasound

B-PSA

C-s. Creatinine

D-CT

E-urine cytology

25. Sarcoidosis with skin lesions- skin biopsy

26. Sarcoidosis xray with nodules - dyspnea, 12months loin pain, 24hours hematuria…what inv have
to do NEXT?

a) CT chest b)Serum ACE c)USG of abdomen d) urine r/m/e e)cystoscopy

27. A child with fever and hx of sore throat 10 days back . He developed pain in wrist later developed
arthralgia n swelling in ankle joint. skin rash present . what's the most initial step of management ?

a. ESR

b. Full blood examination

c. throat swab

d. USg
28. Viral pneumonia trip to Asia

29. old man stopped taking drugs ,now he has edema up to knee , HR 106 HTN with clear lungs

what next ?

a.fursmide

b.spironolactone

c.digoxin

d.BB

E.recommence ACE ,BB,other drugs but not diuretics

30. 14 y. old girl.. Hx of wt loss...BMI 15..when asked about any interest ,she said why should I? what
sholud this entire world mean?

what is the most relevent to her Dx.?

A. fatigue

B. sleep dist.

C.moving around with friends

D. feeling of inferiority

E.idea of guilt

31. A man aged 64 yr. his psa is 3.8ng now.2 years before 1.5.he did 12 biopsies this time and one
showed a focus of adenocarcinoma and gleason score 4 . What is the management for this man?

A) radical prostatectomy – psa doubling time within 2 tears and young age

B)external beam radiotherapy

C) orchidectomy

D) continued surveillance

32. child with asthma. On exam urinary ketones found and glucose high. Next to follow up

a)Culture and sensitivity

b)Glucose level

c)hba1c

33. A scenario of COPD. Patient was treated with augmentin ( amoxicillin + clavulinic acid ) and some
other antibiotcis. Was on inhaled salbu and ipneb 8 hourly and oral prednisolone 30mg/day. Was not
getting better. Had yellow sputum. Became dyspneic. ABGs done. pH 7.35. pCO2 50. pO2 80. HCO3
35. What to do as the most appropriate next step?

a) Bronchodilator 4 hourly

b) Change augmentin to ceftriaxone

c) Change oral predniosolone to IV hydrocortisone

d) Admit in ICU for intubation

e) CPAP

34. Old woman 75years is admitted to the hospital following a community acquired pneumonia. She
received antibiotics and is well on the 5th after discharge she suddenly develops rigors, chills and
high grade fever. Had crepitations on chest. What is the most likely diagnosis?

A. Hosp acquired pneumonia

B. Iv cannula related bacteraemia

C. Empyema

D. Pulmonary embolism

35. restless leg syndrome- iron studies

36. Hutchinson's freckle- local excision

37. An aboriginal female has mitral stenosis and dyspnea sincedaily 6 weeks. On examination she
had lung basal crackle,and a temperature of 37.9. (Xray looked like pulmonary hypertension – bat
wing appearance, but I could also see a very light haziness in the right apical lung

Whats is the diagnosis.

A. Pulmonary Hypertension.

B. Pulmonary Fibrosis

C. LVF

d. TB

38. 1.Arab woman who doesn’t speak English with 16. weeks pregnancy, was referred by the mid-
wife for which she suspects her mental condition. The woman seems irritable, worried(or anxious)
about the people around and she’s uncomfortable when her husband is not around. She is also
irritated with her 2 children. What condition in her history will be present to lead you to diagnose
this patient? (looks like prodromal symptoms)

1. Panic attacks

2. Paranoid personality disorder


3. Family member with schizophrenia

4. History of trauma

39. A case of lateral epicondylitis. Treatment?

a.Lateral epicondylectomy

b.Immobilization of the fingers

c.Analgesic

d.Sling below elbow

e.Brace below elbow

40. 30.Pt with RA she takes ibuprofen and methotrexate to control her disease ,they mention the
time it was years for both of drugs , pt complains of some symptoms , and her labs are given ALT
,AST GGT all were high , the Q is asking about the cause of this ?

A-Methotrexate induced hepatitis

B-Ibuprofen induced hepatitis

C-Autoimmune hepatitis

DViral hepatitis

41. An elderly woman on polypharmacy GFR was 6 weeks back came with dyspepsia, was put on
Olazpine and anatacids and is now presenting with confusion, loss of appetitie and nausea.

Sodium 129, k 5.3, calcium 3, phosphorus high , creatinine high, urea very high and bicarbonate 20,
what is the reason for the confusion?

a) Uremia

b) hyponatremia

c) hyperk

d) hyper ca

e) metabolic acidosis

42. Osteoporosis with gerd - zolendroic acid

43 osteoporosis with menopausal symptoms_ hrt

44. Menopausal symptoms - dvt with hysterectomy history in the past- transdermal estrogen

45. a term baby with uneventful delivery except for light meconium staining, after 1 min has low
apgar, severe bradycardia- hr 40, next?
a.oxygen with bag and mask

b.intubate

C. Nasopharyngeal suction

46. A case of ankylosing spondylitis, patient not taking any treatment other than physiotherapy pain
not subsiding. Next treatment

1.methotrexate

2.salfasaline

3.indomethacin

4.infliximab

5. Naproxen

47. Female 42 years of age presents with mild hypertension for which she was started on ramipril.
After 2 months she presents with B.P 170/100 , nocturia and Blood chemistry showing increase in
urea and creatinine well above baseline. Which of the following will point towards her
disease?There was Presence of 3 cysts in left kidney and 2 cysts in the right
- pkd

48. abroriginal child comes with purulent ear discharge for 6 weeks what will u do after clearing the
discharge

a) amoxcilin…if acute

b)ear toilet…if chronic >6 weeks

c)prednisolone

d)reassure

e. Cipro ear drops

49. Scenario of acute closure angle glaucoma in old age, haloes around light, red sclera, IOP 25
mmHg, asked long term management?

A. Trabeculectomy

B. Peripheral iridotomy!!!!!

C. Acetazolamide

D. Pilocarpine

50. Acute Gout – joint pain with warmth with ckd - Prednisone

29. A scenario of an old man with history of atrial fibrillation and now comes with severe abdominal
pain. What other than CT angiography will you do?
a) CT abdomen

b) Serum lipase

c) Serum lactate

51. 75 year old man comes to you with progressive constipation for last 3 weeks with absence of
flatus. Abdominal distension is becoming worse. Mild tenderness is present. He had history of
cholecystectomy 25 years ago. Dilated bowel loops along ascending transverse and descending
colon. Rectal examination is empty. What is the diagnosis?

Adhesion obstruction

Sigmoid volvulus

Cancer of Sigmoid colon

Ceacal volvulus

19.4.2018 (Singapore 9:00am) Recall

That hemolytic jaundice and melanoma eye scenarios as usual.

1. Female BMI 20 with repeated treadmill exercise and looks herself every hour in mirror and changes
clothes three times per day, dx:
a. Body dysmorphic disorder
b. Anorexia Nervosa
c. Obsessive compulsive disorder
d. bulimia
e. She is fond of exercising ( anorexia nervosa could be normal BMI )

2. 22. 12 year old girl at foster care. she's having difficult time in school in reading and writing. she
has a collection of her own soft toys she is very proud of it. and plays with them with her imaginary
friend . her foster parents/carers are irritated because she's picky at food. what immediate danger to
her in future?

a. OCD
b. sexual abuse
c. drug abuse
d. Anorexia nervosa
e. schizophreniform psychosis

3. 25-year-old man came to ER with history of back pain L4-L5 level. He denies any history of back
injury. Previously, he was drug abuser and Hepatitis C positive. Physical examination is normal. He
has an erythema at the back which is painful. Which of the following is the most appropriate?
A. HIV serology
B. CT spine
C. MRI spine
4. 216. A man comes with clumsiness of hands and tripping over. Upper limb weakness and
fasciculation. Lower limb muscles also has weakness, increased tendon reflexes. Ankle reflex is not
increased in one side. No wasting. No sensory loss is given. What is the initial investigation?
A. EMG
B. MRI Spine
C. CT scan
D. CSF examination
E. Cervical spine X-ray

5. . A mother come with her 8 months old child complaint of continuous bleeding after fall from
coffee table. On examination there is bleeding frenulum and some old bruise in forehead and leg.
There was no petechiae, lymphadenopathy, hepatomegaly. What is the cause?
A. ITP
B. Non accidental injury
C. VWD
D. Hemophilia A
E. Hemophilia B

6. Young aboriginal male presents to you with insomnia, fear of darkness and seeing “mamu”. He has
been having these symptoms after the death of his mother. Which of the following should be next step
in his treatment?
A-give him benzodiazepine
B-consult to aboriginal health worker
C-urine drug screen
D-antipsychotic
E-drug and alcohol abuse counseling

7. 4 year old child with cough and nasal discharge. The child is febrile and unwell. There is noisy
cough, intercostal recession. RR-increased, PR-increased. Lungs clear. ENT examination is normal.
What is the most likely diagnosis?
A. Asthma
B. Acute tracheitis
C. Acute laryngotracheobronchitis
D. Acute epiglottis
E. Acute bacterial Pneumonia (exact option)

8. A boy come with high fever (38.5) and swollen knee, pain at the medial tubercle of the knee, what
treatment will you give?
a. Penicillin G
b. Penicillin G + gentamycin
c. flucloxacillin
d. erythromycin
e. Ticarcillin- ??? (I forgot the co-drug).

9. Woman feels she has worms in her stomach because of a recent volcano eruption in indonesia.
Previously on trifluorperazine but stopped due to hand stiffness. What to give now?
A. Quetiapine
B. Olanzapine
C. Recommence trifluorperazine
D. Stop trifluoperazine

10. A schizophrenic patient admitted involuntary. He wanted to sue hospital against his admission. He
briefs that God wants him to swim across Pacific Ocean. And he says other people in his religion have
same belief. What's most important justifying his continuing involuntary admission.
a. His poor insight
b. His belief placed him at risk
c. Fixity of his belief
d. Presence of positive psychiatric behavior
e.

11. 4 min synto scenario

12. 39 weeks pregnant lady came with labour pain. Synto was given appropriately. CTG was done
which shows heart rate of 140 which dropped to 70 and came back to 140 in 2min. asking next
appropriate treatment. (Que does not include any basic measure given for her like Left lateral position,
Oxygen, Fluid)
A. Stop syntocinon
B. Fetal scalp sampling
C. C section
D. Give IV dextrose, normal saline
E. Give O2 to mother via face mask

13. On interview. Asking about the appetite. Patient told “Well, that is thought about food. Food
which does not comes from moon, moon is not made of cheese. But I came her by bus”. Which of the
following will have in this patient?
A. Depression
B. Dissociation
C. Disorganized behavior (exact option)
D. Delusion
E. Depersonalization

14. A 30 years old woman with increasing dyspnea, dry cough over few months with painful lumpy
skin lesions on legs and shins. On examination, there are red, painful, lumpy lesions in leg and
bilateral ankle swelling. ACE level increased. Which of the following is the best next step to get
diagnosis? (No given X-ray)
A. CT scan chest
B. Skin biopsy
C. Blood culture
D. Aspiration
E. ANA

15. decrease breast milk production recall Asking common cause of decreased milk production in
breastfeeding
a. Less frequent feeding
b. Decrease intake of fluids
c. Spend less time while baby is sucking
d. The baby has a problem in development.

16. A lady presented with her baby 5 times in 2 weeks at 10 weeks following delivery. she also
presented at 8 weeks all normal established breastfeeding , baby growing well on 90th percentile.
what relevant info you will ask in her history?
A. her past bad obstetric history
B. her premorbid personality
C. her mood (exact option)
D. history of psychosis
E. Her thoughts about care of baby

17. 64 years man having check-up for prostate CA, last 2 years PSA was – 1.5ng/ml, now PSA is
3.8ng/ml, do the 12 slides of biopsy & only one foci show adenocarcinoma, gleason stage is 4, how
will u mange this patient?
a. TURP
b. radical prostatectomy
c. continued surveillance
d. EBRT
e androgen

18. Mother living in mining town comes with her daughter for normal blood test. Her daughter’s test
show lead level 0.72 (normal 0.32 given) what will be your next appropriate step?
A. Assess IQ testing of child
B. Educate mother about environmental risks
C. Refer for chelation therapy
D. Move the family to another town
E. Urine test to confirm dx

19. A mother comes with hemochromatosis in brother. she has 2 kids, aged 2 and 5 years respectively.
Wants advice regarding screening?
A. screen only mother
B. mother and kids
C. ask her to come with husband
D. Screen only children

20. Aspirin 100

21. Pheumaceutical company promotion to you. Showed documents of well-peered certificate. Which
study would you like to do to detect the safety (something like that?)
A. case-control
B. systemic review
C. cohort
D. cross-sectional
E. case study

22. Famous intern question where the intern finds the wrong dosage of the warfarin, approached the
RMO, but RMO refuses to accept the mistake saying that the prescription is correct - What to do –
a. inform the Board
b. ask the pharmacist to cross check
c. refer the case every day before the shift
d. inform the director of clinical training

23. 58 year old man presented with severe lower back pain and tenderness in the lower lumbar, he
has weight loss of 10 kg and on examination, there was tenderness over L4-L5 vertebra, On DRE the
prostate was enlarged and irregular, what is the most appropriate test that lead you to the diagnosis?
A-PSA
B- whole body bone scan
C-trans rectal ultrasound
D-PET SCAN
E-spinal CT

24. + 25. Post-op oliguria- with catheter and without catheter.

Post abdominal surgery a patient with catheter inserted has urine output of 100 ml in 24 hrs. He
started on IV fluids at 80 ml/hr. Fever 38.5. What is the next appropriate investigation?
A. Serum Creatinine and electrolytes
B. Pulse oximetry
C. Bladder scan
D. CT scan
E. Blood culture

26. Pt on Digoxin, Bisoprolol, thiazide (one more med I can't remember at the moment) Longterm pt
of HTN and CHF & AF. stopped taking meds for 3 weeks while on vacations. Now presents with
edema upto his knees & a raised JVP of 4 cm but with clear chest sounds. BP 145/90, pulse irregular.
What would u do as initial management of this pt?
1- commence furosemide
2- commence spironolactone
3- commence metoprolol
4- commence digoxin, bisoprolol

27. 25 year indigenous lady presented with sob and cough since 3 weeks. o/e temp 37.8, PR, RR, BP,
saturation all are within normal limits, mitral stenosis with bilateral basal crepts. X-ray given with
features like bilateral symmetrical upper lobe infiltrates (X-ray seems like apical but infiltrates are not
too high at the apex). Diagnosis?
A. Left ventricular failure
B. pulmonary hypertension
C. rheumatic fever
D. Peumocystis pneumonia

Saw “Bat wing appearance” so clearly.

28. Pt with warfarin, HTN, smoker, DM started amiodarone 1 week, came with now right thigh
swelling about 4 cm > other side, fever 37.8 diagnosis?
a) DVT
b) Cellulitis
c) drug interaction
d) embolus

29. A child 6 years old with history of asthma presented with upper respiratory tract infection, urine
examination showed very high serum glucose and ketone body of 2+.
What is the most appropriate test to follow up this child?
A. HbA1C
B. Serum creatinine and electrolyte
C. FBS
D. OGTT
E. ABGA ( RCH and RACGP )

30. Poorly controlled Diabetic with mild renal impairment asking about risk for baby due to diabetes
A. IUGR
B. Macrosomia
C. Renal agenesis
D. Intrauterine fetal demise
E. low Birth weight

31.

32. Acute painful red eye, fixed pupil, IOP 20 mmHg.


a. Timolol
b. Steroid
c. Chloramphenicol
d. Atropine
e. Acetazolamide
33. A patient with spiking fever, Shortness of breath and was inserted prosthetic heart valves 3
months ago. And diagnosis as due to Staph aureus, and given Flucloxacillin for treatment. The fever
subsides after 2 weeks of treatment, but SOB still persists, diastolic murmur at the left sternal edge.
What will you do for investigation?
A. Transthoracic Echo
B. Chest xray
C. Troponin
D. Blood culture for every week
E. CT scan

34. A child presents with increasing fever and cough of 2 days duration. X-ray was given which
showed right sided pleural effusion. Which investigation aids in the definitive diagnosis of the
causative organism?
a. blood culture
b. chest x-ray
c. pleural aspirate
d. pneumococcal PCR
e. Sputum culture

35. Picture of Dupuytrens contracture, Farmer drinking 4 standard drinks of alcohol everyday. Had a
cut injury to that finger few months ago. What is the cause for this.
A. Alcohol
B. Genetic
C. Use of farming tool
D. Cut injury

36. Alcoholic patient history of surgery for perforated diverticulum disease , post op 3 day , get
agitation, confused and oxygen 88%(exact) chest examination normal. Most appropriate next invx
A. chest x ray
B. CTPA
C. blood glucose
D. Blood alcohol level
E. Urine creatine and electrolyte

37. Old patient k/c of COPD and a smoker for more than 20 years. Came with complain of
progressive dyspnea. (SOB, orthop noea, stridor, plethora/
cyanosis, oedema of face and arm, cough, headache, engorged neck veins)
His face was plethoric, edematous and neck veins was engorged till the jaw line. What is diagnosis?
A) Lung CA
B) SVC obstruction
C) Bronchiectasis

38. CT (head) given- Cerebral infarct or Cerebral tumour?

The old patient was presented to you with confusion and decline in daily function. Forgetfulness is not
mentioned.

39. Young woman 26 years of age comes to you complaining of heavy menstrual bleeding with pain.
She describes that clots of blood are lost in the first few days of menses with severe back pain. You
ask her for some investigations that need to be performed. She gets irriated and says she just wants a
treatment. Which of the following is the apporpraite next treatment ?
a. Tranexemic acid from day 1 of menses
b. Mefanimic acid from day 1 of menses
c. Mirena
d. Oral contraceptive pills
e. oral medroxyprogesterone acetate

40. 40 years old woman with menorrhagia and hysteroscopy. She had a dilatation and curettage three
months ago. Which of the following is the MOST appropriate management plan?
a. nortestosterone
b. levonorgestrel IUD
c. continuous medroxyprogesterone
d. Tranexamic acid during period
e. Mefanamic acid during period

41. History of appendicetomy & cholecystectomy presented with 3 weeks history of abdominal
distension on ascending colon, transverse colon, descending colon, rectum is empty. There is mild
tenderness of the abdomen and loud borborygmi. What is the diagnosis? (CT was given) Ca sigmoid
with CT picture à not apple core deformity
A. Sigmoid volvulous
B- Adhesive IO
C- CA Sigmoid
D- Fecal impaction
E- Caecal volvulus

42. 75 yrs old man with 3 weeks h/o of abdominal pain and distension, had h/o of appendicectomy
and cholecystectomy, X ray (not given) shows dilated ascending, transverse and descending colon, on
examination, rectum is empty, what Dx?
a. Sigmoid volvulus
b. Ca sigmoid
c. Adhesive IO
d. Caecal volvulus

43. Farmer came with left swelling in a groin. U did fnac and it shows sqaumous cells. Where is the
lesion
A.Left leg
B.Anus
C.Rectum.
D.testis.
E.penis

44. Man with childhood immunization history as OPV and other vaccines
Now going to india for holiday trip. Which vaccines you’d give?
- polio
- MMR
- dTPa
- polio, MMR and dTPa

45. child with URTI, protein+, rbc+ on urinalysis, came back 2wk later, urine, rbc+ of non glomerular
origin, no cast. Which investigation?
A. Urine culture
B. ASO titer
C. USG
D. DMSA

E. Renal Biopsy

46. Pt with RA she takes ibuprofen and methotrexate to control her disease for 5 years. patient
complains of some symptoms, and her labs are given
protein - 9 (6-8)
albumin - 4 (3.5-5.5)
ALT - >200 (7-56)
AST - >200 ( 10-40)
GGT - >200 (0-30)
What is the cause of her symptoms?
A- Methotrexate induced hepatitis
B-ibuprofen induced hepatitis
C-Autoimmune hepatitis
D-Viral hepatitis

47. You want to know the Hep C in newborn baby born to Hep C positive mother. How to
investigate?
A. Antibody of baby at birth
B. Antibody of baby at 18 months
C. Antibody of baby at 12 months
D. HCV PCR of mother

48. A 25 years old primigravida presents for results of antenatal screening tests. She is a drug injector
in the past. But has not get any drugs in the last 5 years. Her results show Hep C Ab +ve, Hep C RNA
–ve, HIV –ve, Hep B Ab +ve. Which of the following will reduce the risk of transmission of Hep C to
her baby?
A. LSCS at term
B. Formula feeding of newborn
C. Avoid pH monitoring of fetal scalp electrodes
D. Anti-retro viral therapy

49. . 4 years old child comes with high fever. He is very toxic. Soft inspiratory stridor. No cough.
Chest clear. What is the causative organism??
A) RSV
B) H influenzae
C) Influenza virus
D) strep pneumoniae

50. alcoholic patient present with ascites, gynecomastia, spider nevi, parotid gland enlargement and
flapping tremors and confusion (not mention about abdominal pain). Vitals given. What investigation
to do?
a. Ammonia level
b. Full blood examination
c. Liver function test
d. Abdominal scan
e. Abdominal paracentesis
(No option for IV antibiotics)

51. 18 month baby. Diarrhea for 6 weeks. Stool tests normal. First copious watery diarrhea now
undigested food particles in stool. What to do?
a. Gluten free diet
b. lactose free diet
c. reduce red meat in diet
d. continue same diet
(Explanation or reassurance- not optional)

52. 18 year old male with bloody diarrhea, you made sigmoidoscopy which shows 10cm friable
mucosa. What is the initial management?
A. Sulfasalazine
B. Rectal Corticosteroid
C. Injection Corticosteroid
D. Lopramide
E. Methotrexate

53. A man presents with black lesion in his cheek which lasts for 6 years, now size is increasing.
What is the next step of management? (Given picture)
A. Local excision
B. Excision with a 2 cm margin.
C. Fluorouracil cream
D. Review in 12 months
E. Shave biopsy
If 2 mm margin present then that will be the ans

55. 54yrs old male patient has sudden retrosternal chest pain. Severe vomiting proceed by pain. He
has HTn and controlled with Thiazide. On examination, dullness lower left lung and reduce breath
sound on left lower zone of lungs. Which of the following Investigation to reach diagnose?
A) CT chest
B) Gastrograffin swallow
C) USG
D) CXR
E) Echo
57. 7 days old infant presented to you complained with poor feeding and bile stain vomiting. His birth
was at 38 weeks gestation and weight 2600g.He is breastfeeding with no immediate post natal
complications.The child has a history of passing meconium on day 4 of birth.Now, he had mild
jaundice and abdominal distension present.What is the appropriate diagnosis?
A. Meconium Ileus
B. Hirschsprung's Disease
C. Duodenal Atresia
D. Necrotizing Enterocolitis
E. Volvulus

58. 7 days old baby presented with jaundice since 4 days of life, his birth was at term, not complicated
and he is breast feeding, serum bilirubin level was 240 (normal <200) with 120 (SURE) direct
bilirubin, his liver is 1 cm palpable below costal margin. What is the most appropriate next step?
A-thyroid function test
B-abdominal ultrasound for liver and biliary tree
C-coombs test
D-stop breast-feeding and follow up
E-phototherapy

59. Woman around 30-35 years of age attends for Pap smear, Her previous smears are all normal. She
doesn’t have sexual activity in last two years. What’s the reason of testing chlamydia in her?
A. chlamydia is asymptomatic
B. chlamydia can cause infertility
C. Both partners could be asymptomatic
D. No need for screening

60. Couple come for infertility problem for the last 12 months. On testing examination and tests of the
female are unremarkable. Male has azospermia. And bilateral absence of vas deferens. Which of the
following is most important test before the starting the treatment of infertility?
A. No testing required as they cannot have a child
B. Testing of both male and female for cystic fibrosis
C. Refer for IVF
D. Serum FSH and LH for male
E. Serum Testosterone level

61. An old man has been taking many medications for some kind of cancer pain. His son found him
unconscious, bringing him to hospital. The patient had pinpoint pupils. He was given naloxone in
ambulance and came back into consciousness. He said the medications he has been taking are
methadone, oxycodone, nicotine patch, paracetamol and another drug (I don’t remember). Which of
the following drug causes this presentation?
A-methadone
B-oxycodone
C-nicotine patch
d. paracetamol
e.

62. . Patient with ACTIVE RA and chronic pyelonephritis, feels tired and lethargic, Lab is done and it
shows
Hb is round about 9
MCH is low normal
Serum iron decreased
Serum ferritin normal
TIBC decreased
What is your next appropriate management?
A. Packed RBCs
B. IV iron
C. Low dose prednisolone
D. Erythropoietin stimulating agent
E. Folic acid

63. A 6 month old child came with recurrent upper respiratory infection. The immunization history
reveals that the child has received his hepatitis B vaccination at his birth and no further immunization
history at 2 and 4 month of age due to recurrent upper respiratory infection. Now the child has fever
37.8 and clear nasal discharge. What is your management regarding immunization?
A. Give hepatitis B immunization and institute catch up schedule
B. Give hepatitis B immunization and recommence standard schedule
C. Investigate underlying cause of URTI
D. Give hepatitis B immunization after fever subside
E. Give hepatitis B immunization at now

64. A patient came with confusion. Was on multiple drugs. Known case of DM and HTN. Serum
sodium was less. Glucose was 8. It was mentioned that urine specific gravity was normal. Asking for
cause of confusion? (Lab values given) sodium- reduced, potassium- increased, Chloride- normal.
a) Hyponatremia
b) Hyperosmolar syndrome
c) Hyperkalemia

65. . woman present with loss of appetite,sleeping badly, she is afraid in the dark and has been crying
frequently(for which I take sleeping badly as unable to sleep coz of her symptoms) 2 weeks after
death of her husband due to prostate cancer. She thinks he died because of her infidelity during her
young age. she had similar episode after the death of her child. prompt treatment asked ?
a. Risperidone
b. venlafaxine
c. ECT
d. temazepam
e. Citalopram
upto this
66. Woman 51 years of age presents after menopause for the last 8 months. She complains of
irritability, moodiness, low libido and hot flushes. Which of the following would be most appropriate
to give to this woman?
a. Combined continuous estrogen and progesterone throughout the cycle
b. Continuous estrogen throughout the cycle
c. Continuous estrogen with progesterone first 12 days of the cycle
d. Continuous progesterone with testosterone

67.
8yr old child brought by his mother , he complained from episode of staring suddenly that occur along
with fidgeting of right hand and movement of right arm head twitching to right side, sometimes
chewing and lip smacking each episode last for 60-90 second then the child remain dizzy and
confused for 1-2 minutes after the episode . These occurs in 3-4 days and then the child back to his
normal activity and behaviour for several weeks . what is the most likely diagnosis
A- Temporal lobe epilepsy
B- Juvenile myoclonic epilepsy
C- Absence seizure
D-Myoclonic seizure
E. Tourette
68. A woman works at a part-time job (OFFICE CLEANER), and complaints of early morning
headache, frontal & bilateral, dull in character, varying in intensity, she takes paracetamol &
ibuprofen, which only cause relief for 2-3 hours. What is the cause?
A. Drug rebound headache
B. Migraine
C. Tension headache
D. Cerebral tumor
E. Pre-menstrual headache

69. 42 years woman brought to you by her friends who wasn’t go out much for 10 years and stay in
home. But she was enjoying doing gardening and crafting and staying at home. What in history will
help you for her management?
A. School refusal (exact words)
B. Night terror
C. Alcohol consume
D. Her orientation

70. Old man presented to you complaining of rest tremor and bradykinesia, also taking respiridone
and he has a known history of visual hallucination and forgetfulness. what is the most likely dx
A. parkinson disease.
B. lewy body dementia
C. alzheimer disease
D. acute delirium.
e. schizophrenia

71. Prodomal Parkinson

72. Which of the following development can be assessed in a 24 mth child who is quite well and
brought by mother?
a. knows 2 pronouns
b. knows name of all colors
c. speaks full sentence
d. know family name
e. know age

73. Teenage boy comes to you. His exam is near. He looks so anxious, sleeping less and saying that
meat is poisonous. What is happening to him?
A. Hypochondriasis
B. Delusion
C. Paranoid?

74. 6 years old girl with hereditary spherocytosis came with marked pallor and palpable spleen,
history of URTI last week lasting for 3-4 days .
HB – reduced (more than 8.5)
WBC – normal
Platelet – normal
Retic count – 0.1 % (not increased compared to given normal value)
a. adeno virus
b. cytomegalovirus
c. parvo virus
Anemia or hyperbilirubinemia may be of such magnitude as to require
exchange transfusion in the neonatal period. Anemia usually is mild to
moderate; however, sometimes it is very severe and sometimes it is not
present. Splenomegaly is the rule, and palpable spleens have been
detected in more than 75% of affected subjects. Severe hemolytic anemia
requires red cell transfusions.
In chronic congenital hemolytic anemia (ie, HS), long periods of
asymptomatic disease depend on a fragile equilibrium in which the
excessive destruction of cells is balanced by accelerated erythropoiesis.
Disruption of this equilibrium can lead to rapid and dramatic falls in blood
hemoglobin levels, producing an aplastic crisis. Most, if not all, aplastic
crises are caused by infection with type B19 human parvovirus (HPV).

75. Indigenous child with chronic suppurative otitis media presents with chronic ear discharge. Ear
swab done showed existence of pseudomonas aeruginosa organisms. After ear toileting what will u
do?
A. Oral Amoxicillin
B. Oral Augmentin
C. Ciprofloxacin ear drop
D. Paramycetin with steroid ear drops
E. Hearing aids

76. A young man brought to ED after fight in bar in which he hit someone. He is alcoholic, take
multiple drugs and aggressive. What in history will you to know if he has personality disorder?
a. Childhood sexual abuse
b. H/o cruelty to animals in adolescence
c. Drug and alcohol dependence
d. H/o of hitting partner one week back
e. H/o depression in mother

77. complaining of soreness on lateral epicondyle at the end of work, the patient attributes it due to
computer mouse clicking.
On examination , tenderness on lateral epicondyle .Asking treatment?
A-lateral epicondylectomy
B-finger immobilization splint
C-lateral epicondyle immobilization splint
D-bracing under lateral epicondyle
E. oral steroid

78. year old male with a history of fall on an outstretched hand with pain and swelling. X-Ray was
given, (showed scaphoid fracture. Very clear fracture line through the middle)
What is the best treatment option for this patient
a. Crepe bandage
b. Plaster cast
c. Analgesics
d. Compression screw
e. Plate fixation

79 4-day-old child with jaundice. No complication is seen (as far as I remember). Total Bilirubin is
380. Management?
A. Phototherapy
B. Exchange transfusion
C. Follow up 1 week later
D. Formula milk
80. 35 year old lady comes to you regarding screening for breast cancer screening. She says her
paternal aunt was diagnosed with breast and ovarian cancer at the age of 60 years which was
diagnosed as being associated with BRCA1.Which of the following is the most appropriate advice?
A-Genetic counselling assessment
B-refer for BRCA 1 screening
C-screen now
D-tell her to look at her breast
E-annual mammography
81. 14-year-old boy doesn’t want to take drugs. Police brought him to you for Anti-social behavior.
What is your treatment?
A. Give IM haloperidol
B. Change to Respo depot.
C. Explain the benefits and outcome of taking the drugs
Depend on full stem

82. MVA. CT scan (given)- pneumothorax. Breathing is limited due to pain. He is distress, oxygen
saturation is reduced. What is your next appropriate management?
A. Morphine
B. Needle thoracotomy
C. Chest tube drainage

83. Young man after a quarrel had a fracture of floor of eye what is the most consistent symptom with
that?
a-Conjunctival haemorrhage
b-loss of visual activity
c-anaesthesia around the cheek
d-Cant open the mouth completely
e- Epistaxis

84. 36-week gestation. BP 145/90. Headache, flashes in vision. What finding will lead you to an
emergency intervention?
A. five clonus in ankle jerk
B. BP recheck raising 150/80 after 30 minutes
C. 36 weeks of gestation
D. hand edema

(no option for urinary protein or seizure) ( going to eclampsia )

85. Mother brought his young child who presented with four lesions on his right face. She tried
antiseptic but the ulcers like lesions showed no improvement. What is your most appropriate
treatment?
A. mupirocin (bactroban)
B. topical hydrocortisone
C. topical acyclovir
D. topical idoxuridine
E. An antifungal with trade name? (Like fluconazole? Not sure.)
(Picture given. Lesions around lip. They do not have honey crust color but looks similar as impetigo.)

86. Old lady presented few other features were given with narrow introitus. Biopsy showed lichen
sclerosis management.
A. 0.5% steroid
B. oestrogen cream
C. Surgical removal

87. A patient is given Amoxicillin and gets allergy. Which drug to use?
A. Cephalothin
B. Cefotaxime
C. Chloramphenicol
D. Vancomycin
88. An old woman who is widow who seems well in past but since she became a widow and she
moves to the house and start to stay alone one of neighbor had noticed her strange behavior. She starts
to dig some ground in front of the house and when neighbor reached near her and looked at her, she
then aroused him and get aggressive and accused him and all neighbors as imposters. After that she
became calm down and agreed to be seen at the medical clinic. Which one of the following will help u
to get a diagnosis?
A) Thought forms
B) Mood
C) Orientation
D) Delusion
E) Memory ( paranoid personality disorder ,,capgrass )

89. 80 year old man presented with loin pain, urine examination showed hematuria +++. What is the
most appropriate Mx?
A. X-Ray
B. IVP
C. CT Abdomen
D. Retrograde pyelography
E. Urine culture

young man presented with loin pain , urine examination showed hematuria . what is the most
appropriate next step?
A-X-ray
B-abdominal ultrasound
C-CT abdomen
D-MRI
E-urine culture
young man presented with loin pain , urine examination showed hematuria . what is the most
appropriate next step?
A-X-ray
B- ivp
C-CT abdomen
D retrograde pyelography
E-urine culture
final pls

90. A 25 year old man from local correction facility and the police brought him as a request for
checking him up . He complained of with abdominal pain n tenderness for a day. He has many tattoos
on his body and poor dentation(SURE) (No other history provided) What is your dx?

a. Factitious disorder
b. Schizophreniform
c. Developmental instability
d. Eating disorder
e. Schizophrenia
(A CXR is given with some radio-opaque curvy lines which seem to be hairs, and a straight
radiopaque line likely to be needle was seen.)
91. mother borderline and father on antipsychotics, came to u for a sedating med for their baby as
mother says that baby cries a lot and and on examination baby is weak, and lethargic something.
1- Report child protection
2- Give med
3- treat mother
4- treat father
5- good parental education
If I think any abused done by parents or anyone: must notify ,whether it is really abused or not

92. A sudanese boy had sex with his girlfriend. after a few days he found urethral discharge from his
penis. He has other sexual partners. what specimen will help you make a diagnosis of the STI
a.1st catch urine PCR
b. Mid stream urine pcr
c. first stream urine microscopy and culture
d. Urethral swab gram stain and culture
e. mid-stream urine microscopy and culture

93. a 56 year old man with epigastric hernia which extends from umbilicus to xiphisternum. He
looked morbidly obese and had a waist circumference of 110 cm. Asking for most appropriate advice
for the patient?
A. Weight loss therapy
B. Abdominal binder
C. Mesh herniorrhaphy
D. Hernioplasty
E. Observation
What is the most appropriate management of this patient.
A. Weight loss therapy
B. Abdominal binder
C. herniorraphy with mesh repair
D Hernioplasty
E Observation

94. Old man present with large abdominal swelling from umbilicus to epigastric for 3 months , no
pain, pic given, Swelling is most prominent when his head raise or his leg raise, and also in most
prominent with coughing, what is your most appropriate management
A. Open mesh herniorraphy
B. Hernioplasty
C. Laparoscopic repair
D. Physiotherapy
E. Reassurance
(Photo was given here.)

95. Doctor was trying to educated an obese lady. (I remember so.)


What is the most important indicator in finding health risk of obesity?
A. BMI
B. Waist Circumference
C. Waist hip ratio
D. High LDL
E. Low HDL

96. 8 years old with type 1 diabetes. On small and intermediate acting insulin. every morning high
glucose levels. what to do?
a. check blood glucose levels at 2:00-3:00 am for 3 days
b. check early morning insulin levels
c. increase the evening intermediate insulin
d. give insulin before breakfast
e. give another dose before sleep

97. The patient now presents with tiredness and fatigue. He had history of CLL. Recently (maybe
some weeks ago?), he presented with rash and treated by meropenum and azithromycin. What to do
now? (Please check others’ recall for this scenario for I am not sure about it.)
1. Skin biopsy
2. Drug allergy (test)
3. CMV PCR
4. Bone marrow examination
5. immunological study?

98. Patient with weakness of the left upper limb, weakness of interosseous muscles and right plantar
response is equivocal, left is increased; reflexes are normal. What investigation will you do to reach
diagnosis?
a. MRI cervical spine
b. Ach receptor antibodies
c. EMG
d. CT brain
e. Respiratory function test?
99. Man 1 week after prostatectomy, his wife came to consultation because she is concerned her
husband is acting different, more irritated, aggressive, shouting. what in the history will help u reach
diagnosis?
A. Worsening symptoms at night
B. marked interpersonal aggression
C. history of cruelty towards animal
D. history of drug use

100. You are intern. The patient is intubated and one of his family members says the patient wants to
change the will. What should you do?
1. Witness the will
2. Refuse to witness
3. Ask surgical register if he can do it
4. Seek legal advice
5. Ask the nearby kins and witness the will

101. Peptic ulcer case with H. pylori positive, took triple therapy (amoxicillin+metro+PPI) for two
weeks 6 weeks after, urea breath test still positive after completion of course .asking the reason-
a. resistant to metronidazole
b. Resistant to amoxicillin
c. unreliable urea breath test
d. improper timing of test after treatment

102. 74 year old man brought from resident home, he is masturbating near nurses’ station, tried to
cuddle and kiss the nurses while bathing him. History of Alzheimer’s and is on donepezil and
moclobemide. Diagnosis?
a) progressing Alzheimer’s
b) Side effect of donepezil
c) Side effect of moclobemide (MAO inhibitor)
d) Frontal lobe space occupying lesion
e) Dementia?

103. COPD acute presentation to ED. Oxygen saturation falling, Hypercapnia (+). what do u do
initially?
A. spirometry
B. chest X-ray
C. arterial blood gases
D. ECG
E. sputum culture

104. Young couple with infertility. Female with one child in previous marriage and female
investigations were normal, what in history you will ask the male patient that will be most useful to
guide u to cause of infertility?
A. Do u drink alcohol so much?
B. Did u shave often?
C. Do you use Marijuana?
D. Do u have regular unprotected sex?
E. Do u often use ectasy and party drugs?

105. Woman on HRT on 6 years. well controlled of her symptoms. DEXA scan show femur -1.7.
Vertebra -1.2. How will u manage her?
A. Continue same treatment
B. Cease HRT
C. Add vitamin D and calcium
D. HRT Change to alendronate
E. Add alendronate

106. 45 yr old lady smoker and htn uses ocps for many years now comes re prescription ,she says
doesn't want to come off because it has controlled her menorrhagia and also says doesn't want to
concieve. What to do now
A. Tell her stop smoking then can give ocps
B. Use implanon
C. Use copper iud
D. Use progestogen only pill (safe for DVT)
E. mirena

107. patient with history of claudication, smoke 30 pack cigarette per day, drink alcohol, obese with
diabetic history. he refuse surgery although he was fit for it. He asked you for the appropriate advice
that will improve his symptoms of claudication?
A-Reduce smoking
B-reduce alcohol drinking
C-supervised exercise
D-control his hypercholesterolemia

108. A lady with BMI of 35. How will you manage her in addition to exercise for long term
management?
A. Low Carbohydrate food
B. Lipase inhibitor
C. Diuretics
D. Surgery
E. 4000 kJ/ day

109. Woman whose son is a drug addict, beats her every day. She is afraid and presents to you. What
will u do for the safety of the woman?
A. Tell her that you are obliged to inform the police
B. Go to the police
C. Tell her to go to refugee shelter
D. Advise her to go support group
E. Send the son to correctional facilities

110. 25-year-old boy complain of fatigue. Cousin has blood disorder, require frequent blood
transfusion. On peripheral blood film, there was mentioned target cell found. What will you do next
most app?
A. CBC
B. Serum ferritin
C. HB electrophoresis
D. Bone marrow examination?

111. 58-year-old man having prostate carcinoma with painful bony metastasis taking opoids/
morphine for pain relief for 12 months but from previous 1 month, his pain is not controlled by
anything. Now he comes with agitation, insomnia and reduced feeding. What therapy should you
give?
A. Sleep hygiene
B. interpersonal therapy
C. Positive analytical approach
D. Supportive psychotherapy
E. Dialectal therapy
112. . Pt on regular ACEI, Metformin, taking amoxicillin 2 days ago, now with painful tongue and
oedema especially on the right side of tongue, on examination pt can’t talk much & point to his
tongue, asking what cause the symptom?
A. Amoxicillin
B. ACEI
C. Metformin

113. A patient presented with macroscopic hematuria. 6 months back he was diagnosed as c-ANCA
positive vasculitis causing hematuria and started on Prednisolone and Cysclophosphamide. He
presents now for hematuria. The RBCs on examination are 20% dysmorphic and 80% normal. What is
the most appropriate next step?
a. Repeat renal biopsy
b. Stop cyclophosphamide ( dx : SLE )
c. CT abdomen
d. Renal USG
e. Cystoscopy
6. Young boy developed pain in the right knee .. mild progressive marked swelling .. no other
systemic symptoms as I recall Blood results given .. anemia .. low platelets .. normal WBCs What
is your diagnosis
Juvenile rheumatoid arthritis
Acute leukemia
Apalstic anemia
SLE

7. Which of the following is notconsidered an extra-articularmanifestation of Rheumatoid
Arthritis?
a) Osteoperosis
b) Peripheral neuropathy
c) Cutaneous nodules
d) Pericardial effusions
e) Hepatomegaly
8. couple came to you they r in relationship with each other from last 4 yrs now they are
planning for pregnancy what will you check in female partner
b) Rubella b) CMV c) Parvovirus d) Toxopalasmosis e) Varicalla

9. Commercial driver had seizure and was adequately treated .. You told him that he souldn't
drive ..however he came back after 3 weeks telling you that his work needs made him drive
yesterday and was no problems at all in that ..he thinks he is totally fit now and can drive
Management
1. Talk to him and tell him how dangerous is that to drive now
2. Call his boss and tell him about the disease
3. Tell him that you have to report what he did for the official driving agency for safety
4. Ask him to give you his license

10. Pregnant lady with full term baby at delivery, what CTG expected to show if
with cord prolapse presentation?
A. early decelerations
B. late decelerations
C. variable deceleration
6. 20 year old came for HPV examination ( pap smear). She is sexually active.
On examination, yellowish exudates obstructing the endocervical space. What
best next?

a) do pap smear as requested

b) tell her it is not beneficial for her age

c) treat infection and perform pap smear after 3 months


7. Which one of the following can be used for data analysis in Cross sectional?

F. Odds ratio
G. Relative risk
H. Chi square test
I. Attributable risk and Chi square test
J. odds ratio and attributable risk
8. A 16-year old female comes in for treatment of severe cystic acne with
Accutane(isotretinoin). She lives alone and is self-supporting with a job as a
waitress. She hasbeen out of her parents'house for a year and pays all her own
bills. you have just fin_ishecl informi'g her of the potentially sevcre
teratogenicitv of isotretinoin. Her acneis severe and she would still like the
isotrctinoin.What should you tell her?

a.This medication cant be taken by women of reproductive age

b. Use benzol4 peroxidc topically instead

c. l will give you the isotretir.roin if your parents accompany you

d. I will give you the isotretinoin as requested.

e. I will treat you with isotretinoin ifyou have a consent signed by your parents

9. Which of the follorving most closely represents the role of risk managernent
in the hospital

a to ensure proper ethical management ofpatients


b. To ensure proper clinical care ofpatients
c. To act as a patient advocated.
D. To minimize the legal risk to the hospital from litigation

10. A young man has a syncopal attack while weight lifting. He has had
similar episodes twice before. His father died of cardiac disease. What is
the most appropriate management? DX HOCM
a) Holter monitor b) Echocardiogram c) Stess test d) BP in supine &
lying down e) CT scan

11. All of the following are true about duodenal ulcer, except? a) Hunger
pain b) Loss of appetite c) Weight gain d) Relapses & remissions e)
Relief by antacids

12. A 68 years old man with COPD was brought to your surgery from
nursing home by ambulance. On his way to hospital he received O2, 10
L/m by mask. He is still unarousable and his ABG most likely-- a) Ph 7.29
PaCO2 65 PaO2 85 b) Ph 7.15 PaCO2 50 PaO2 68 c) Ph 7.25 PaCO2 25
PaO2 100

13. Opening snap indicates: a) Mitral valve mobility b) Atrial fibrillation


causes disappearance of the opening snap c) Replaces S3 d) Best heard
at 2nd right intercostals space

14. A 37-year-old female gives birth to her 2nd child. At the 10th day the
child becomes jaundiced. His mother tells the doctor that her first child
had jaundice after birth and then developed bilateral cataract after the
jaundice. What is the diagnosis? a) Congenital rubella b) Neonatal
hepatitis c) Diabetes Mellitus d) Syphilis e) Galactosaemia

15. The commonest cause of hip pain in a 3-yearold child? a) Transient


synovitis b) Perthes disease c) Slipped upper femoral epiphysis d)
Arthritis e) Tuberculosis

16. Regarding unilateral undescended testis, which of the following is


most commonly associated? a) Malignancy b) Varicocele c) Ingiunal
hernia(Indirect) d) Hydrocele e) Torsion of the testis

17. female had mastalgia for the last 1 year. Conservative treatment has
not worked. How would you manage this patient? a) Bromocriptine b)
Danazole c) Clomiphene d) OCP e) NSAID’s
18. skin condition COMMONLY responsible for persistent dandruff > a)
Irritant dermatitis > b) Seborrhoeic dermatitis > c) Tinea capitis > d)
Lichen planus > e) Pityriasis versicolor

19. Extensive psoriatic lesions on hand and legs. Treatment? > A- UV-B >
B- diathranol > C- steroids > D- erythromycin

20. Australian government appointed you to go through a community


based service and help a local doctor there. Hepatits B data given in a
community of total population of 200. 2009 2010 Antibody +ve cases 10
15 Antigen +ve cases 40 55 Need to calculate the prevalence per 1000
population in the year 2010 A- 350 B- 250 C- 70 D- 50 E- 125

21. 39 weeks pregnant lady came with labour pain. Synto was given appropriately. Ctg was done
which shows heart rate of 140 which dropped to 70 and came back to 140 in 2min. asking next
immediate appropriate treatment.

A. Stop syntocinon

B. Fetal scalp sampling

C. C section

D. Titrate to increase dose of syntocinon

22. weeks pregnant lady came with labour pain..she was put in left lateral position having oxygen
mask in place.iv fluids with syntocinon is running. Ctg was done which shows heart rate of 140 which
dropped to 70 and came back to 140 in 4 min asking next immediate appropriate treatment

A. Fetal scalp sampling

B. Continuuous ctg monitoring

C. Stop syntocinon

D. C section

23. 38 year old g1p0 38 weeks coming to ER after Spontaneous rupture of membrane. After 3 hrs, no
active contractions and synto was commenced.. after 4 hrs assessement, uterine contractions good
but on contious CTG montioring there is 60 bpm with slow recovery to normal. On exam:LOA
position, caput 1+ mould 1+? , cx dilated to 9 cm.What is your next appropriate mx?

Add 1 L of NS
Stop synto

Forceps delivery

Vaccum delivery

Urgent CS

24. Man with nocturia n urinary frequency. mass above the pubic symphysis DRE showed enlarged
prostate with palpable median sulcus palpable.what is the most appropriate next step?

A-trans rectal ultrasound

B-PSA

C-s. Creatinine

D-CT

E-urine cytology

25. Sarcoidosis with skin lesions- skin biopsy

26. Sarcoidosis xray with nodules - dyspnea, 12months loin pain, 24hours hematuria…what inv have
to do NEXT?

a) CT chest b)Serum ACE c)USG of abdomen d) urine r/m/e e)cystoscopy -

27. A child with fever and hx of sore throat 10 days back . He developed pain in wrist later developed
arthralgia n swelling in ankle joint. skin rash present . what's the most initial step of management ?

a. ESR

b. Full blood examination

c. throat swab- i didnt choose this bcoz sore throat was 10 days before

d. USg

28. Viral pneumonia trip to Asia

29. old man stopped taking drugs ,now he has edema up to knee , HR 106 HTN with clear lungs

what next ?

a.fursmide

b.spironolactone

c.digoxin

d.BB
E.recommence ACE ,BB,other drugs but not diuretics

30. 14 y. old girl.. Hx of wt loss...BMI 15..when asked about any interest ,she said why should I? what
sholud this entire world mean?

what is the most relevent to her Dx.?

A. fatigue

B. sleep dist.

C.moving around with friends

D. feeling of inferiority

E.idea of guilt

31. A man aged 64 yr. his psa is 3.8ng now.2 years before 1.5.he did 12 biopsies this time and one
showed a focus of adenocarcinoma and gleason score 4 . What is the management for this man?

A) radical prostatectomy – psa doubling time within 2 tears and young age

B)external beam radiotherapy

C) orchidectomy

D) continued surveillance

32. child with asthma. On exam urinary ketones found and glucose high. Next to follow up

a)Culture and sensitivity

b)Glucose level

c)hba1c

33. A scenario of COPD. Patient was treated with augmentin and some other antibiotcis. Was on
inhaled salbu and ipneb 8 hourly and oral prednisolone 30mg/day. Was not getting better. Had
yellow sputum. Became dyspneic. ABGs done. pH 7.35. pCO2 50. pO2 80. HCO3 35. What to do as
the most appropriate next step?

a) Bronchodilator 4 hourly

b) Change augmentin to ceftriaxone

c) Change oral predniosolone to IV hydrocortisone

d) Admit in ICU for intubation

e) CPAP
34. Old woman 75years is admitted to the hospital following a community acquired pneumonia. She
received antibiotics and is well on the 5th after discharge she suddenly develops rigors, chills and
high grade fever. Had crepitations on chest. What is the most likely diagnosis?

A. Hosp acquired pneumonia

B. Iv cannula related bacteraemia

C. Empyema

D. Pulmonary embolism

35. restless leg syndrome- iron studies

36. Hutchinson's freckle- local excision

37. An aboriginal female has mitral stenosis and dyspnea sincedaily 6 weeks. On examination she
had lung basal crackle,and a temperature of 37.9. (Xray looked like pulmonary hypertension – bat
wing appearance, but I could also see a very light haziness in the right apical lung

Whats is the diagnosis.

A. Pulmonary Hypertension.

B. Pulmonary Fibrosis

C. LVF

d. TB

38.Arab woman who doesn’t speak English with 16. weeks pregnancy, was referred by the mid-wife
for which she suspects her mental condition. The woman seems irritable, worried (or anxious) about
the people around and she’s uncomfortable when her husband is not around. She is also irritated
with her 2 children. What condition in her history will be present to lead you to diagnose this
patient? (looks like prodromal symptoms)

1. Panic attacks

2. Paranoid personality disorder

3. Family member with schizophrenia

4. History of trauma

39. A case of lateral epicondylitis. Treatment?

a.Lateral epicondylectomy

b.Immobilization of the fingers

c.Analgesic

d.Sling below elbow


e.Brace below elbow

40. 30.Pt with RA she takes ibuprofen and methotrexate to control her disease ,they mention the
time it was years for both of drugs , pt complains of some symptoms , and her labs are given ALT
,AST GGT all were high , the Q is asking about the cause of this ?

A-Methotrexate induced hepatitis

B-Ibuprofen induced hepatitis

C-Autoimmune hepatitis

DViral hepatitis

41. An elderly woman on polypharmacy GFR was 6 weeks back came with dyspepsia, was put on
Olazpine and anatacids and is now presenting with confusion, loss of appetitie and nausea.

Sodium 129, k 5.3, calcium 3, phosphorus high , creatinine high, urea very high and bicarbonate 20,
what is the reason for the confusion?

a) Uremia

b) hyponatremia

c) hyperk

d) hyper ca

e) metabolic acidosis

42. Osteoporosis with gerd - zolendroic acid

43 osteoporosis with menopausal symptoms_ hrt

44. Menopausal symptoms - dvt with hysterectomy history in the past- transdermal estrogen

45. a term baby with uneventful delivery except for light meconium staining, after 1 min has low
apgar, severe bradycardia- hr 40, next?

a.oxygen with bag and mask

b.intubate

C. Nasopharyngeal suction

46. A case of ankylosing spondylitis, patient not taking any treatment other than physiotherapy pain
not subsiding. Next treatment

1.methotrexate

2.salfasaline

3.indomethacin
4.infliximab

5. Naproxen

47. Female 42 years of age presents with mild hypertension for which she was started on ramipril.
After 2 months she presents with B.P 170/100 , nocturia and Blood chemistry showing increase in
urea and creatinine well above baseline. Which of the following will point towards her
disease?There was Presence of 3 cysts in left kidney and 2 cysts in the right
- pkd

48. abroriginal child comes with purulent ear discharge for 6 weeks what will u do after clearing the
discharge

a) amoxcilin…if acute

b)ear toilet…if chronic >6 weeks

c)prednisolone

d)reassure

e. Cipro ear drops

49. Scenario of acute closure angle glaucoma in old age, haloes around light, red sclera, IOP 25
mmHg, asked long term management?

A. Trabeculectomy

B. Peripheral iridotomy!!!!!

C. Acetazolamide

D. Pilocarpine

50. Acute Gout – joint pain with warmth with ckd - Prednisone

29. A scenario of an old man with history of atrial fibrillation and now comes with severe abdominal
pain. What other than CT angiography will you do?

a) CT abdomen

b) Serum lipase

c) Serum lactate

51. 75 year old man comes to you with progressive constipation for last 3 weeks with absence of
flatus. Abdominal distension is becoming worse. Mild tenderness is present. He had history of
cholecystectomy 25 years ago. Dilated bowel loops along ascending transverse and descending
colon. Rectal examination is empty. What is the diagnosis?

Adhesion obstruction
Sigmoid volvulus

Cancer of Sigmoid colon

Ceacal volvulus

20 April Sydney

I can remember this much now. Please keep me in your prayers. Good luck to all.

1. .Colles fracture recall but with different options xray at 6 week followup was normal and now pt
having mild pain and tingling sensations what to do
A. Xray again
B. splint for few days more
C. occupational therapy to check on him
exercise analgesics
X-ray or MRI to see ligs

2. 16 years old opportunistic screening
a. Skin cancer
b. Scoliosis
c. Depression
The USPSTF recommends the screening of adolescents (aged 12–18 years) for major
depressive disorder when systems are in place to ensure accurate diagnosis, psychotherapy
(cognitive, behavioural or interpersonal) and follow-up (B) 59
Risk factors for major depressive disorder include parental depression, having comorbid
mental health or chronic medical conditions, and having experienced a major negative life
event

This is from the RACGP redbook 9th ed


3 Elderly patient presents with 10 minutes loss of vision and weakness of one half of the face.
How will you investigate?
Carotid doppler
Echo
Mri
Ct ( TIA – amuorosis fugax )

10 minutes weakness of right side of body with some speech trouble. normal with no carotid bruit
and blurring of vision. .Now normal What could be ?
a) Lacunar infarct
b) Vertebro basilar
C) ischaemia caotid artery
2.A patient comes with difficulty in speech and weakness in the side of the body for 10 min.No
loss of vision and no bruit in neck.Diagnosis?
Carotid artery stenosis
Vertebra basilar ischaemia
Lacuna infract 1431jm

4. Stop syntocin 2minutes and 4 minutes recall

5. Groin lymphnode. Primary lesion? Left leg

6. A student after the exam felt strong headache.. He presented to the doctor with belive that he has a
tumor in his brain.. Ct scan done everything was normal. What is thr type of disorder?!
A. Somatization disorder
B. Somatic disorder
C. Hypochondriosis
D. Forgot

7.Pregnant lady with 16 weeks come to you for Down syndrome test. Which test to do?

Amniocentesis

8. Hutchison frecke pic.local excision

9. Pic of hand with polyarthritis scenario asking treatment

10. Picture of SLE face rash asking for long term treatment

SLE diagnosed(anti DtsDNA+ and ENA+)hving pain in joints.wht long term treatment?
a.Naproxen
b.Methotrexate
c.Hydroxychloroquine
d.paracetamol
53. SLE -Rash and Joint pain .Best Nxt?
a.predisolone
b.Hydroxychloroquine
c.Methotrexate
54. Pic of female with SLE with malaria rash. Diagnostic investigation?
ANA
Anti Smith
ANCA

11. Scenario of ADHD boy asking for diagnosis.

12. Scenario of cluster headache asking investigation-trial of 100% oxygen

13.ecg WPW asking treatment

14. Old women with alzheimer accusing people of stealing her things- worsening of alzheimer

15. Old man with MMSE 26/30. He still drives.What to do next-

Cease driving at night

Old man presented with his wife. Wife says he sometimes gets confused and forgetful. He still
drives andhas stopped reading newspapers. MMSE was 26/30. CTscan of the brain is normal.
What is the most appropriatemanagement?
a. Cease driving at night
b. Repeat MMSE after 3 months
c. MRI of brain
d. Encourage physical and mental activities
e. exclude treatable causes of dementia

Encourage to do more physical and social activity

16. 42 years old lady come to you. Mother has hTN from age 55years.what check she need now

17. Boy with anger outburst. What is management

-carbamazepine

Na valproate

Risperidone ( dx ODD )

18.child with growth chart. 30 kg, bmi 19.5.

BMI-age chart given. 7-year-old boy 30kg wt with BMI 19.5. which of following is correct?
a. Underweight
b. Overweight
c. Normal wt
d. Obese
e. BMI is not applicable in children


19.pt with melancholic depression. Whom to seek permission before giving ECT?

Permission from mental health tribunal

Under duty of care

From wife

20. Man with HTN, HB A1c is 6.2. Heavy smoker. What next?

-assess his interest to stop smoking

21.same scenario with 6.9 hba1c. Use metformin.

-i chose increase metformin

22. Girl with hematuria. Urine analysis showed non glumerular rbc and no cast.

22.statistics ques. Dental amulgam and dementia

23. Rotavirus and low birth weight baby. What study to do?

24.82 years old man comes with wife. Wife accused him of physical abuse during cooking.What is the
most likely cause- elder abuse, dementia

25.ECG with ramipril and K 5.5. What next

26. 2 carcinoma sigmoid colon scenario

27. Diverticular ct with next treatment- antibiotic and gut rest

28.18 years old with single painless ulcer. rpr positive with 1:64 titre. what is immediate management?

im penicillin, notify public health, perform contact tracing, do other std screening

21 APRIL 2018 - SYDNEY

121. Picture of red eye with scenario said acute painful red eye, no discharge. Case
mentioned old age came to hospital at night. Which eye drop is used for this condition?
a. Timolol
b. Prednisolone
c. Chloramphenicol
d. Atropine
e. saline

122. Female patient with RA she takes ibuprofen and methotrexate to control her disease
for 5 years. Recently patient complains of some symptoms (jaundice), and her labs are given.
protein - 9 (6-8) slight increased
albumin- 5 (3.5-5.5) normal
ALT - >100 (7-56) Liver enzymes >100 in my exam question
AST - >100 (10-40)
GGT - >100 (0-30)
What is the cause of her symptoms?
a. Methotrexate induced hepatitis
b. ibuprofen induced hepatitis
c. Autoimmune hepatitis
d. Viral hepatitis

123. Woman known with polymyalgia rheumatica and has prednisone 10 mg daily,
antihypertensive medication for HTN and metformin for DM, she feels dizzy and unwell as
she went overseas and didn’t take medication at all. Labs- Glu 4.0, Na 125, K 5.0 diagnosis??
a. Adrenal insufficiency
b. Indapamide
c. Metformin
d. Lisinopril

124. 75 years old man present with severe retrosternal chest pain. pain is preceded by vomiting.
with BP 100/70 mmHg, dull on percussion and reduced breath sound at base of the lung. he
has HTN DM asking most diagnostic test?
a. non-contrast CT
b. electrocardiogram
c. chest X-ray
d. Gastrograffin swallow

125. Patient after pacemaker insertion presents with difficulty in breathing and chest
pain. faint heart sounds +, spo2 96%, bp-90/60mmhg, JVP-6 with no ECG given.
Investigation of choice needed.
a. Chest X-ray
b. CTPA
c. ECG
d. ECHOCARDIOGRAM
e. SERUM TROPONIN

126. Diabetic patient well controlled with mild renal impairment (given Creatinine level
but I forgot). Which of the following risk will have in baby?
a. IUGR
b. Macrosomia
c. Renal agenesis
d. Intrauterine fetal demise
e. Low Birth weight

127. Old age, History of appendicetomy & cholecystectomy presented with 3 weeks history of
abdominal distension on ascending colon, transverse colon, descending colon, rectum is
empty. There is mild tenderness of the abdomen and loud borborygmi. What is the
diagnosis? (CT was given) CT unclear for me à I choose CA sigmoid according to scenario.
a. Sigmoid volvulus
b. Adhesive IO
c. CA Sigmoid
d. Fecal impaction
e. Caecal vovulus

128. Insufficient milk production, reason?
a. insufficient fluid intake
b. not putting for sufficient frequently
c. not putting for long time during feeding
d. exhausted
e. poor positioning

129. Old female with depression for 6 wk after dead of her partner ,sleeps badly, she lives alone
now , she had the same symptoms in the past when she lost her child , what is the prompt Rx
a. Olanzapin
b. ECT
c. Midazolam
d. Venlafaxine
e. temazepam

130. 39 weeks pregnant lady came with labour pain…on iv fluids with syntocinon is running 5 unit
in 500 ml of Ringer latate.with irregular uterine contractions. Ctg was done which shows heart
rate of 140, which dropped to 70 and came back to 140 in 2min.what is the most appropriate
next step.
a. give oxygen to mother via face mask
b. stop syntocinon
c. C section
d. fetal scalp sampling
e. Titrate to increase dose of syntocinon

131. Similar questions but they didn’t mention came back to normal. They just mentioned heart
rate dropped and then “slowly progressing”.
a. give oxygen to mother via face mask
b. stop syntocinon
c. C section
d. fetal scalp sampling
e. Titrate to increase dose of syntocinon

132. Lady full term with adequate contraction was monitored on CTG. Everything was
progessing normally. Suddenly there was a deceleration of FHR dropping to 70bpm for 4
mins. O2 is given to mother, what is the next most appropriate step?
a. Stop syntocinon
b. Continue synto at an increased dose
c. Immediate caesarean
d. Ventouse delivery

133. patient has acute gout with h/o chronic renal failure and previous h/0 gout. what should be
given-
a. allopurinol
b. NSAID
c. Colchicine
d. Ibuprofen
e. prednisolone

134. An 18 year old girl presents with epistaxis, malaise and tiredness for months. no other
symptoms . Hb is 8.5, microcytic hypochromic picture. INR is 1.5. Calcium is 1.9. What is nex
most appropriate to reach diagnosis
a. Hb electrophoresis
b. serum electrolytes
c. Iron studies
d. anti glidian antibods
e. stool culture

135. Farmer came with left swelling in a groin. U did FNAC and it shows squamous cells. Where
is the lesion?
a. Left leg
b. Anus
c. Rectum
d. Testis
e. Penis

136. A mother come with her 8 months old child complaint of continuous bleeding after fall
from coffee table. On examination there is bleeding frenulum and some old bruise in
forehead and leg. There was no petechiae, lymphadenopathy, hepatomegaly. What is the
cause?
A. ITP
B. Non accidental injury
C. VWD
D. Hemophilia A
E. Hemophilia B

137. Study on Aspirin effects on preventing MI on 100 people. If those who took aspirin of the
100 ppl only 1 person had a MI. Of those who didn't take aspirin 2 ppl had MI. What is the
increase in relative risk in people not taking aspirin?
a. 1%
b. 10%
c. 100%
d. 200%

138. scenario with female patient always angry, and can’t control herself, unstable
relationships. Self-harm was there, treatment was asked
a. anger management
b. dialectal behavioral therapy
c. Psychoanalytical therapy
d. Interpersonal therapy

139. 40 years old lady with back pain at L4 level with severe shooting pain to leg to toe
next, no neurological deficit
a. Observation
b. X-ray lumbosacral
c. CT lumbosacral
d. MRI lumbosacral

140. An old man with acute onset pain in the lower back with urinary symptoms.
significant weight loss. On DRE, irregular prostate +. What is your appropriate investigation
to lead you to diagnosis?
a. chest x-ray
b. CT spine
c. TRUS
d. bladder scan
e. PSA

141. A child 6 years old with history of asthma presented with upper respiratory tract infection,
urine examination showed very high serum glucose and ketone body of 2+.
What is the most appropriate test to follow up this child?
a. HbA1C
b. Serum creatinine and electrolyte
c. FBS
d. OGTT
e. ABGA

142. 8 years old with type 1 diabetes. On small and intermediate acting insulin. every morning
high glucose levels. what to do?
a. check blood glucose levels at 2:00-3:00 am for 3 days
b. check early morning insulin levels
c. increase the evening intermediate insulin
d. give insulin before breakfast
e. give another dose before sleep

143. Child (3-4 year) with fever 39C and history of cough unwell for 3 days. I think he has
wheeze but minimal lung signs on exam. CXR given with pleural effusion. Asked which
investigation what u do for further assisting in diagnosis? (similar with this picture)
a. pneumococcal PCR
b. pleural aspirate
c. blood culture
d. FBC

144. A man with history of limb claudication on 100 meters relieved by rest, on examinations
there was absent left femoral pulse and absent dorsalis pedis pulse, ABI was done and it was
0.25. What is the most appropriate test leading you to the diagnosis?
f. Arteriography
g. Ct angiography
h. compression Doppler ultrasound
i. MR angiogram
j. X-ray

145. old patient who drinks about 10-12 standard drinks per day and also smoker,
underwent some surgery. on 3rd post op day he developed agitation ,O2 saturation 88%.no
fever. what next ?
a. blood alcohol
b. blood glucose
c. CTPA-best
d. XRAY CHEST
e. urine culture


146. 72 year old man living alone is brought to ED by daughter. He’s so confused and
gave incomprehensible answers to questions and couldn’t follow any commands. There’s no
other abnormality on examination. CT was given large dark area is in left temporo- occipital
region with shifting of left lateral ventricles plus surrounding oedema. Asked diagnosis
a. Cerebral tumour
b. Cerebral infarction
c. Cerebral haemorrhage
d. Cerebral tuberculosis
e. Subarachnoid haemorrhage

147. 42yo woman who smokes 20 cigarettes/d presents with complains of heavy bleeding
prolonged menstrual period. What is the most appropriate treatment for her?
a. Tranexemic acid
b. COCP
c. Mefenemic acid
d. IUCD
e. Norethisterone

148. young girl Came with menorrhagia. Absent from work because of menses pain & heavy
flow.
From history & exam, cannot find cause. You ordered several lab investigations but she
insists to give medication. Her cycles were regular
a. OCP
b. Oestrogen patches
c. Mefenamic acid during period
d. tranexamic acid during period

149. 45 yr old lady smoker and htn uses ocps for many years now comes re prescription ,she
says doesn't want to come off because it has controlled her menorrhagia and also says
doesn't want to concieve. It mentioned that she doesn’t want to change method of
contraception. What to do now
A. Tell her stop smoking then can give ocps
B. Use implanon
C. Use copper iud
D. Use progestogen only pill
E. mirena

150. Woman with a history of hysterectomy and DVT. Now complaining of hot flushes and
insomnia. What will you give her?
a. Oral low dose estradiol
b. Transdermal estradiol
c. Progesterone only pills
d. HRT
e. Clonidine


151. mother with hepatitis C, what will reduce vertical transmission?
a. Monitor fetus with scalp clipping should be avoided
b. Caesarean section is indicated
c. Avoid breast feeding

152. Patient with deep jaundice, moderate ascites, and confused complaining of abdominal pain
due to tense ascites. Flapping tremors and hepatic encephalopathy feature. Liver is difficult
to palpate. Complaint of high fever 38’C. What is your next appropriate initial step?
a. abdominal paracentesis
b. Blood culture
c. USG
d. Fluid restriction
e. IV antibiotics

153. scenario where a girl came with menorrhagia ,looking pale and hypocalcemia there
lab value was inr- 1.9 with no S/s.bt another thing was there was micocytic anemia,asking
for what to do?
a. hemolytic screen
b. iv calcium injection
c. blood electrophoresis
d. give vit k injection
e. give FFP
f. higher percentage of unexplained DUB has been observed in celiac
women. All celiac patients were undertaking gluten free diet for at least 3
months and the celiac patients who reported the history of DUB were
again interviewed for any signs of unexplained DUB
due to vit k ,calcium and other nutritional deficiency

154. One ECG picture (no obvious changes, a bit tall T in V2,V3). Serum potassium 5.5, patient
has no symptom. He is on Ramipril and other drugs. Asking what to do next?
a. IV glucose
b. Cease Ramipril
c. Iv Calcium gluconate
d. Resonium

155. 7 days old infant presented to you complained with poor feeding and bile stain vomiting.
His birth was at 38 weeks gestation and weight 2600g.He is breastfeeding with no
immediate post natal complications. The child has a history of passing meconium on day 4 of
birth. Now, he had mild jaundice and abdominal distension present. What is the appropriate
diagnosis?
a. Meconium Ileus
b. Hirschsprung's Disease
c. Duodenal Atresia
d. Necrotizing Enterocolitis
e. Volvulus

156. Lady comes on 10th post-partum day with pain in perineum tear. History of perineum
laceration during delivery which was sutured. Now on examination there is a perineum
wound 2 cm long 1 cm wide and 1 mm deep. Clean with no discharge. What is next?
a. Apply local antibiotics
b. Put on oral antibiotics
c. Simply keep the wound clean
d. Suture the wound under LA
e. Suture the wound after cutting the edges

157. 3yr old boy with ear discharge for 6 weeks no fever. Asking treatment after ear toilet is
done.
d. Po Amoxil
e. Ciprofloxacin ear drop
f. Other antibiotics

158. Woman around 30-35 years of age attends for Pap smear, Her previous smears are all
normal. She doesn’t have sexual activity in last two years. What’s the reason of testing
chlamydia in her?
a. chlamydia is asymptomatic
b. chlamydia can cause infertility
c. Both partners could be asymptomatic
d. No need for screening

159. A 6 month old child came with recurrent upper respiratory infection. The immunization
history reveals that the child has received his hepatitis B vaccination at his birth and no
further immunization history at 2 and 4 month of age due to recurrent upper respiratory
infection. Now the child has fever 37.8 and clear nasal discharge. What is your management
regarding immunization?
A. Give hepatitis B immunization and institute catch up schedule
B. Give hepatitis B immunization and recommence standard schedule
C. Investigate underlying cause of URTI
D. Give hepatitis B immunization after fever subside
E. Give hepatitis B immunization at now

160. A case of gynaecomastia pt on risperidone. What other drug to counteract the side effects
a. Quetiapine
b. Aripipazole
c. Olanzapine
d. Clozapine
e. Mirtazipine

161. Woman 51 years of age presents after menopause for the last 8 months. She complains of
irritability, moodiness, low libido and hot flushes. Which of the following would be most
appropriate to give to this woman?
a. Combined continuous estrogen and progesterone throughout the cycle
b. Continuous estrogen throughout the cycle
c. Continuous estrogen with progester one first 12 days of the cycle
d. Continuous progesterone with testosterone

162. Old lady 58yrs with urinary incontinence, when she makes some stress such as
cough, laughing, urine flow out. On examination she has only rectocele, no other cystocele.
told to do pelvic floor exercise and a little improved but still incontinence and embarrassed.
She is not satisfied. She had DM and well controlled with diet. What is the next most
appropriate treatment in this patient?
A. Weighted vaginal cone
B. Bladder neck suspension
C. Imipramine
D. Oxybutyline
E. Surgery for rectocele

163. Old man presented to you complaining of rest tremor and bradykinesia, also taking
respiridone and he has a known history of visual hallucination and forgetfulness. what is the
most likely dx?
a. parkinson disease
b. lewy body dementia
c. alzheimer disease
d. acute delirium
e. schizophrenia

164. 24 months old, developmental task for language?
a. 2 or more personal pronouns
b. Sentence with 4 syllables
c. Name 4 color
d. Know age
e. Know family name

165. Patient with weakness of the left upper limb, weakness of interosseous muscles and
right plantar response is equivocal, left is increased; reflexes are normal. What investigation
will you do to reach diagnosis?
A. MRI cervical spine
B. Ach receptor antibodies
C. EMG
D. CT brain

166. A young man brought to ED after brawl in bar in which he hit someone. He is
alcoholic, take multiple drugs and aggressive. What in history will you to know if he has
personality disorder?
a. Childhood sexual abuse
b. H/o cruelty to animals in adolescence
c. Drug and alcohol dependence
d. H/o of hitting partner one week back
e. H/o depression in mother (shahriar vai er coaching search )

167. a man works whole day in a kitchen cabinet, he complains of pain in elbow.
Tenderness in lateral side of elbow. what the cause of this pain.
a. bursa inflammation
b. tendon rupture
c. fracture
d. lateral epicondylitis

168. Patient complaining of soreness on lateral epicondyle at the end of work,the patient
attributes it due to computer mouse clicking. On examination , tenderness on lateral
epicondyle .Asking treatment?
a. lateral epicondylectomy
b. finger immobilization splint
c. lateral epicondyle immobilization splint
d. bracing under lateral epicondyle

169. 3.2 kg baby was delivered vaginally with no complication except for slight meconium
stained liquor, normal fetal heart rate at birth, after one minute, become limp, cyanosis,
decreased HR, what is most appropriate next step?
a. Intubate
b. Bag and mask ventilation
c. Aspiration for meconium
d. CXR
e. Na bicarbonate to infant

170. A young girl who is 9 years of age with history of recurrent URTI last time was two weeks
ago and did urinalysis at that time showed blood (3+), proteinuria (1+). Now she came after
two week. Urinalysis is unchanged except for Blood (2+) which is non-glomerular and there
are no casts in the urine. Which of the following is the next best investigation?
a. Urine culture
b. DMSA
c. Ultrasound
d. IVP
e. ASOT

171. enlarged gall bladder with multiple stones, found dilated bile duct and stone obstruction in
common bile duct, patient developed toxic shock, what to do after fluid resuscitation?
a. Laparoto cholecystectomy
b. laparoscopic bile duct explores
c. percutaneous bile drainage
d. ERCP

An appropriate diagnostic description of the syndrome of biliary tract


infection and septic shock should therefore include a description of the
underlying biliary disease as well as the term acute biliary shock. In this
series, emergency surgical management by removal of gallstones and
drainage of suppuration was felt to be the most appropriate treatment

172. Woman whose son is a drug addict, beats her every day. She is afraid and presents to you.
What will u do for the safety of the woman?
a. Tell her that you are obliged to inform the police
b. Go to the police
c. Tell her to go to refugee shelter
d. Advise her to go support group
e. Send the son to correctional facilities

173. Picture showing dupuytren contracture, Farmer with inability to extend his left ring finger,
history of cut injury in that fingers 3 months ago. He drinks 4 cans of beers per day and 30
pack of smoking per year. What is the cause of his injury?
a. chronic infection
b. His farmer occupation
c. Alcohol
d. familial history
e. Genetic

174. A new screening procedure has been introduced.which point indicates 100%
sensitivity of the new technique-

Points Comment
A 0% Specificity
B 100% Sensitivity, 50% Specificity
C 75% Sensitivity, 75% Specificity
D 100% Specificity, 50% Sensitivity
E 0% Sensitivity( have to memorise )

175. Old woman with GERD history and taking PPI, T Score given -2.5 in vertebra and -2.5
in femur , in labs
a. details was given , ca vit d was normal and asking treatment
b. alendronate
c. HRT
d. Strontium ranelate

176. Lateral epicondylitis and brace around elbow


A case of lateral epicondylitis. Treatment?
a. Lateral epicondylectomy
b. Immobilization of the finger
c. Analgesic
d. Sling below elbow
e. Brace at the elbow

20th April 2018, Sydney.

A.K.M. Hasan Siddiquei

ALHAMDULILLAH. Got common topics all together. Got less recent recalls and less questions
from Handbook. Can’t revise the paper but otherwise exam was good. Please keep me in
your prayers. THANK YOU.
1. Xray given (joint space is a bit narrow , may be little displacement) Football player ,
can’t stand , painful ankle joint movement , ask diagnosis ?
A. Fibula fracture
B. Fibula fracture with fracture articular surface of tibia
C. Fibula fracture with fracture articular surface of tibia & joint
displacement
D. Fibula fracture with joint displacement
E. Fibular fracture with displaced mortise

2. Central retinal artery occlusion


3. Central retinal vein thrombosis

4. Retinal detachment

5. Psoriasis pic asking treatment –


a. Bactroban
b. Chloramphenicol
c. Betnovate
6. old pt with hamaturia ,increase urea ,creatine ,arthralgia,footdrop,what will lead u to
dx:
a.Sural nerve biopsy
b.Renal biopsy
c.ANCA ( dx depends on stem )
7. Microscopic polyangiitis (Dx Given), asking for Ix-
a. Anti CCP
b. ANA
c. ANCA
8. 4 Young girl presented to you with heavy menstruation since 2 years. She also
complains of pain during her periods. She is pale on examination.You told her that
multiple investigations are required but she refused and asked for something to help
with the bleeding.
A. OCP
B. Tranexamic acid
C. Mefenamic acid
D. Mirena
9. A 42yo woman who smokes 20 cigarettes/d presents with complains of heavy
bleeding and
prolonged menstrual period. What is the most appropriate tx for her?
a. Tranexamic acid
b. COCP
c. Mefenamic acid
d. IUCD
e. Norethisterone
10. Old patient with ischiorectalabscess, has this problem recurrent many times, what’s
the cause?
A. Anal fistula

B. Diverticular disease

C. Crohn’s disease

D. Diabetic

11. Patient with perianal pain with h/o of constipation, examination not possible d/t pain,
treatment asked?
Glyceryl trinitrate cream (anal fissure)
12. 6 yr old girl with hereditary spherocytosis came with marked pallor and palpable
spleen, history of URTI last week lasting for 3-4 days .
HB – reduced (more than 8.5)
WBC – normal
Platelet – normal
Retic count – 0.1 %
a.adeno virus
b.cytomegalovirus
c.parvo virus
13. Patient writes bad things about doctor on facebook. What to do:
Inform defense bureau (something like that)
Give feedback on the post and explain
Write to ask him to remove post
Ignore
14. A man aged 68 years. his psa is 3.8ng now. last 2 year(sure) it was 1.5.he did 12
biopsies this time and one showed a focus of adenocarcinoma and Gleason score 4.
he had operation for cardiac stent placement 18months back & on antiplatelet since
then.What is the management for this man?
A) radical prostatectomy
B) external beam radiotherapy
C) orchidectomy
D) continued surveillance
15. Child 6 years old with history of asthma presented with upper respiratory tract
infection, urine examination showed very high serum glucose and ketone body of
2+.
What is the most appropriate test to follow up this child?
A. HbA1C
B. Serum creatinine and electrolyte
C. FBS
D. OGTT
16. You are intern. Patient wishes you to witness
her will in the presence of lawyer.
What should you do?
1. Witness the will
2. Refuse to witness
3. Ask surgical register if he can do it
4. Seeklegaladvice
17. 5 yr old child with lacerated wound. Receives DTPa at 2 month and 4 month after
that no booster. What to give him now ?
a. TT or TIG
b. DTPa now and booster after two months
c. TT +topical antibiotic ointment
d. DTPa andTIG
18. Women feels pain in abdomen during gardening. pain settles down but happens
again and again for half hour episodes. On USG examination dilated small bowel
loops and air in biliary tree. What is the most likely diagnosis?
a. Cholelithiasis
b. Pancreatitis
c. Crohn’s ileitis
d. Mesenteric ischemia
19. 32 year old history hysterectomy and oophorectomy for menorrhagia has -2.8 Z
score for bone marrow. ca & vitamin D normal. What to give?
a) Vit D
b) Calcitriol
c) Estrogen
d) Estrogen and progesterone
e) Alendronate ??????
20. Post hysterectomy patient with history of DVT post-operative after hysterectomy.
Now comes with hot flushes. Which HRT is best?
a.Low dose Oral estradiol
b.Oral estrogen plus Progesterone
c.Oral Progesterone alone
d.Transdermal estradiol

21. Invasive intraductal carcinoma with no lymph node involvement, Mx asked-


a. Total mastectomy
b. Partial mastectomy
c. Radiation
22. pt with h/o hemicolectomy for diverticulitis and colostomy done since 5 days , now
there is serosanguinous discharge(key word) came around the colostomy , the
wound edges are hyperaemic ,erythematous asking for diagnosis :
a-wound dehiscence

b-wound haematoma

c-small intestinal fistula

23. 50 year old man present with hypertension with asthma and reflux nephropathy.
labinv were given. There was high urea,high creatinine and proteinuria 900 mg/day.
What is the choice of anti HTN?
1.amlodipine

2.losartan

3.perindropil

4.indapamide

5.metoprolol ( no ACE as causes dry cough – in ques asthma )

24. Female with 5 m baby withbronchiolitis after treating the baby u found that his
booklet of immunization isn’t checked for any immunization u have short
conversation with the mother and she said that she don't want to immunise him
what to do:
a) Reinforce her wish

b) try to persuade her

c) call child protective authority

d) Respect her wish

e) Call police

25. Child with systolic murmur on left sternal border and no cyanosis feature, involve
whole precordium, Dx-
a. ASD
b. VSD
c. TGA
d. TOF
e. Physiological
26. Intern at the hospital. He knows about all the cases of needle stick injury in the. All
cases in the hospital were manage correctly with no big problem. Now there is
accreditation and the intern was asked about this issue. What the intern should
answer:
1. Give info about the protocol of management
2. Leave the answer to your supervisor
3. Tell a case that he knows
4. Say that there’s no problem with needle stick injuries in this hospital
27. Case of one colleague smoke Marijuana and some depression s/s, what you will do?
a. Speak with him regarding cease marijuana otherwise I will inform the
authority
b. Inform medical board
28. A man with dyspnoea and SOB , taking metformin and rosiglitazone for diabetes,
amiodarone for recurrent ventricular tachyarrhythmias, enalapril and metoprolol.
Which drug may have caused these symptoms?
A. Amiodarone
B. Metformin
C. Rosiglitazone
D. Enalapril
E. Metoprolol
29. Recall on treatment of chronic osteoporosis with BDS results of 2.7 and 1.7, vit D &
Ca level normal, I think options were?
A) alendronate

B) colchicine

C) naproxen

D) prednisolone

30. pregnant woman, health care worker, came to advise for influenza vaccination. She
is at risk of getting influenza. what you should do?
A- give influenza vaccination
B- influenza shouldn't give to her
C- influenza vaccination after birth
31. Know case of chronic liver disease, comes with bilateral parotidomegaly ,
gynaecomastia , spider naevi , left supraclavicular lymph node enlargement
,moderate ascites , what clinical feature favours need for further investigation?
A. Bilateral parotidomegaly

B. Gynaecomastia

C. Spider naevi

D. Left supraclavicular LN enlargement

E. Ascites

32. A man with type 2 DM taking insulin complaints of severe night sweats. His present
daily dosage is ISOPHANE 20IU morning LISPRO 15IU (not sure) evening what
changes will you make in his dosages?
a. increase morning dosage of lispro
b. increase morning dosage of isophane
c. reduce evening dosage of lispro
d. reduce evening dosage of isophane
33. A woman presents with her daughter and was diagnosed of a malignant cancer, she
is not in good terms with her husband which you are aware of. Husband also is your
patient. He presents asking for the results of his wife’s test. What is your reaction?
Inform husband of the test results
Tell husband u will call wife to get permission and give him the test results
Tell husband to discuss this with his wife
Obtain permission with medical board
34. A man with swelling in the lateral part of his neck as shown, how to diagnose?
Ask him to swallow saliva and observe movement of tongue
Ask him to drink water and observe tongue movement
The rest not relevant

35. A pt had knee injury knee is swollen, & medial ROTATION PAINFUL. cause TEAR
OF??
A. ACL
B. PCL
C. Medial meniscus
D. Lateral meniscus
36. YOUNG MALE after an episode of Severe bout cough developed chest pain, after
12hrs he went to ED,it revealed 15% pneumothorax, where they gave him morphine
after analgesia. What initial management you will do
A. Reassurance
B. Admit and review in 24 hrs
C. Aspirate and review
D. Chest tube insertion with water seal
37. ECG with STEMI had low blood pressure, sweating, chest pain pulse 100, best mx?
A echo
B cardiac enzyme
C refer to catheter Center(golden hour)
38. Pt came with restlessness, fainted few times in last 24 hours, 5 ECG stirps was given
–Choose “Heart Block” strips
39. Sarcoidosis xray with 3yrs dyspnoea, 12months loin pain, 24hours haematuria…what
inv have to do next?
a) CT chest
b)Serum ACE

c)USG of abdomen

d) urine r/m/e

e)cystoscopy

40. man recently migrate to Australia married say that people in his country are after
him.... blablabla.... what to check in the collateral hx to reach Dx?
A. family hx of MOOD disorder
B. low sociability in the primary school
C.immigration visa status
D.the truth of his saying
E. hx of previous physical trauma
41. 10 days old child, mother bring 5th time and saying child is crying and difficult to
cuddle, on exam no problem with the child. growth is normal. what clues give to aid
the dx?
A. premorbid personality of the mother

B. parental divorce

C. weight loss of the mother


D. thought of harming baby.

E. her child care skill.

42. Short term dementia MMSE given 24, wants to change will. Daughter says it would
be illegal to do so
A. She can change her will
B. She can't
C. Refer for neuropsychiatric review
D.Ask lawyer
43. Recall on mass on the left side of scrotum, cannot get above the mass. Asking for
management
A) usg

B) surgical exploration

C) Orchidectomy

D) FNAC

44. 35 year old pregnant woman presented at 15 weeks’ gestation asking you for a
diagnostic screening of down, what is the diagnostic screening test you should do for
this woman?
A-nuchal translucency ultrasound

B-maternal serum and ultrasound

C-maternal serum

D-amniocentesis

E-chorionic villous sampling

45. Eye pic of an ild main with sudden o set of pain, dilated pupil and red eye. T/m?
1-Atropine

2-Timolol

3- pilocarpine

4- trabeculoplasty

5- steroid

46. Immigrant lady with syphilis, was separated from her husband for 6 years during
which she was raped, beaten, comes to you with her husband, husband agrees to
get tested, is negative for syphilis, what advise do you give to them –
a. use condoms for six months

b. get tested for syphilis antibody after 3 months

c. treat the husband as well


47. Male came from Thailand 2 months ago .. presenting now with marked confusion
and marked lymphadenopathy more at the axillary and inguinal groups (may be mild
hepatosplenomegaly .. not sure) .. CT showed bilateral periventricular lesions (no
details mentioned) .. what inv to do next
Test for HIV
Test for IMN
Test for Malaria
MRI of the brain
48. 40 year old woman of retro orbital pain and reduced visual acuity for 3 days. What is
the most appropriate ix?
1. Temporal artery biopsy
2. CT scan
3. VEP
4. Fundoscopy
5. Gonioscopy
49. Pt came with fever, weakness, fatigue, some infection, lab Ix-Hb reduced, WBC
increased, Neutrophil increased, Dx asked-
a. ALL
b. CML
c. Multiple myeloma
d. ????
50. child with fever drowsiness n cold extremities
Rash was mentioned asking treatment

a. Iv cefatholin
b. Iv gentamicin
c. Oral penicillin
d. Oral roxithro was there
51. Elharhossain
52. Q1. Pregnant 32 ws as part of program of flue vaccine what option u choose
a. Defer to after pregnancy
b. wait until u have the infection and use I forget something like passive immunity.
c. take the vaccine now
d. defer to after a month
53. Q2. woman 45 or 50 ys present with dysuria, freguency and enuresis, urine analysis show
leucocytes and nitrite positive. what treatment options
a. methoprim
b. penicillin
c. tetracyclin
d. I forgot others
54. Q3. Kid has swelling behind his ear that is tender, fever. O/E tender and red. What could be
the cause:
a. staph
b. h influenza
c. streptococcus pneumonia
d. strep pyogen
55.
56. Q4. Patient male depressed wife left him, drink alcohol, smoker, he has chronic lower back
pain for which he is taking paracetamol, morphine tab, others I forget today he ate all
medication in his bathroom feel drowsy nausea depressed respiration. EMT gave him
naloxone. When arrived ER he still have same symptoms with depressed respiration. asking
options:
a. give him more naloxone
b. check his paracetamol level
c. give him NAC
d. I forget others
57. Q5. Patient I forget the cenario but he has loss of planti flexion, and inversion, also loss of
unkle jerk; But he has intact dorsiflexion and eversion. Knee jerk normal. Which nerve injury:
a. sciatic
b. tibial
c. common peroneal
d. forget others.
58. Q6. Patient complaining of loss of sensation over lateral arm, lateral forearm, loss of triceps
jerk, asking about the site of injury
a. C5,6
b. C6,7
c. C8, T1
d. Can’t remember
59. Q7. Pt presented with amenorrhea, taking resperidone for what ever, has whitish discharge
from nipples, investigation given show prolactin level 2500 highly increase, other lab work
normal. Vitals given all within normal limit; CT of the pituitary gland done and was
normal.asking about the cause:
a. microadenoma that is very tiny
b. resperidone
c. forget
60. Q8. Ct scan given show solitary mass in the liver asking Dx
a. hepatoma
b. metastases fron the gut
c. metastases from stomach
d. metastases from breast
61. Metastatic mass is not solitary in nature CT
62. Q9. Post partum lady irritable agitated what to help Dx
a. Hx of depression
b. Thoughts to harm her baby
c. Suicidal thoughts
63. Q10. Anorexia nervosa pt brought by parents concerned about her losing wt involved in lots
of exercise program, do not eat much , asking what important to make the Dx
a. walk 3 miles every day
b. sexual abuse
c. eating problem
64. Q11. Diabetic man on olanzapine taking to control his symptoms for last 2 years, notice
increase wt what to do
a. decrease olanzapine
b. change to other drug
c. stop it as he feel good now
65. Q13. Want to do study involve University students want to use vit C to decrease risk of flu
choose group give them vit C, other group do not give vit C, what make your test succeed
a. randomisatiom
b. random sampling ( randomized control trial )
c. strict way to follow up ,something like that
66. Randomized control sampling : 1)random sampling 2)non biased sampling ( ques comes in
tow pattern )
67. Q 14. 4 ys old child present with headache, ataxia, vomiting , O/E there is a mass in the
upper part of post triangle of the neck. He has also nystagmus I remember, asking what is
the Dx
a. medulloblastoma
b. meningioma
c. astrocytoma
68. Q15. What is the U/S benefit used as abdominal diagnostic measure
a. injury to solid organs
b. to Dx fluid filled cyst
c. To Dx ruptured viscus
69. Q16. 35ys old man brought by wife, presented with confusion, ataxia, right upper abdominal
pain, changed behavior, memory loss, weakness, pt drink alcohol and smoke some per day, T
37.5, vitals normal.
He has his father and paternal aunt has same history when at his age. Asked what
investigation to reach Dx
a. CT brain
b. CT abdomen
c. MRI brain
d. Check serum ceruloplasmin
70. Q17. Woman married to a man known to have hepatis C positive, she came to know what
increase risk of transmission of hep C the most to her
a. unprotected sex
b. use his tooth brush and his razor blades
c. eat together
71.
72. Q18. 32 weeks pregnant not immunized come want advice, her son 8 years old diagnosed
having chicken pox, options
a. give her IG
b. give her vaccine
c. check her IgG status
d. reassure her
e. ask her to come in a week for U/S
73. Q19. Patient came want advice about his cardiovascular disease risk you did exercise
challenge test was positive, PPV 55%, NPV 90%, what is his risk
a. 10%
b. 45%
c. 55%
d. 90%
e. 100%
74. #shumayla
75. Gbs stem.how to monitor respiration.
FEV
FVC
Monitor chest expansion
76. Post op incarcerated hernia 1st day aggitation stem fever sob O2 sat 88% xray given after o2
what initial step.xray had patches! What initial
IV antibiotics
Heparin
Thrombolysis
Droperidol
77. Chest pain 12 hrs pain reduced 4/10 on ecgst depression o2 given nxt
Morphine
GTN
Heparin
Thrombolysis
78. Wpwhr 280 nxt
Verapamil
Amiodarone
Cardiovert
79.
80. Central chest pain incresing on inspiration..has fever ecg given cudnt make it out exactly
what it was! Guess pericarditis
What next aft O2
Heparin
Nsaid
Cath lab
Thrombolysis
81. Old lady fever agitation admitted uti given trimethoprium.was taking carbamazapin
4wks..PTU since yrs n antihtnives.labs now shows Na 120 what next
Stop carbamazapine give hypertonic saline
Stop trimethoprium give N/S
Stop ptu give saline
Stop carbamazapine n restrict fluids
82. Temporal lobe epilepsy child n lady in shopping center both old n same stems.
83. scenrio of a schizophrenic patient walking naked in streets.which of the following is LEAST
effective for this patient.
A.olanzapine
b.quietiapine
c.amisulpride
d.clozapine
84. Encoparesis 7yrs girl fighting with brother wen he call him smelly
Normal developmnt
Delayed developmnt
ODD
Regression
Depression
85. Stem of copdxray was totaly not of copd :( options
Copd
Asthma
Heart failure
86. Pic of sle rash face .arthritis, mouth ulcers trmnt
Methotrexate
Hydroxychloroquine
Cyclosporin
87. Man with tirdness lethargy cough dyspnea
Labs shows
Hb 104
Tlc 40
Plt 90
Blast cells 60%
Diagnosis
Acute leukemia
Bone marrow infiltration by CA lung
CML
CLL
88. Psoriasis rash on whole legs since 6 monstrtmnt
Uv therap
Calcipotriol
Predsinolone
89. Postop cataract 5th day got up with pain n blurring of vision red eye pic
laterlyicudntapreciate exactly ant eye had pus or not :(
Hypopyon
Glaucoma
Scleritis
90. MR pt going focolonscopy what prophylaxis need
Diffrnt antibiotics in 4 options
No prophylaxis required
91. Pt allergic to penicilline staph aureus on culture
Ceftriaxone
Cefazoline
Cefalaxine
vancomycin ( vancomycin is completely safe for killing stap aureus and 3rd gen cefa
sometime cause allery )
92. Xray given plumber resp symptoms n signs mix.fev n fvc values given frgot what read the
topic.asked next
CT
Pleural biopsy
Trial of steroids
93. Few very poor quality xrays..i did xrays in my prep btcudnt diagnose those.

JO BOLE SO NIHAL, SAT SRI AKAL !

Gout picture. Treatment asked. Patient was already on Allopurinol and


intermittent colchicine and urate was below normal level
A)increase allopurinol
B) Naproxen
C)Prednisolone

42.24 years old aboriginal male with lack of interest in normal life and is
having visual hallucination watching his ancestors he is saying my
mother died last month but it is not the case and mother died year ago,
also has sucidal thought what is most likely diagnosis?
A. Personality Disorder
B. Depression with unresolved grief
C. Depression with psychosis
D. Schizophrenia
E. major Depression
What is the emergency case: hepatic injury, free fluid in the peritoneum,
intra vascular splenic injury
3. A girl diabetic saying that she found HbA1C was 5 to 8 at her home by
self checking. But at hospital you found HbA1c-11. What is the cause?
Pt compliance not good
13.Pt admitted to hospdue to gentourianry infection not sure though!
History of rash topenicillin. staph aureus on culture which is the antibiotic
of choice for her
A.Ciprofloxacin
B.Cefazoline
C.Cefalaxine
D.vancomycin
E.ceftriaxone
Obese lady with dizziness after gym where she goes to lose weight.
Proximal muscle weekness. Truncal obesity HR-35 .- next - atropin
/TFT[bradicardya treat first-confuse]

Case of intramedullary nail after tibia # has pain with passive extension
of big toe what is most apropiate Mx.
*Analgesic
*Tight bandages
*Review afte 1 wk
*Check again after 2 hour > still i think of compartment i choose this

82 year old husband stabbed wife by accident. On examination not


coperative for MMSE.
Asking Dx.
1. Dementia
2. Delirium [confuse]
3. Alcoholic influence

32. a male with h/o orchiectomy came with infertility complain . semen
exam shows 1 million sperm count, what next investigation
a. karyotyping
b. FSH LH
c. Y – chromosome deletion test
d. testosterone[confuse between b,d.bcz jm say b,but here orchidectomy
so 1st testosterone]

1. A 19 yr girl having 3 sexual partner came for HPV vaccine. What to


do? Give her, check HPV DNA, Do PAP

1.hutchinson image freckle - treatment


2. case of malanoma, man asking how to protect his 3 and 5 years old -
apply sunscreen when going in the sun ?
3. mitral stenosis - aboriginal woman recall
4. Handbook question - cerebral infarction with ct
5. Grandmother wanting to protect new born – GIVE dTPa
6. TETANUS - 3 QUESTIONS: AFTER INJURY - 5 year old boy,
7. ecg - hyperkalemia - asking how to treat - stop ramipril
8. A man with peptic ulcer who is taking triple therapy (PPI, amoxicillin,
metronidazole) for weeks but urease breath test was positive for H.pylori
a. what is the reason?
b. a. resistance to metronidazole
c. b. resistance to amoxicillin
d. c.unreliable urea breath test
9 10 cm FRIABLE MUCOSA , what is ur INITIAL Management ?
1-injection CS
2-rectal CS
3-sulfaslazine
4-loperamied
5-methotrexate
10. new born - meconium aspiration scenario - do oxygen and bag and
mask
11. 8. A 15year old girl having difficulty in concentrating. She constantly fights
with father, need support in school because she can’t maintain tasks, use
multiple drugs, like amphetamine, marijuana, telling it’s difficult for her to
sleep, denied any hallucination or suicidal thought, what will you do?
Liase with father
Discuss the safety plan with parents
Give ssri
CBT
Tell school to support her more
9. . an old women who is widow who seems well in past but since she became a
widow and she moves to the house and start to stay alone one of neighbour had
noticed her strange behaviour. she starts to dig some ground in front of the
house and when neighbour reached near her and looked at her, she then aroused
him and get aggressive and accused him and all neighbors as imposters. After
that she became calm down and agreed to be seen at the medical clinic.Which
one of the following explains her condition?
A)thought forms
B) mood
C)orientation
D)delusion
E)memory
mother Hep C positive. What to avoid during pregnancy to avoid transmission
in infant?
a) breastfeeding
b) vaginal delivery
c) Antivirals in pregnancy
d) Fetal scalp sample
child got superficial abrasions and lacerations after falling in a garden bed. he
has h/o 2 DTPa
vaccinations. most app step after cleaning the wound?
a. give Tetanus toxoid and topical antibiotic cream
b. give tetanus toxoid and oral penicillin
c. give tetanus toxoid and immunoglobulins
d. DTPa and booster after 2 months
e. DTPa and immunoglobulins
Post hysterectomy patient with history of DVT post operative after
hysterectomy. Now comes with hot flushes. Which HRT is best?
Low dose Oral estradiol
Oral estrogen plus Progesterone
Oral Progesterone alone
Transdermal estrogen patch
transdermal estrogen and progesterone

35 year old lady comes to you regarding screening for breast cancerscreening. She
says her paternal aunt was diagnosed with breast and ovarian cancer at the age of
60 years which was diagnosed as being associated with BRCA1.Which of the
following is the most appropriate advice?
A-Genetic counselling assessment
B-refer for BRCA 1 screening
C-screen now
D-tell her to look at her breast
E-annual mammography
An old Australian male, came with the following complaint as in the picture
below, what is the most appropriate action?
a. local excision
b. Excision with 2cm margin
c. Topical podophyllin
d. Topical imiquimod

51 years old woman had her last menstruation 8 months back. She is complaining
of host flushes and insomnia. Which preparation is best for her?
a) Continuous HRT with oestrogen and 12 days progesterone(continuous
sequential)
b) Continuous HRT with oestrogen and progesterone
c) OCP
d) Oestrogen patches

7 days old infant presented to you complained with poor feeing and bile stain
vomiting. His birth was at 38 weeks gestation and weight 2600g. He is
breastfeeding with no immediate post natal complications. The child has a history
of passing meconium on day 4 of birth. Now, he had mild jaundice and abdominal
distension present. What is the appropriate diagnosis?
a) Hirschsprung's Disease
b) Meconium Ileus
c) Duodenal Atresia
d) Necrotizing Enterocolitis
e) Volvulus

60 year old man on DRE prostate is normal, first psa was 1.5 then PSA is 3.2
(n<4.0) On USG-guided biopsy 1 locus pathology showed adenocarcinoma
of Gleason 4 stage. What is the management?

a) radical prostatectomy

b) continue surveillance (<60yrs)(PSA<20,gleason <6,stage 1-2)

c. transurethral prostatectomy
d. EBRT
. Child with asthma,during blood test ketones and high sugar found what test for
follow up?

a) HBA1c

b) FBS

c)OGTT
d)2hr OGTT
e)random blood sugar
Orbital floor fracture, what will you find?

a) anasthesia of the cheek


b) instable TMJ
c) subconjunctival hemorrhage
d) loss of visual acquity
The same scenario of a woman with and her son is a drug addict, he hits his mother
something and she presents to you, she fears the danger of her life, what is the
safety of your next management?
a) Provide woman with a refugee shelter
b) Obliged her to inform policy
c) Tell her to go to the police
d) Join a support group
e) hospatalize her son
35yr old women has relationship instablity, insomnia, agitated, many suicidal
actions and can’t control her anger. What treatment??
a.Dialectal therapy
b.CBT
c.personal therapy
d.promblem solving
e.antidepressant
A case of gynaecomastia pt on amisulpride.
What other drug to counteract the side effects
A. Quetiapine
B. aripipazole
C. olanzapine
D. Clozapine
E. Mirtazipine
Ans : A/B
35 year old pregnant woman presented at 15 weeks gestation asking you for a
diagnostic screening of down, what is the diagnostic screening test you should
do for this woman?
A-nuchal translucency ultrasound
B-maternal serum and ultrasound
C-maternal serum
D-amniocentesis
E-chorionic villous sampling
Patient on post operative day 3 started becoming agitated, irritable and had
Shortness of breath.
What is the next and best appropriate investigation?
a. CTPA
b. D-Dimer
c. Chest x-ray
D. Blood gases ( may be due to PE so we should do ABGA>cxray>CTPA)
E. Blood sugar level
A mother came with her child complaining of continuous bleeding after fall
from coffee table.
On exam there is MINIMAL bleeding from the frenulum and child had some
old bruises on forehead and legs. there was no petechiae, lymphadenopathy and
hepatomegaly. What is the most likely cause
a. Idiopathic Thrombocytopenic Purpura
b. non accidental injury
c. VWD
d. clotting defect

A patient who had a normal pap smear 6 months ago was advised to do a
screening test for chlamydia . She wanted to know why or if the test is
necessary ?
A. Chlamydia causes infertility
B. Chlamydia is asymptomatic
C. Reassure her the test is not important because the pap smear was negative.

Poorly controlled Diabetic with mild renal impairment asking about risk for
baby due to diabetes
A. IUGR
B. Macrosomia
C. Renal agenesis
D. Intrauterine fetal demise
E. low Birth weight
A patient was sent on a stress leave after employer finds him using cocaine. The
patient admits ,using cocaine and also has memory impairment, agitation and is
also irritable, what is the next step
a. urine drug screen
b. collateral history from employer
c. history of sexual abuse
d. past history of drug use
24yrs old with dysmenorrhoea and menorrhagia asked to do investigation but
she insist she wants medication
a. mefenamic acid
b. tranexamic acid
c. ocp
d. iucd
Another question mention only menorrhagia?? Tranexamic acid
Heart failure case man with AF on Ramipril, metoprolol, digoxin. Went on trip
for 2 weeks and stopped his medication. Now with bilateral oedema up to knee
in legs. And ankle swellings Lungs is clear. And the patient is not dyspneic,
What is the most appropriate management?
A- Frusemide
B- Recommence all his medications
C- Ramipril
D- Digoxin
. Farmer with left inguinal swelling biopsy noted squamous cell ca asking which
region is primary
A)colon
B)rectum
C)left leg
D)anus
a pt has alzhemiers dementia and ca colon. U have planned colostomy. One of
her daughter asked as her mother is not capable of caring and she don’t want her
mother to be operate.her MMSE is 20/30. You talk to pt and pt has given
consent for operation. What is your most appropriate advice?
a) you will proceed for operate as pt has given consent
b) you will not operate as pt is not capable of giving consent
c) discuss with family physician and surgeon regarding management of pt.
d) as her daughter has power of attorney she can give consent for not operating
her mother

decrease breast milk production recall Asking common cause of decreased milk
production in breastfeeding
a.Less frequent feeding.
b.Dec intake of fluids
c.Spend less time while baby is sucking
old female pt, obese with mild rectocele, no cystocele, c/o stress incontinence,
improve with pelvic floor exercises but she feel socially embarrass. Next step in
her management?
a) weighted vaginal cones
b) bladder suspension surgery
c)reduce weight

Osteoporosis - i got 4 questions - one with esophagitis option - asking treatment


, >zolendronic acid
with breast cancer >raloxifen
, with vertebrae -2.7>alandronate
, and one in which she is menopausal and asking treatment - her VIT D Levels
are normal, but -2.5 in bones ,

Mamu recall , aboriginal child - ear ciprofloxacin recall,


menorrghagia - she wants treatment urgent - doesn’t want any tests - give
mfenamic acid.
I got 3 hernia cases - asking advice, treatment and diagnosis. i opted for
watchful observation - as patient was asymptomatic and there was no
underlying disease.
36 yrs old female migrates to Australia after living 6 yrs away from husband. In
her country she was abducted and raped by the militia. Now inv shows her to be
syphilis positive but her husband RPR was negative 1 month back. What to do
to husband?

a. treat husband with ceftriaxone

b. do nothing

c. repeat RPR after 3 months

d. do serial RPR for 3 months

a child presents with increasing fever and cough of 2 days duration. X-ray was
given which showed rt sided pleural effusion. Which investation aids in the
definitive diagnosis of the causative organism

a. blood culture

b. chest x-ray

c. pleural aspirate

d. pneumococcal pcr

old man , masturbates infront of Nurses - progressive alzheimer’s


Epigastric hernia picture of a 56 year old man which extends from
umbilicus to xiphisternum. Helooked morbidly obese
and had a waist circumference of 110 cm

Asking for appropriate advice for the patient.


A. Weight loss therapy
B. Abdominal binder
C. herniorraphy with mesh repair
D Hernioplasty
E Observation
Another question is same but ask for treatment??
Picture showing dupuytren contracture, Farmer with inability to extend
his left ring finger, history of cut injury in that fingers 3 months ago. He
drinks 4 cans of beers per day and 30 pack of smoking per year. What is
the cause of his injury?
A.chronic infection
B.His farmer occupation
C .Alcohol
D.familial history
E. Genetic

post operative pt, oliguric wz indwelling catheter, fever 38.5, what is ur most
appropriate management ?
1.abd x ray
2. blood culture
3.serum electrolytes and Cr
4-abd us
another question post op pt, no catheter, present with oligouria, ask
treatment?
young man presented with loin pain , urine examination showed
hematuria . what is the most appropriate next step?
A-X-ray
B-abdominal ultrasound
C-CT abdomen-best
D-MRI
E-urine culture
24yr boy come with penile discharge and come to your clinic. He has recent
history of sex with a new female partner who is asymptomatic. What to do??
a.first urine catch up PCR for chalmydia
b.mid stream urine PCR for chlamydia
c.urine culture

scenario od CTG 2mins and 4 mins


A man comes with clumsiness of hands and tripping over. Upper limb
weakness and fasciculation. Lower limb muscles also has weakness, increased
tendon reflexes. Ankle reflex is not increased in one side. No wasting. No
sensory loss is given. What is the initial investigation?
A. EMG
B. MRI Spine
C. CT scan
D. CSF examination
E. Cervical spine X-ray
An old man with acute onset pain in the lower back with some urinary
symptoms. With significant weight loss. Anal sphincter tone was normal. On
DRE enlarged irregular prostate. Dullness on palpation 2 finger breaths above
symphysis pubis. Slightly elevated temperature. What is next?
A. Chest X-ray
B. CT spine
C. TRUS
D. Bladder scan
E. PSA
A young girl who is 9 years of age with history of recurrent URTI last time was
two weeks ago and did urinalysis at that time showed blood (3+), protienuria
(1+). Now she came after two week. Urinalysis is unchanged except for Blood
(2+) which is non-glomerular and there are no casts in the urine. Which of the
following is the next best investigation?
a. Urine culture
b. DMSA
c. Ultrasound
d. IVP
e. ASOT
12 year old girl at foster care. she's having difficult time in school in reading n
writing . she has a collection of her own toys and plays with them with her
imaginary friend. her foster parents/ carers are irritated because she's picky at
food. what immediate danger to her in future ?�
a. OCD
b. sexual abuse�
c. drug abuse�
d. Anorexia nervosa�
e. schizophreniform psychosis - this is the answer!!! already discussed with
psyc doct

Mother living in mining town comes with her daughter for normal blood test.
Her daughter’s
test show lead level 0.72 (‹0.32)
what will be your next appropriate step?
a.Assess IQ testing of child
b.educate mother about environmental risks
C chelation therapy
I opted for chelation therapy -as in the brackets in the exam the normal was
(0.4)


7 days old baby presented with jaundice since 4 days of life, his birth was at
term , not complicated and he is breast feeding , serum bilirubin level was
240 (normal <200) with 120 direct bilirubin (it’s 120 not 12 confirmed), his
liver is 1 cm palpable below costal margin. What is the most appropriate
next step?
A-thyroid function test
B-abdominal ultrasound for liver and biliary tree
C-coombs test
D-stop breast-feeding and follow up
E-phototherapy

Primigravida gave birth at term to 3200 g baby, normal vaginal delivery


without any complications and there was slight meconium staining of
liquor. Baby was normal at birth with normal heart rate. After 1 minute,
suddenly stopped breathing.HR decreased. Cyanosed. No response on
stimulation. What is the most appropriate next step?
Intubate
Nasopharyngeal aspiration of meconium
Bag and mask ventilation
Indigenous child with chronic suppurative otitis media presents with
chronic ear discharge. ear swab done showed existence of pseudomonas
aeruginosa organisms.
After ear toileting what will u do
a. oral amoxicillin
b. oral augmentin
c. ciprofloxacin ear drops
d. framycetin with steroid ear drops

A patient with a painless neck swelling moves with swallowing. Now


hoarseness and difficulty in breathing at night. increased over 3 weeks. O/E
smooth swelling in the anterior triangle of neck. dullness of percussion at
upper sternal border. most likely dx? (not mention about hyperthyroid sign
and symptoms)
a. Anaplastic cancer
b. papillary carcinoma
c. MN Goiter
d. hemorrhage in nodule of MND
e. follicular CA
Patient who became agitated and irritable after sustaining head injury. Na+
=123 K+=4.8. What is the reason for his symptoms?
a) SIADH
b) physiological response to injury
c) acute renal failure
i opted for SIADH!
Woman 25 years old is going to travel to India presented asking advice about
vaccination. She has previous 1 dose of polio vaccine , had DPT at birth, 2 and 4
months age , 1 dose of MMR at 13 month ,
and tetanus booster at 15 year age. What
vaccines should she received before travel?
A-MMR, diphtheria and tetanus
B-polio, diphtheria and tetanus
C-polio, MMR, diphtheria and tetanus
D-polio, MMR
E-MMR
Elderly lady with osteoporosis and some
Gastric issues with normal vitamin D asking
management?
A. Alendronate
B. VITAMIN D and Calcium
C. Strontium.
D. HRT

Mother bringing baby 5 times- A lady presented with her baby 5 times in 2
weeks at 10 weeks following delivery - all normal, established breastfeeding,
baby growing well on 90th percentile......What relevant info will you ask the
mother?

A. Past bad obstetric history
B. premorbid personality
C. history of psychosis
D. Thoughts of harming her baby
E. How to think about of her child’s behavior - i went for this option

An old woman taken many tablets, now unconscious, pinpoint pupil, his son
brought her to hospital, and also tablets she might have taken, methadone,
oxycodone, buspirone patch
A-methadone(ans)
B-oxycodone
C-buspirone
D- alcoholpt

presented with fever. 38.with mild ascites, gynecomastia , spider nevi, flapping
tremors. vitals given. whats ur initial step of management of this patient ?
a. full blood examination
b. LFTs (hepatic encephalopathy)
c. ct scan
d. abdominal USG
e. Abdominal paracentesis

A 55-year-old woman presented with discomfort in her both legs with an urge
to move for last 4 months.She remains awake at night due to creeping and
sometimes electric current-like sensations.Her current medication include
paracetamol, oxycodone and multivitamins.Blood tests show normal renal
function, normal electrolytes and slightly low haemoglobin.
What will you do next?
a. Cease oxycodone
b. Fluoxetine
c. Iron studies
d. Mirtazapine
e. MorphineCouple coming for infertility.

Wat to ask husband-
.couple comes for infertility evaluation.female already has a kid.on evaluating
the male for infertility,what is the relevant question .
a.how much alcohol do u consume
b.chicken pox history.
c.marijuana use
d.ecstasy and other drug use

Couple inferitlity. woman was ok , Male with absent vas def. -


Couple come for infertility problem for the last 12 months. On testing
examination and tests of the female are unremarkable. Male has azospermia.
And bilateral absence of vas deferens. Which of the following is most
appropriate before the starting the treatment of infertility?
a. No testing required as they can not have a child
b. Testing of both male and female for cystic fibrosis
c. send for IVF
D.serum fsh and lh for male - i went for this option
E. serum testosterone level

brother hemachromatosis- check only mother not kids
A mother comes with hemochromatosis in brother. she has
2 kids aged 2 and 5 yrs respectively. Wants advice regarding
screening
A. screen only mother
B. mother n kids
C. ask her to come with husband
D. Screen only children
Pt with RA she takes ibuprofen and methotrexate to control her disease ,they
mention the time it was years for both of drugs , pt complains of some
symptoms , and her labs are given ALT ,AST GGT all were high , the Q is asking
about the cause of this ?
A- Methotrexate induced hepatitis
B- Ibuprofen induced hepatitis
C- Autoimmune hepatitis
D Viral hepatitis

weeks pregnant lady came with labour pain..she was put in left lateral position
having oxygen mask in place.iv fluids with syntocinon is running. Ctg was done
which shows heart rate of 140 which dropped to 70 and came back to 140 in 4
min. asking next appropriate treatment.
A. Fetal scalp sampling
B. Continuuous ctg monitoring
C. Stop syntocinon
D. C section
year old male with a history of fall on an outstretched hand with pain and
swelling. XRay was given, ( showed scaphoid fracture. Very clear
fracture line throught the middle)
What is the best treatment option for this patient
a. Crepe bandage
b. Plaster cast
c. Analgesics
d. Compression screw
e. Plate fixation
30yrs old indigenous woman, with history of Mitral Stenosis. Shortness of
breath for 6weeks. Low grade fever, cough. CXR given (see below; looks like
this one) on examination, bilateral basal crept present in both base of lung.
What is the diagnosis?
A. Pulmonary hypertension
B. Pulmonary fibrosis
C. TB
D. Left heart failure
E. Lymphoma
Young aboriginal male presents to you with insomnia, fear of darkness and
seeing “mamu” . He has been having these symptoms after the death of his
mother. Which of the following should be next step in his treatment?‫ ؟؟؟؟؟‬
A-give him benzodiazepine????
B-involvement of aboriginal health worker
C-urine drug screen
D-antipsychotic
E-CBT
89Pt with active RA on methotrexate with chronic pyelonephritis ,feels tired
and lethargic labs done showed the ff

Hb 9.5 g/dl
Mch low normal
Serum iron decreased
S.ferritin normal
Tibc decreased
What is treatment of choice
A. packed RBCs
B. iv iron
C. low dose corticosteroid
D. erythropoietin injection
Old man. Afib. Abdominal pain. Wat other test with ct- serum lactate
A scenario of an old man with history of atrial fibrillation and now comes with
severe abdominal pain. What other than CT angiography will you do?
a) CT abdomen
b) Serum lipase
c) Serum lactate
d)ERCP
CT given - haemopneumothorax. Painful shallow breathing. O2 given by face
mask, what's next?

1. Morphine
2. Intubate
3. Drain
A 74 year old man is being screened for prostate Ca. His PSA 2 years ago was
1.5ng/ml; now PSA is 3.8ng/ml. Only %5 of biopsy done showed
adenocarcinoma, Gleason score is 4. How will you manage this patient?
a. TURP
b. Radical prostatectomy
c. External beam radiation therapy
d. Androgen therapy
e. Active surveillance
On interview. Asking about the appetite. Patient told “That is concern with
food. Food which does not comes from moon, moon made of cheese. But I
came her by bus”. Which of the following will have in this patient?
A. Depression
B. Dissociation
C. Disorganized behavior
D. Delusion not ans
E. Depersonalization

A 40yrs old woman on cocp for 8yeaes. She is a smoker and her BP is high. She
doesn't want to get off pills. What is your advise?
A. Stop pills and rely on condom
B. Copper IUD
C. Implanon
D. POP

Lady comes on 10th postpartum day with pain due to perineal tear.Now on
examination there is perineal wound that is 2 cm long,1cm wide and 1 mm
deep.It was clean just serous discharge was there...
What to do next?
Apply local antibiotics
Put on oral antibiotics
Simply keep the wound clean
Suture the wound under local anaesthesia
Suture the wound after cutting the edges

worsening symptoms at night Alzheimer

History of appendicetomy & cholecystectomy presented with 3 wk history of
abd pain, distension. X-ray shows dilated ascending colon, transverse colon &
descending colon, no air. Rectum is empty.
A. Sigmoid volvulous
B. B- Adhesive IO
C. C- CA Sigmoid

17 girl has dyspareunia, worry about endometriosis because sister has
infertility because of this. Examination: nodule on ligament on vaginal
palpation and some other description, what is important to diagnose
endometriosis:
D. nodule on uterosacral ligament,
E. dyspareunia,
F. menorrhagia,
G. family history

girl with type 1Dm ,on intermediate insulin at night, short acting insulin in the
morning, her parents concerned about her high glucose levels of 15 at
morning, what next? �
A)increase intermediate insulin at night
B)increase short acting insulin in the morning
C)prevent fruit in evening �
D)measure glucose levels at 2am and 3am

A 9 month old child came with recurrent upper respiratory infection. The
immunization history reveals that the child has received his hepatitis B
vaccination at his birth and no further immunization history at 2 and 4
month of age due to recurrent upper respiratory infection. Now the child
has fever 37.8 degree Celsius and clear nasal discharge. What is your
management regarding immunization?
1. Give hepatitis B immunization and institute catch up schedule
2. Give hepatitis B immunization and recommence standard schedule
Others option irrelevant
39 weeks pregnant lady came with labour pain. Synto was given
appropriately. CTG was done which shows heart rate of 140 which
dropped to 70 and came back to 140 in 2min. asking next appropriate
treatment.
A. Cease syntocinon
B. Fetal scalp sampling
C. C section
D. Give IV dextrose, normal saline (not like dextrose)
E. Give O2 to mother via face mask

B. Man comes to ED with severe chest pain which occur after a bout
of vomiting, on examination, there is dullness over left lower lobe
of lung, what is investigation?

a. CT chest

b. Gastrografin swallow

c. Oesophagoscopy

d. CXR

e. USG

24 months old, developmental task?


A. 2 or more personal pronouns
B. Sentence with 4 syllable

C. Name 4 color

D. Know age

E. Know family name

Pt with smaller kidney on one side, what to do to check individual


kidney's function?
A. DMSA

B. DTPA

C. IVU

D. USG

E. X ray Woman on HRT for 6 years, menopausal symptoms are well


controlled, bone density score for spine vertebra is -1.7 and for
femoral head is - 1.2.What is next? Vit D and calcium level are
given Normal level.(<-1.5àosteoporosis)

A) Cease HRT
B) Continue same
C) Change HRT to Alendronate
D) Add Alendronate
E) Vitamin D and Calcium supplement
Pt with claudication pain. Doc advises sx(by pass , stent), but pt
not fit to do surgery. Femoral bruits+. He has DM, HT and
obese. What will you advice for long termmanagement to
reduce claudication symtoms?(OHCM 565)
a. supervise exercise
b. reduce smoking (not stop smoking)
c. HT control
d. DM control

An 18 year old girl presents with epistaxis, malaise and tiredness for
months. no other symptoms. Hb is 8.5, serum ferritin is 35, INR is 1.5.
Calcium is 1.9(n-0.9to 1.2). What is next to do?
a. Blood transfusion
b. Iv calcium
c. Inj vitamin k
d. fresh frozen plasma
e. Vitamin D tabs
8 yrs old recently started having movements of his arm and head turned
towards one side. Eyes fixed, stays for 30 to 60 seconds. Many times a
day for 2 weeks and then nothing in other week.Could it be?
a) Temporal lobe epilepsy
b)juvenile myoclonic epilepsy absence + partial seizures h
c) Absence seizures

2.. mother came with child complain of episode of staring suddenly with
movement of hand. head twitching lip smacking for 60-90 sec. then
become confused for few min. dx?
Temporal lobe epilepsy
Juvenile myoclonic epilepsy
Abs seizure

same scenario as above - THEY WERE ASKING TREATMENT - no


option for carbimazole , only sodium valproate.
• Child from daycare with post streptococcal Glomerulonephritis. When can she return to
school?

a. Immediately after discharge
b. After throat swab is done for all students
c. After prophylaxis for all students
d. After microscopic hematuria resolves
b.Low protein diet and minimal activity


Disease or condition Exclusion of case Exclusion of contacts

Amoebiasis (Entamoeba Exclude until no diarrhoea for 24


Not excluded
histolytica) hours.
Exclude until no diarrhoea for 24
Campylobacterinfection Not excluded
hours.

Candidiasis See thrush.

Any child with an immune deficiency


(for example, leukaemia) or receiving
Exclude until all blisters have dried
Chickenpox chemotherapy should be excluded for
(usually 5 days)
their own protection. Otherwise not
excluded.

Cytomegalovirus
Exclusion is not necessary. Not excluded
infection (CMV)

Young children and others unable to


Cold sores (herpes comply with good hygiene practices
simplex type 1, fever should be excluded while lesion is Not excluded
blisters) weeping. Lesions should be covered
by a dressing where possible.

Exclusion is not necessary, but a


Common cold person with a cold should stay home Not excluded
until he or she feels well.

Exclude until discharge from eyes has


stopped (unless a doctor has
Conjunctivitis Not excluded
diagnosed non-infectious

conjunctivitis).

Exclude until no diarrhoea for 24


Cryptosporidiuminfection Not excluded
hours.

Diarrhoea (no organism Exclude until no diarrhoea for 24


Not excluded
identified) hours.

Exclude until medical certificate of


recovery from SA
Health's Communicable Disease
Exclude contacts living in same house
Control Branch is received following
until cleared by SA
Diphtheria at least two negative throat swabs, the
Health's Communicable Disease

first not less than 24 hours after


Control Branch
finishing a course of antibiotics and
the second, at least the next 48 hours
later.

Exclude until well – no vomiting or


Food poisoning Not excluded
diarrhoea for 24 hours.

Exclude until no diarrhoea for 24


Giardia infection Not excluded
hours.

Glandular
fever(infectious
mononucleosis, EBV or Exclusion is not necessary. Not excluded
Epstein-Barr virus
infection)
Hand, foot and mouth
Exclude until all blisters are dry. Not excluded
disease

Haemophilus Exclude until person has received


influenzae type b (Hib) appropriate antibiotic treatment for at Not excluded
infection least 4 days.

Exclude until appropriate treatment


Head lice (pediculosis) Not excluded
has commenced.

Exclude until medical certificate of


recovery is received, and until at least
Hepatitis A Not excluded
7 days after onset of jaundice or

illness.

Hepatitis B Exclusion is not necessary. Not excluded

Hepatitis C Exclusion is not necessary. Not excluded

Hepatitis D Exclusion is not necessary. Not excluded

Exclude for 2 weeks after illness onset,


or if jaundice (yellow eyes or skin)
Hepatitis E Not excluded
occurs, for 7 days after the onset of

jaundice.

Herpes simplex
See cold sores
infection, type 1

Exclusion is not necessary. If the


Human person is severely immune
immunodeficiency virus suppressed he or she will be Not excluded
(HIV) infection vulnerable to other people’s

infections.

Hydatid disease Exclusion is not necessary. Not excluded

Impetigo See school sores.

Influenza and influenza-


Exclude until well Not excluded
like illnesses

Legionnaires’ disease Exclusion is not necessary. Not excluded

Exclude until clearance has been given


by an infectious diseases physician,
Leprosy dermatologist or SA Not excluded
Health Communicable Disease Control
Branch doctor.

Immunised and immune


contacts are not excluded. Non-
immunised contacts are excluded
Exclude until 4 days after the onset of
Measles from childcare until 14 days after rash
the rash.

onset in the last case in the facility,


unless given vaccine (within 72 hours
of first exposure) or Normal Human
Immunoglobulin (within 144 hours of
first exposure). All immune
suppressed children should be
excluded until 14 days after rash onset
in the last case in the facility.

Exclude until well and has received


Meningitis (bacterial) Not excluded
appropriate antibiotics.

Meningitis (viral) Exclude until well. Not excluded

Exclude until appropriate antibiotic


Meningococcal infection Not excluded
treatment completed and until well.

Molluscum contagiosum Exclusion is not necessary. Not excluded

Methicillin Exclusion is not necessary unless


resistant Staphylococcus infected skin lesions on exposed
Not excluded
aureus (MRSA) skin surfaces cannot be completely covered
infection with a dressing.

Exclude for 5 days after onset of


Mumps Not excluded
swelling.

Exclude until no vomiting or diarrhoea


Norovirus Not excluded
for 24 hours.

Parvovirus infection Not excluded, but people who are


(fifth disease, erythema anaemic, immune suppressed, or
Exclusion is not necessary.
infectiosum, slapped pregnant should be informed of
cheek syndrome) possible risk of getting infection.

Pertussis See whooping cough. Not excluded

Respiratory syncytial
Exclusion is not necessary. Not excluded
virus (RSV) infection

Exclude until the day after appropriate


Ringworm/tinea Not excluded
treatment has commenced.

Ross River virus Exclusion is not necessary. Not excluded

Exclude until no vomiting or diarrhoea


Rotavirus infection Not excluded
for 24 hours.

Rubella (German Exclude until fully recovered or for at


Not excluded
measles) least 4 days after the onset of the rash.

Exclude until no diarrhoea for 24


Salmonellainfection Not excluded
hours.

Exclude until the day after appropriate


Scabies Not excluded
treatment has commenced.

Scarlet fever See streptococcal sore throat.

School sores (impetigo) Exclude until appropriate treatment Not excluded


has commenced. Any sores on exposed
surfaces should be completely covered
with a dressing.

Exclude until no diarrhoea for 24


Shigella infection Not excluded
hours.

Exclude until blisters have dried up


unless rash can be covered with a dry
Shingles Not excluded
dressing or clothing so others are not
exposed.

Exclude until the person has received


Streptococcal sore throat
appropriate antibiotic treatment for at Not excluded
(including scarlet fever)
least 24 hours and feels well.

Thrush (candidiasis) Exclusion is not necessary. Not excluded

Toxoplasmosis Exclusion is not necessary. Not excluded

Exclude until medical certificate is


Tuberculosis produced from SA Tuberculosis Not excluded
Service of SA Health.

Exclude until clearance has been given Not excluded unless considered
by a doctor or SA necessary by SA
Typhoid, paratyphoid
Health's Communicable Disease Health's Communicable Disease

Control Branch. Control Branch.

Varicella-zoster See chickenpox or shingles

Viral gastroenteritis Exclude until no diarrhoea for 24


Not excluded
(viral diarrhoea) hours.

Warts (common, flat and


Exclusion is not necessary. Not excluded
plantar)

For exclusion of contacts in a childcare


setting, seek advice from SA
Health's Communicable Disease
Control Branch. Usually, childcare
contacts (in the same childcare group
Exclude from childcare, school or or room) and household contacts of
workplace and similar settings until 5 the case who are under 6 months of
Whooping cough days after starting antibiotic age and have received less than 3
treatment, or for 21 days from the doses of pertussis containing vaccine
start of any cough. should be excluded from childcare for
14 days from the first exposure to the
infectious case, unless he or she has
completed 5 days of recommended
antibiotic treatment, after which he or
she may return to childcare.

If diarrhoea present, exclude until no


Worms Not excluded
diarrhoea for 24 hours.

Yersinia infection Exclude until no diarrhoea for 24 Not excluded


hours

• pt comes. with history of headache (no word of mild or severe just headache) with
retrorbital pain and ptosis. He has history of previous recurrent headaches also. Now
gives history of neck pain but NO STIFFNESS. neurological exam is normal...what 2 do
A) Do urgenf CT
B) give him trial of 100 % Oxygen


• Woman on HRT for 6 years, menopausal symptoms are well controlled, bone density score
for spine vertebra is -1.7 and for femoral head is - 1.2. What is next? Vit D and calcium level
are given Normal level.

• Cease HRT

• Continue same

• Change HRT to Alendronate

• Add Alendronate

Vitamin D and Calcium supplement

Patient on amiodarone and many drugs. Developed maculopapular skin rash on hands and face after
2-3 days of started on amiodarone. The rash was more prominent when he went out in the sun.
Asking cause. JM
• Fixed drug eruption
• pseudoporphyrin
c.phototoxicity(exact words)
d.pemphigus


A man complaint of ‘feeling something in his body ‘and anxiety. BP160/- .No
other features of hyperthyroidism given. Ox 220

Give beta blockers==initial
Investigate for pheochromocytoma==if ask diagnostic investigation??
Investigate for hyperthyroidism
Arrange psychiatric counselling

A man with pain on the right side of the abdomen which radiates to the groin which is colicky in
nature. He also had haematuria in his urine. What will you do next?
Right abdominal USG
Plain X ray abdomen
CT urogram

Middle age man, pain in buttock and thigh during 100 m walk on ground, can walk 20 m
uphill, femoral pulse not palpable, dorsalis pedis is palpable, ABI 0.3. Best way to diagnose?
A. Arterial Doppler
B. Digital subtraction arteriography
c. ct angiogram (for intervention)
d. arteriography


• 58 year old woman after lifting some heavy thing, develops pain in the right buttock and
tenderness in her back at L4-5, has difficulty and restrictions of movement in extension and
flexion and rotations , what will you give beside analgesic?
C- bed rest
D- referral to orthopedics
E- keep active
No option for spinal Xray

76.. Mom has fever , cough. Her child had fever,rash,sore throat last week. Her CSF examination
reveals Glucose 3.5 mmol/L and ??? clear fluid ( i think it is bcus of viral )

A. Enterovirus
B. Influenza
C. HSV
D. Meningococcus
E. Pneumococcus ???

81.NSTEMI ECG??? pt has chest pain now relieve 4/10 he is taking ramipril atovastatin aspirin
now no murmur, no dyspnoea , all normal
A . Morphine
B. Clopidogrel
C. Thrombolysis

97.Patient was just shifted from ICU after paemaker insertion. patient suddenly complains of chest
pain and difficult breathing. Pulse 88 Beats/min Oxygen 98% heart sound faint. Next?
A.CXR
B.CTPA
C.CT
D.Troponin
E.Echo
Lead-related complications

Early lead-related complications include the following:


Atrial or ventricular arrhythmias
Chamber perforation [13]
Pneumothorax and pneumopericardium (with an atrial lead)
Intercostal or diaphragm pacing
Pectoral muscle stimulation
Lead dislodgement
Tricuspid valve laceration
Cardiac tamponade
Pericardial friction rub
Hypotension
Bleeding






111..One Xray with Left side has no diaphragm outline like paraoesophageal hernia
p/t has abd pain no guarding, nausea vomiting, only GI symptom

on auscultation , Chest is clear on both side . no Resp symptom


p/t post of 3days I think

• CTPA

• CXR

• Abd USG

• Stool for ova and cysts???

39. A man goes to ER after having dinner, wife said he ate seafood, has swelling of the
tongue. He takes Ramipril and indapamide for BP, metformin for DM and statin for high
cholesterol . What caused this? A) C1 def. B) Ramipril c) metformin d) statin.


39yrs old lady with blood discharge from nipple . There’s a 8mm lump too in the Areola. Dx?
Introductal ca
Invasive ca
Duct papilloma


Lady of zuclophenthixol depot for psych issues bcz she’s non compliant with oral meds. Tried
olzapine n resperidone previously but non compliant. Now with ridigidty n tremor what will u do?
Switch to quietiapine
Resperidal consta

Resperidone
Clozapine

59.african lady had curcimsition now come for antenatal checkup.what to do?

Normal antenatal checkup

Vaginal dilator everyday

Reverse circumsition

60.worried mother come with children to have antibiotic for meningitis.as boy visited child care
where a girl dx as meningitis from mon –Wednesday. but that child visit just on Friday. what to do?
fever of child not mentioned.

Inform her about sign symptom of disease

Send her to imergency department


Give antibiotic

A baby delivered by normal vaginal delivery, is well after birth. On the 4th day, the baby is
found collapsed in the cot, breathless and floppy. On examination there are no murmurs.
Possible cause could be--
a) TOF DURING ATTACK NO MURMURS
b) PDA
c) TGA with VSD ??
d) Pulmonary stenosis
e) L. Ventricular Hypertrophy

o 4days old infant found in cot cyanosed but no murmur heard, O2 saturation decreased in
spite of giving O2, asking Dx? Maybe choanal atresia???
a) Transposition of great vessels== early and progressive cyanosis that does not
correct with oxygen

b) TOF
c) VSD
parents brought 4 days child collapsed in bed CYANOSED HAND AND FEET .hr110 rr40
asd
vsd
ps
tof

A 5day old baby born at term suddenly collapse and was


severely dyspnoeic while still in the hospital. Examination
reveals a centrally cyanosed baby, tachycardic HR164b/m and
tachypneic RR %56cpm, temperature was normal, spO2 78%.
No murmur heard on cardiac auscultation. What is your
diagnosis?
a. PDA
b. Transposition of great vessels with associated VSD
c. Left ventricular hypoplasia present on day5 because pda closes ans
d. Patent foramen ovale
e. Tetralogy of Fallot


3.patient went with wife to Thailand and came back. Have dyspenea and altered consciousness. Wife
is well. As you prepare to intubate, no frothy sputum was noted. Temperature 38.9.what is the most
likely diagnosis?

Pneumonia

dvt

Pulmonary embolism

malaria.

6.A woman with history of multiple vulva ulcer of herpes type, she has a boyfriend. What advice to
give ?

A. Advice use of condom? Aussie site

B. Abstain from sex during flare of vesicles?

C. Use of anti viral on patient

D. Use of antiviral on boyfriend

7.After a night party a girl comes to the ER in the moring with a high fever of 40c, agitated,
decreased consciousness level, muscle rigidity, tremor. What will be the best initial step in
management?

a. I/V infusion

b. Cool blanket and ice pack

c. I/V Diazepam

antidote= cyprohaptedine
drug= ecstacy

14.2 year old child presenting, mother concerned as he still doent walk. On neuro exam he had
exaggerate reflex of lower extremity. He was born gestation age 26 weeks. At delivery he was given
gentamycin and dexamethasone for respiratory. Cause of his current symptoms?

- Gentamycin toxicity
- Antenatal infection?

- Preterm delivery

- Antenatal hypoxia


an infant who was delivered prematurely at 28 weeks with birthweight of 1100 grams. After
delivery he was given surfactant and intubated and kept in a ventilator for 3 days, now the baby
present with spastic paresis in his lower limbs. Cause?
a. prematurity
b. respiratory distress syndrome
c. intrapartum hypoxia
d. low birth weight
e. intrapartum infection

19..Girl comes to clinix C/o just one genital ulcer U did syphilis & other screening she didn't come
after that for 2weeks

now come with for test result

Her viral culture is all negative but RPR test is 1:64 titre positive

how will u Mx?

A.test for other STD

B.contact tracing

C.advice her to use condom

D.HIV testing

, these present with a low titer (≤1:8) in patients with the following: [2]
• Autoimmune disease (systemic lupus erythematosus, antiphospholipid antibody
syndrome)
• Intravenous drug use
• HIV infection
• Treponemal infection (yaws, pinta, bejel)
• Pregnancy
• Advanced age (10% in patients >70 years)
• Malignancy
• Tuberculosis
• Malaria
• Viral disease (hepatitis, mononucleosis)
• Hansen disease

Nontreponemal (RPR/VDRL) titers can be used to monitor treatment
response. Successful treatment is associated with a 4-fold or two-tube
decline in titer (ie, from 1:32 to 1:8) 6 months after therapy for primary or
secondary syphilis and 1 year after therapy for latent syphilis
If the RPR is also positive (especially at >1:8) and there is no history of treatment for syphilis, a
diagnosis of syphilis is made and the patient should receive treatment. 3. Most people become
negative for RPR with adequate treatment, though some patients who present with later stage
disease may maintain a low titer RPR

32 A 65 years old man with nocturia and long hx of dribbling urine problems presents to you. On
examination prostate is enlarged and smooth with median sulcus palpable. There is dullness in the
suprapubic region. What will be the appropriate next step.

A. Urine cytology (not culture or microscopy)

B. TRUS

C. Serum urine creatinine

D. CT scan abdomen

E. Ivu

45 Senior physician is very rude to a patient who presents with superficial lacerations on arm..he
asks her to get out as she has no place in the hospital..what do u do? A) ask patient to leave b) refer
her back to triage c) get a formal psych evaluation on her from a psychiatrist d)do psych assessment
urself on her e) ask the senior physician to write these remarks himself in the file

79. 24. Pain and erythema ,with midline tenderness on calf in a man. What is the best management?
(Is the DVT? Midline tenderness!!!)

- Unfractionated heparin

- Elevate leg and rest?

- Ice?


past hx of post partum depression. worried about recurrence in this preg. now at 12wks. what is the
next best advice?

a.hold the tx till delivery

b.give fluoxatin now

c.discuss with a phycologist


Hi.., Here are some from me.. (21 April)

1. Lady comes on 10th post-partum day with pain in perineum tear. History of
perineum laceration during delivery which was sutured. Now on
examination there is a perineum wound 2 cm long 1 cm wide and 1 mm
deep. Clean with no discharge. What is next?
A. Apply local antibiotics
B. Put on oral antibiotics
C. Simply keep the wound clean
D. Suture the wound under LA
E. Suture the wound after cutting the edges

2. Mother come to you with her son for control, the boy has ADHD, However
the boy not compliance with his medication. He doesn’t want to take his
medication in afternoon (Lunch time) at his school. What you should do

I chooseà change to the long acting one

3. Man, with severe depression treated with venlafaxine now presented with
agitation pressured speech euphoria and mania ask what next appropriate
thing to do
a. add sodium valproate.
B. add clozapine
c. add olanzapine
d. Withdraw venlafaxine
e. add risperidone

4. Patient got operated for appendicectomy. There was carcinoid incidentally


detected which was resected along with appendicectomy. Persistent
diarrhoeapresents ever since preoperative period. No other symptoms of
carcinoid. Wat is your initial treatment of choice.
1. Loperamide
2. Octreotide

5. 36 years old female migrates to Australia after living 6 years. away from
husband. In her country she was abducted and raped by the militia. Now
investigation shows her to be syphilis positive but her husband RPR was
negative 1 month back. What to do to husband?

A. treat husband with ceftriaxone


B. do nothing
C. repeat RPR after 3 months
D. do serial RPR for 3 months

6. Pregnant women 28 weeks gestation, oral glucose tolerance test 8.9(SURE)


what to do?
a) dietician
b) start metformin
c) start insulin
d) do hba1c

7. 7 days old baby presented with jaundice since 4 days of life, his birth was at
term, not complicated and he is breast feeding , serum bilirubin level was
240 (normal <200) with 120 direct bilirubin (it’s 120 not 12 confirmed), his
liver is 1 cm palpable below costal margin. What is the most appropriate
next step?
A-thyroid function test
B-abdominal ultrasound for liver and biliary tree.
C-coombs test
D-stop breast-feeding and follow up
E-phototherapy

8. Man get headache suddenly in the middle of night and wake him up from
sleep. It is unilateral headache with lacrimation as well. Ask the most likely
Diagnosis.?
A. Cluster headache
B. Tension headache


9. 75-year-old (I forgot man or woman) complaint of having difficulties with
swallowing solid food. The patient also got food being regurgitated. What is
the most likely Diagnosis.?
A. Pouch Pharyngeal-zanker diverticulam ,in elderly pt charecteristics
symptom is coughing immediately after a meal and regurgitation of
food
B. Achalasia
C. Stricture oesophagus (something like that., I forgot) –uncommon wth
H/O GERD

10.A woman on Carbamazepine, come to you and ask about contraception (


enzyme inducer –increase the dose of OCP )

11.Food should avoid during pregnancy?


A. Soft cheese (I choose)
B. (I forgot the rest)

12.There is a surgeon always shouting to the staff or others. It’s happened
several times. The staffs get upset and some crying until left the operation
room during operation. You are intern, what you should do:
talk to surgeon,
talk to anaesthetist,
raise the problem at the intern meeting,
report to director of clinical training
(Forgot the last option)–

13.Female 74 years, H/O of productive cough with shortness of breathing, little


sputum for 2 years, progressive dyspnoea with no orthopnoea, H/O cardiac
stent for 4 years, chest lung examination (hyper resonant), come to
emergency, 02 saturations 80% with no distress, fine basal crepitation,
diagnosis (Lateral and PA X-ray given, look like COPD)
a) asthma
b) COPD
c) left ventricular failure


14.Testicular swelling scenario, testes is soft and asking what investigation to
do?
a. Ultrasound
b. AFP and beta HCG

15.64 years old man with PSA 1.5,after I think 2 months time PSA becomes 3.0
(n<4.0). On DRE, prostate is enlarged and regular. On USG-guided biopsy of
12 slides only 1 locus pathology showed adenocarcinoma of Gleason 4
stage. What is the management?
A Radical prostatectomy
B. Continue surveillance
C.Transurethral prostatectomy
D.Radiotherapy
E. EBRT


16.Man presents with one alopecia patch on scalp and rash in hands and legs.
He recently travelled Asia alone and live in Darwin for 3 months. What is
initial investigation for his condition?
a. Syphilis serology
b. Thyroid function test
c. Serum zinc level
d. Malaria blood film
e. Don’t remember now and it is blood test too

17. Old female with depression for 6wk after dead of her partner, sleeps badly,
she lives alone now , she had the same symptoms in the past when she lost
her child , what is the prompt Rx?
A)Olanzapine
B) ECT
C) midazolam
D) venlafaxine

E) temazepam

18. 35-year-old lady comes to you regarding screening for breast cancer screening.
She says her paternal aunt was diagnosed with breast and ovarian cancer at
the age of 60 years which was diagnosed as being associated with
BRCA1.Which of the following is the most appropriate advice?

A-Genetic counselling assessment


B-refer for BRCA 1 screening
C-screen now
D-tell her to look at her breast
E-annual mammography

19.A pregnant woman presented at 16 weeks gestation asking you for a
diagnostic screening of down, what is the diagnostic test you should do for
this woman?
A-nuchal translucency ultrasound
B-maternal serum and ultrasound
C-maternal serum
D-amniocentesis
E-chorionic villous sampling

20.Old man presented to you complaining of rest tremor and bradykinesia,


also taking risperidone he has a known history of visual hallucination and
forgetfulness. what is the most likely dx?
A. Parkinson disease
B. Lewy body dementia
C. Alzheimer disease
D. acute delirium
E. schizophrenia

21.A student thinks that he has brain tumour. The final exam is near, and he
gets more tired, headache and stressed due to tumour. All neuro exam and
CT are normal. After CBT, he still thinks he has brain tumour. What is the
diagnosis?
a. Hypochondriasis
b. Somatisation disorder
c. Conversion disorder
d. Factitious disorder

22.3.2 kg baby was delivered vaginally with no complication except for slight
meconium stained liquor, normal foetal heart rate at birth, after one
minute, become limp, cyanosis, decreased HR, what is most appropriate
next step?
a. Intubate
b. Bag and mask ventilation
c. Aspiration for meconium
d. CXR
e. Na bicarbonate to infant

23.Aboriginal woman, needs hospitalization but refuses to get admitted,
wanted to show her condition to a traditional healer. What do you do –
a. inform the police
b. admit her without her consent
c. involve an indigenous officer
d. arrange a meeting with the healer

24.The woman who is working as baby sitter is 7 weeks pregnant, the child is
confirmed to get rubella. She is concerned. What to advice?
A. Check rubella IgG
B. Vaginal USG
C. Chorionic villous sampling for rubella PCR
D. Vaccination for rubella

25.72 years old Man come with low back pain. On investigating X ray, found
L4-L5 compressed fracture. Many lab investigations: low Hb around 85 ,
mild leucocytosis , ESR 108, What will you do to know the cause of fracture
?
A. PSA
B. Bone marrow examination
C. MRI spine
D. Spine X ray

26.Woman whose son is a drug addict, beats her every day. She is afraid and
presents to you. What will u do for the safety of the woman?
A. Tell her that you are obliged to inform the police
B. Go to the police
C. Tell her to go to refugee shelter
D. Advise her to go support group
E. Send the son to correctional facilities

27.Pregnant woman planning pregnancy has essential hypertension well
controlled is already on medication Ramipril I think what action u do next
A. change Ramipril to methyldopa
B. stay on the same medication
C. reduce the dose
D. quit medication

28.Female worried about baby. How to prevent SIDS?
Sleep with baby
Keep baby in supine position

29.Patient with deep jaundice, moderate ascites, and confused complaining of
abdominal pain due to tense ascites. Flapping tremors and hepatic
encephalopathy feature. Liver is difficult to palpate. Complaint of high fever
38’C. What is your next appropriate initial step?
A- diagnostic paracentesis
B- Blood culture
C- USG
D- Fluid restriction
E- IV antibiotics


30.Patient came with penicillin allergy, what’s most important thing to
consider ?
a. Time of onset
b. Fever
c. Itchiness
d. Red papule

31.A lady come to you with complaint for enlarge breast cyst. She already
checked it with other doctor 12 months ago and he said it was fine, no need
further test. Now the result from biopsy showed malignancy. She is keen to
know your opinion about this matter. What will be the best answer to her
question?
A. tell her it is not important now to focus on mistake now
B. ask her physician who said it is cyst
C. give her more information
D. ask her to go the lawyer to revise her medical report
E. tell her that we don’t have her file any prove that she had cancer
before.

32.Girls refuse going to school. She claims that she has abdominal pain in the
morning. So, she stays at home. Her academic at school good. She also
enjoys playing with friends. When you talk to her you find that Her mother
just sick or diagnosed (I think cancer or something like that)). What is the
cause of her school refusal.

33.Concerning woman came to you and asking for Anti biotic for her son who
is feverish. Her son visited child care and she notice that one of the girl in
child care admit to hospital due to meningococcal meningitis. That girl
usually come to child care Monday to Wednesday, but her son go to child
care on Friday. Hospital give to all contact of girl rifampicin as prophylaxis.
What to do:

A. Give her son AB
B. Give her son rifampicin
C. Ask her to get her son to ER for review
D. Give her explanation about meningitis

34.Scenario about Cataract (Patient with DM.., now has problem with his
visual, particularly in bright become dazzled). sorry I forgot in detail.
Aske Most Likely Diagnosis of his condition.

35.I got X-Ray CaecalVolvulus (Ask Diagnosis) I forgot the detail of scenario.

That’s all I remember so far. HOPE WE ALL IN BEST OF LUCK… Aamiin


I didn't get much recalls..only 10-15 recalls I guess.. I did around last 6 months recalls and realized it
wasnot enuf ...got many new questions or old recalls may be which I didn't go thru....overall xam
was not good for me but lets hope for the best and prepare for the worst...

I got many xrays of viral pneumonia..bacterial pneeumonia..TB...pulm htn..aortic dissection which


were really difficult to diagnose..I still think I am wrong in all those..very very unclear xrays

Ct of hamopneumothorax...ct of ?tuberculoma or ?ncc


Many many many ecg...these also very unclear and weird scenarios...

wpw...young pt

v tach... after mi sceanario

brugada ecg as well...

one ecg seemed like t wave inversion( question was with severe central chest pain)

I got total of 9 ecgs and others I don't remember now

No fundoscopy..no ctg..no usg pic

Confusing pic of rash in a child seemed like measles rash asking for exclusion in school== 4 days after
onset of rash.

One weird question about prevalence of suicide in australia..just some % were given in option...was
a blind guess for me

Aboriginal.mamu recall=aboriginal worker

plaster cast of scaphoid #....

fibular fracture xray with unstable mortise...( there was some gap between lateral malleoli
and articular surface)
ray given (joint space is a bit narrow , may be little displacement) Football player , can’t stand ,
painful ankle joint movement , ask diagnosis ?
A. Fibula fracture
B. Fibula fracture with fracture articular surface of tibia
C. Fibula fracture with fracture articular surface of tibia & joint displacement
D. Fibula fracture with joint displacement
E. Fibular fracture with ? mortise


Abd paracentesis recall....

thoughts of harming baby recall

Lady after car accident tubular vision recall


Patient will h/o DM and HTN hurt her wrist when she crashed into a parked car. On examination
there was significant peripheral vision loss which she was unaware of. Next appropriate step :
a. Check Intraocular pressure b.CT head

10 days old newborn presented w jaundice started at day 3 his bilirubin level is 380 what is your
appropriate management —-

Phototherapy

Exchange transfusion

Reassure

30 year indigenous lady presented with sob and cough since weeks. o/e temp 37.8, PR, RR, BP,
saturation all are within normal limits, mitral stenosis with bilateral basal crepts. X-ray given with
features like bilateral symmetrical upper lobe infiltrates (X-ray seems like bilateral apical rounded
oapcity but not too high at the apex). Diagnosis?

A. Left ventricular failure

B. pulmonary hypertension

C. rheumatic fever

D. Peumocystis pneumonia

E. TB

40 years old man complain from persistent abnormal thoughts that’s making him washing his hands
at least 10 times after touching the key or door.he is on SSRI but want to try non pharmacological
therapy to help him get rid of this thoughts

Teach him how to avoid touching the doors or key

Tell him that this thoughts is normal to relieve his anxiety


Refer him to neuropsychologist unit

Refer him to insight psychotherapy

teach him about exposure and therapy

25-year-old man came to ER with history of back pain L4-L5 level. He denies any history of back
injury. Previously, he was drug abuser and Hepatitis C positive. Physical examination is normal. He
has an erythema at the back which is painful. Which of the following is the most appropriate? Dx
osteomyelitis.

A. HIV serology

B. CT spine

C. MRI spine

D. Kaposi sarcoma

E. Erythema multiforme

Lady comes on 10th post-partum day with pain in perineum tear. History of perineum laceration
during delivery which was sutured. Now on examination there is a perineum wound 2 cm long 1 cm
wide and 1 mm deep. Clean with no discharge. What is next?

A. Apply local antibiotics

B. Put on oral antibiotics

C. Simply keep the wound clean

D. Suture the wound under LA

E. Suture the wound after cutting the edges

A child previously treated Viral Infection for 1wk.now come with petechiae on trunk & arm,non
blanchable.
Lab
Hb -low
plt -decrease
no mention abt WBC , RBC count
A. Bone marrow aspiration
B. Epstein barr serology
C. coagulation test
D. platelet Function test
E. urine RE

The clinical diagnosis of ITP depends on there being manifestations of
thrombocytopenia without other abnormal findings, in particular no pallor,
lymphadenopathy or hepatosplenomegaly.
Confirmation rests on the adequate exclusion of other causes of
thrombocytopenia. The most important conditions to exclude are acute
leukaemia, other marrow infiltrative conditions and aplastic anaemia. An FBE
(including blood film) will usually confirm the diagnosis.
A bone marrow aspirate is an invasive procedure with some morbidity in
children who bruise easily, and is only necessary if the diagnosis is uncertain.
It is rarely necessary in uncomplicated ITP

8yr old child brought by his mother, he complained from episode of staring suddenly that occur
along with fidgeting of right hand and movement of right arm head twitching to right side,
sometimes chewing and lip smacking each episode last for 60-90 second then the child remain dizzy
and confused for 1-2 minutes after the episode. These occurs in 3-4 days and then the child back to
his normal activity and behaviour for several weeks. what is the most likely diagnosis?

A- Temporal lobe epilepsy

B- Juvenile myoclonic epilepsy

C- Absence seizure

D. Hypsrhythmias

E. Rolandic seizures

67. 45 yr old lady smoker and htn uses ocps for many years now comes re prescription, she says
doesn't want to come off because it has controlled her menorrhagia and also says doesn't want to
concieve. What to do now

A. Tell her stop smoking then can give ocps

B. Use implanon

C. Use copper iud

D. Use progestogen only pill

E. mirena


A 36 year old woman with 3 children and BTL done, presents with some vaginal bleeds after
intercourse. PAP smear shows a CIN 3, her last PAP 2 years ago was normal. What is next in
management?

a. Excision biopsy

b. Repeat Pap smear

c. Hysterectomy

d. Hysteriscopy and biopsy

e. Cone biopsy

9 months old infant with deformed occipito temporal region of skull and prominent frontal region,
what to do?

Change sleeping position

CT

Ultrasound

MRI

Reassure

old patient with decrease in vision from past few months .vision s 6/18 on right side and 6/12 on left
and on correction with pinhole is 6/9 on both sides .What is the cause fr this condition? Jm 879

Retinal degeneration

Glaucoma

Cataract

refractive error

presbyopia

old Pt come with 10 months history of reflux at night, epigastric tenderness, loves spicy food,
smoking 60 cigarettes ,drinks 15 standard drink/ day of alcohol, he is obese waist circumference was
high and asking what will u advice for long term mx
A. Stop alcohol

B. Stop smoking

C. Avoid spicy food

D. Reduce obesity

E. Long term ppi

Man presents with multiple alopecia patches on scalp and rash in hands and legs. He travelled
thailand 2 months back and stayed there for 1 month and had sex with multiple prostitues. What is
initial investigation for his condition?

a. Syphilis serology

b.Thyroid function test

c. Serum zinc level

D. Malaria blood film

E. HIV serology

woman with 28 wks of gestation from rural area, 50km away from base hospital,150km away from
tertiary hospital, presents with rupture of memb, after giving glucocorticoid and antibioands next
best step in her management

send to base hospital

send to tertiary hospital

refer to local town hospital

send home

Induce labour now

Patient with deep jaundice, moderate ascites, and confused complaining of abdominal pain due to
tense ascites. Flapping tremors and hepatic encephalopathy feature. Liver is difficult to palpate.
Complaint of high fever 38’C. What is your next appropriate initial step?

A- abdominal paracentesis

B- Blood culture

C- USG
D- Fluid restriction

E- IV antibiotics

RTA pt with SEVERAL(question said several..no.of ribs not given) front ribs fracture.. in pain which is
radiated to back side with sob..no vitals given...what will you do next..(sceanario seemed like flail
chest or aortic rupture)

Echo

CXR

USG

intubation

Ecg

Depersonalization and derealization occur in :

Depression

Schizo

Ocd

mania

all of the above

Pt collapsed..cpr started and 200j shock given..still same condition no rythm..what to do.next

Continue cpr for 2 min and then give shock

give shock again

old male has metastatic prostate cancer ,he is taking morphine for pain relieve and

self-prescribed cannabis but recently they are not working and he is complaining of insomnia

and agitation ,what will be better for him:

a.sleep hygiene

b.motivational therapy
c.supportive psychotherapy

d.interpersonal therapy

e.refer to drug prescribing authority

20 yo woman coming to ask about screening for ovary cancer. No family history, no symptoms, only
colleague at work has this dg. Options: Start screening at 40, reassure that no need to screen, screen
now

Boy drops out from school, has history of stealing with no regret. What would be predisposing
factors for his condition? Conduct/anti social

14 yo boy, delay at school for 1 yr, starts passing stools in his pants, reason? Developmental delay,
sexual abuse, inflammatory bowel disease

Gyn: 22 yo woman yellow greenish discharge. Etiology? Gonnorhea, Chlamydia, Trichomonas

Travelling to northern Thailand, comes back after 4 weeks fever, headache, etiology: HIV, malaria,
dengue (HIV incubation period 2months to years)

Woman nipple one side discharge, blood stained from one duct. Management: excision of one duct
(JM 1119)

Papillary carcinoma in left lobe, management: Thyroidectomy, Left lobe thyroidectomy

88 year old woman, history of right hip pain, X ray shows osteoporosis, what give? Bisfosfonate,
calcium, vit D, estrogen, estrogen and progesterone
38 yo woman, history of hysterectomy, oophorectomy 5 years ago for menorrhagia, T score lumbar -
2.6, T score femoral head -1.5, normal serum calcium, what to give: bisfosfonate, calcium, estrogen,
estrogen/progesteron, vit D

Shoulder dislocation, what sensory loss: lateral of shoulder, medial of upperarm, lateral of
upperarm,

80 yo woman shoulder pain bilateral, problems with combing hair, no remarkable history, no pain on
touch, what would be most probable cause? Osteoporosis, capsulitis, biceps tendonitis

Woman 3 hrs into labor, everything okay, starts bleeding, bp drops, baby puls drops, reason:
Placenta praevia, amniotic embolist, coagulation, placenta accreta

School boy non compliant for ADHD medicine, grades drop, what to do?
Ask school for support,
let the boy decide about the medication
supervise parents
talk to the boy, (JM 1051)

Woman trauma injury, small pneumothorax, saturation 98% in room air, what to do before she’s
transported to hospital? Chest tube, intubate, give oxygen

Men 5 days post injury after falling from motorcycle, wounds over body, didn’t go to hospital on the
day of accident, develops high fever, what to do after administering antibiotics?
Debridement,
biopsy

Diabetic ulcer on foot with erythema:


Cover
bed rest
antibiotics
skip insulin

Woman refugee raped, reunited with husband after 6 years here in Australia, after a month tested
for syphilis, husband negative, what to do with husband?
Test in 3 wks again
monitor for 3 months
advice to use condoms for 3 months

Men HIV positive, woman negative, men refuses to tell his wife what to do next?
Inform authorities and trace contacts
tell his wife
refer him to infectious diseases department

Aboriginal boy smokes marijuana, threatens everyone in ED to kill them, after calling security, what
to do?
Ask aboriginal worker
discharge him
call police
hospitalize him

A guy came back from Africa, high fever, you check for malaria, took antimalarics, twice negative
results for malaria, the test has 98% sensitivity, 96% specificity, what to do next?
Check for different disease
blood smear-thick and thin
test in high peak of fever

Patient post op hemicolectomy for caecal carcinoma, low Cl, K, Na, reason?
Hyperaldosteronism
SIADH (JM235)
too much glucose administration

Patient sweating, obvious thyroid, pt losing weight, eating a lot, detected nodule, low intake, dg?
Subactute thyoiditis (JM 232) ( Uniform uptake – graves disease,MNG---- irregular uptake,low
uptake--- de quervain thyroiditis/ subacute thyroiditis,thyrotoxicosisfactitia, no uptake---
cyst,Hge,carcinoma)
acute multinodular goitre
acute toxic adenoma ????
Graves
The recall on FNAC of the biggest of thyroid nodes, which showed blood, colloid and follicular cells.
The nodes are non-toxic. What to do next?
a. Repeat FNAC
b Core Biopsy
c. Partial thyroidectomy

Pt with cystic nodule on thyroid.Aspiration of dominant nodule yielded,Follicular cells,Colloid


Cells,Blood..No residual mass after aspiration..Whats Next?
A.Repeat FNAC
B.USG

C. CT
C Marika, aged 62 years, has noticed a swelling in the front of her neck, which is enlarging quite
rapidly. She is otherwise well, without symptoms of hypothyroidism or hyperthyroidism. On
examination, she has a multinodular goitre, with a dominant nodule of 4 cm diameter in the right
upper pole of her thyroid gland. Of the following investigations, which is the MOST important in
assisting diagnosis?

1. Thyroid function tests

2. Thyroid ultrasound

3. Fine needle biopsy


4. Thyroid isotope scan

5. CT scan.CT


Topics:
Nephrotic, nephritic syndrome
ophtalmology
abdo x rays
delirium
personality disorders
statistics-positive and negative predictive value,
A 29-year-old lady was recently diagnosed with nodular thyroid disease.She is worried about getting
thyroid cancer.

Which of the following does not increase the risk of thyroid cancer?

a. Family history of benign thyroid disease b. Family history of thyroid cancer c. Graves disease d.
Chronic goiter e. Familial adenomatous polyposis
7..Lump infront of throat& lymph node enlargement in the anterior triangle, dysphagia& hoarseness
of voice & weight loss what diagnosis

Cancer esophagus

Cancer thyroid

Cancer lung

pic. of old lady with large mass in neck seemed multinodular in appearance,c/o
hoarsness,dysphagia,cause?

Ca thyroid

multinodular goitor

A middle aged man who had a history of surgery for achalasia a few years back presents to you with
a complaint of hoarseness for 1 month and dysphagia for 2 weeks. On examination , vocal cord
paralysis was noted and a mass in the thoracic inlet was seen in imaging. What is the probable
diagnosis?

A. cancer of the laryn

B. cancer of the oesophagus

C. cancer of the lung

D. cancer of the thyroid


Delhi 18 /4/2018

1.Young age primi presented to you at 36w she is telling you that her
friends tell her abdomen looks smaller ,she came at 32 w and
everything was normal
What is the most important indicator of her baby's groth:
a.the fundal level is increasing in every visit
b.her pupic-fundal level is 34 cm at 35 w
fetal movement in previous 36h is (number I forgot it )
d.normal CTG

2.74 yrs old male has metastatic prostate cancer ,he is taking morphine
for pain relieve and self-prescribed cannabis but recently they arenot
working and he is complaining of insomnia and agitation ,what will be
better for him:
a.sleep hygiene
b.motivational therapy
c.supportive psychotherapy
d.interpersonal therapy
e.visual photo therapy (something like that )

3.old male with heart faluire ,HTN,diabetes ,he is on clozapine for three
weeks now complaining of shortness of breath and palpitation ,HR,Bl.p
all normal what next :
a.clozapine level
b.echo
c.FBC
D.irrilevant

4.Old age woman on multiple drugs digoxin ACEI BB ,she stopped all
drugs 3 weeks ago as she was in a trip and ran out of drugs ,she is
complaining of confusion ,and occasional postural dizziness (the exact
words ) ..lab results show :
Creatinine 118 (normal up to 90 )
Non fasting blood glucose : 4
Blood p. sitting 168/70, standing 130 /60
What is the cause:
a. dehydration
b. renal impairment
c. hypoglycemia if <4mmol/l ( dehydration - mild form of renal
impairment )

5.old man stopped taking drugs, now he has edema up to knee, HR 106
HTN with clear lungs what next?
a.fursmide
b.spironolactone
c.digoxin
d.BB
E.recommence ACE ,BB,other drugs but not diuretics

6.A policeman is coming for his collegue that beaten his friend during
an argument,he is intoxicated for alchol what in history will be important
to say personality sdisorder
History of fight with colleague two weeks ago
History of animal crulity during childhood
Alchol and drug abuse

7.7 weeks child with high fever vomiting increased until it is now after
every meal no other signs, urine exam revealed 5*10*9 cells what next :
Oral cipro
Oral amoxi
IV gentamycin
IV cephalosporin
JM 231
EMPERICAL : IV GENTA+IV AMOXY/AMPICILLIN
8.Scanrio of temporal lobe epilepsy treatment
a.valproate
b.ethoxsumide
c.lamotrigine

9.same scenario of progressive constipation through 3 weeks in old age


(3 questions)
one with empty rectum and no gases in x ray just distention of
ascendind ,transverse,descending colon with progressive abd,
distention and constipation
(the three questions the same ..progressive ,3 weeks ,no fever )
Diagnosis :
A cecal volvolus
B. sigmoid volvolus
c.cancer sigmoid

10.old age is coming for vaccine as she will be a grandmother soon,


what is the better for her:
DPT
Measles polio
Influenza

http://www.ncirs.edu.au/assets/provider_resources/schedules/adult-
immunisation-schedule-table-September-2016.pdf

http://www.ncirs.edu.au/assets/provider_resources/fact-sheets/adult-
vaccination-fact-sheet.pdf

11.Old age ,felt 4 hour retrosternal chest pain ,the pain was proceeded
by aggressive vomiting ,left lower lung dullness ..invx:
Troponin
Gastrographin
Irrelevant
12.Old age female ,heavy smoker ,complaining of painless lump in her
neck by examination you found a lump supraclav. Subcutaneous
What will lead to diagnosis
Bronchoscope
Gasteroscope
MRI neck
Mammography
FNAC

13.Young male complaining of pain in groin after lifting a heavy object


,by exam no lump but US revealed 1 cm defect in inguinal ring with fat
herniation but no visible lump ,next ;
Open laparotomy mesh
Laparoscopic repair
Check again if lump appears

14.Young female came with fatigue and tiredness ,Hb decreased from
11 to 9
Ca 1,9
INR 1.6
Na 130
K 5.5
Next best rx?
a.blood transfusion
b.IV Ca
c. vit. K
d.Na
e.k

15.old lady says that when she goes to sleep,before falling asleep she
feels crowling insects and this annoys her otherwise she doesn't feel
anything like this during the day ,,only when go to sleep next
Echo
Sleep studies
irrelevant

#April-Hobart

1. The q of the baby growing well but when her mother breastfeed her she
archs her head away from breast ----- GERD

2. A lady bringing her husband 35 yrs old because he is having strange behaviour
and slow gait , his father and uncle has same condition and died early ----

A 35yr man came with confusion dysarthria wide based gait and as an accountant he has been
behaving odd. His father and paternal aunt died with same issue. What will lead you to
diagnosis?
Copper and ceruloplasmin level
Mri
Genetic testing ( Wilson disease )

3. New Q: of 16 old boy who his grades fall because he doesn't pay attention to
lectures and he do some Sketch drowing instead of studing , he had fight with
one of his colleagues and when you interviewed him he couldn't make eye
contact with you and was unable to explain his condition , he denied taking
Alcohol or non prescribed medications -
A Asperger
B Dysthemia
C bound alcohol and drug
D Conduct
E Schiz

4. Classic case of acute angle Glucoma ( Halos around light ) asking for long
term management
Irridectomy

5. You r in intern doctor nurse call u,a 10%burn want to discharge by
himself, he is heared your conversion, he is very annoyed
something like that's, what is you next mx
1.tell the nurse u will come & see him
2,discharge him & following him in as outdoor pt
3.inform burn registrar unit that pt want to discharge
Rest option i forgot

6. wasting of hand muscle, weakness of tibialis anterior, weak hip flexors,
fasciculation in hand n leg. All lower motor signs with few weeks history.
No sensory or cranial nerve issue Options:
a) Emg
b) ck levels
c) anti ach antibody
d) b12 levels

the question of the lady who wants to change her will but her daughter
refuse because she thinks her mother have mental disorder
on examination she had short term memory loss like the last check ups ,
asking what to do
refer to psychology for further assessment
tell them she can change her will
tell them she cannot change her will
suggest a lawyer

A cyclist fall on the ground with obvious deformity of the tibia
his leg is pale and pulsless
what is the most essential first treatment for this boy
surgical debridement
lavage
reduction

A lady brought by her friends , she feels irritable and frightened
she feels anxious outside and enjoys her life in her home , she doea gardening
and read books and do crafting , what is the most significant in her history
Alcohol or drug abuse
School refusal

lewy body dementia Q

A 12 yrs old girl in foster care as her mother and father were drug addicts and
has large collection of dolls and talks with imiganry friend and picky in her
food and this annoying her carer what's imminent risk for her
OCD
Anorexia nervosa
sexual abuse
Schiz
Drug abuse

Child presented with UTI, on USG the right kidney is smaller than the left.
What is the best investigation to assess renal function?
a) DMSA
b) DTPA
c) Urine culture
d) CT scan

a lady you want to admit to the ICU for serious condition I don't remember
now but she doesn't want to be admitted , she is irritable and her mmse is
fine ,, and you insist on admission what to do
Admit her after court desicion
Family meeting to force her
Counsel with the patient and discuss the need why you want to admit her and
that's for her interest
let her go home upon her wish

the recall of the man who wants to swim the ocean because God told him to
do so , so why you would admit him involuntary
the risk of his beliefs

a lady complains of headache for 8 months that awakes her every morning
She has been taking paracetamol and OTC pain medication that relief her 1 or
2 hours and then feel pain again
she mainly complains that the headache in bitemporal OR frontal
Tension Headache
Rebound Medication Headache
Migraine

A lady was in Operation for hours and stayed in lithotomy position (
mentioned exactly)
After that she suffered of foot drop
where the lesion
S 1
neck of fibula
L 5

Syntocine Qs
avoid head electrodes for the baby for hep C
lateral epicondylitis 2 Q one carpenter and one lady working on comptuter
with using the mouse alot
decreased frequency of breast feeding Q

Low BMI girl 15 and she stopped going to Ballet classes , has fights with her
father and when you interviewed her she tells what's the need for all of these
, what to do next :
Liase with father
discuss safety plan for her and her family

a lady is crowling her leg ,, feeling cramps while in bed , she told you that she
doesn't feel relief unless she walks around the room and do some leg
massage and on the rest of the day she is fine
so the answer is restless leg sy and answer is iron studies


A 40yrs old woman on cocp for 8yeaes. She is a smoker and her BP is high.
She doesn't want to get off pills. What is your advise?
A. Stop pills and rely on condom
B. Copper IUD
C. Implanon
D. POP


Melbourne 21.04.18

1. An Old man 75 years of age has 3 week history of constipation. He presents with mild
abdominal tenderness and BS exaggerated on rectal examination there is no faeces in
the rectum. Which of the following is the most likely diagnosis?
a. Sigmoid volvulus
b. ca sigmoid
c. Caecal volvulus
2. A above 60 years old patient with history of appendicectomy and cholecystectomy
presented with 3 weeks history of abdominal pain, distension. CT abdomen given.
Rectum is empty. What is the cause?
A. Sigmoid volvulus
B. Adhesive IO
C. CA Sigmoid colon
D. Fecal impaction

3. 48 year old woman.complains of heavy menstrual bleeding for last 4 months. She
underwent a hysteroscopy and D&C but the symptoms did not subside. She looks pale
and Her haemoglobin is 8.5 g/dl. What is the most appropriate management in this case?
A-oral tranexamic acid during the period
B-northisterone from 15-25 days of cycle
C-mirena
D-implanon
E-COC
4. A boy was recovered from a beach in Northern Queensland. He was in pain and had jelly
fish tentacles stuck all over his legs. Prior to removing tentacles what will be your
immediate step?
A. Pour vinegar
B. Give IV Morphine
C. Immerse legs in warm water (my ans)D. Give antivenom
E. IV fluids
5. Box gelly fish normally found in western australia ,we will give
6. the scaniro of temporary rt side weakness with no vision loss no carotid bruit..cause...
a.lacunat infarct
b.carotid stenosis
c. vertebrobasilar ischaemia
7. Man with right eye- droopy eyelid, diplopia, eyeball depressed inferiorly and laterally. Blur
on looking left. Cause?
a. Right 6th Nv
b. Posterior communicating artery aneurysm
c. Occipital lobe
8.
9. 58 year old woman present just 24 hour onset of pain in the right buttock and tenderness
in her back at L4-5 after back 2 days lifting some heavy thing has difficulty and
restrictions of movement in extension and flexion and rotations , what will you give beside
analgesic?
A- spinal xray
B- spinal mri
C- bed rest
D- orthopaedic surgeon referral
E- keep active
10. Woman with a history of hysterectomy and DVT. Now complaining of hot flushes and
insomnia. What will you give her?
a. Oral low dose estradiol
b. Transdermal estradiol
c. Progestogen only pills
d. Hrt
e. sertraline(paroxetine

11. 48 year old woman.complainsof heavy menstrual bleeding for last 4 months. She
underwent a hysteroscopy and D&C but the symptoms did not subside. She looks pale
and Her haemoglobin is 8.5 g/dl. What is the most appropriate management in this case?
A-oral tranexamic acid during the period
B-northisterone from 15-25 days of cycle
C-mirena
D-implanon
E-COC
12. Farmer with left inguinal swelling biopsy noted squamous cell ca asking which region is
primary
A) Colon
B) Rectum
C) Left leg
D) Anus
13. Pt smoker for 20 years, enlarged tonsils, on exam 1*1.5 mass in tonsillar fossa. Most
probable Dx?
a) Mets from CA
b) SCC
c) Lymphoma
d) Nasopharyngeal CA
14. Syntocinon infusion is given and fetal bradycardia occurred for next 4 minutes.
15. Same question again with similar options but this time bradycardia for 2 minutes.
What to do next?
Stop Syntocinon
Fetal blood sampling
C section
16. You are intern. Patient wishes you to witness her will in the presence of lawyer. ???
a. Witness will b. Refuse to witness
c. Ask surgical registrar if he can do it
d. Ask the patient if he is happy with intern witnessing his will
e. seek legal advice
17. Doctor knows both husband and wife. Wife comes with the changed will which the
husband has already signed and asks you to sign as a witness.
What will you do?
Refuse
Sign
Call the husband and ask about it
18. 52 year old woman had her last menstruation 8 months back. She is complaining of hot
flushes and insomnia. What is the best treatment option for her?
A. Cyclical HRT
B. Continuous HRT with 12 days Progesterone
C. OCP
D. Estrogen patches
E. Continuous Progesterone for 30 day
19. You are a doctor in a remote area where a study is conducted to determine the
prevalence of Hep B. The study included 200 patients and showed the following results
2009 2010
Patients with Hep B Ag +ve only 5 7
Patients with +ve Hep B core Ab and Hep B Ag 10 12
Patients with Hep B coreAb+ve only 50 60
Calculate the incidence new cases in 2010 in 1000 patients from this population?
25
50
75
300
375
20. A rural area doctor wants to carry out a study to find relation of rota viral diarrhoea in first
year of life and low birth weight
Cohort
Case control
Cross sectional
Randomised trial
Case series
21. What is the best method of study for studying the relation between fatigue
and accident in truck driver?
a. RCT
b. Cohort
c. case-control study
d. Case report
e. systemic review
22. 24 months old, developmental task for language?
2 or more personal pronouns
Sentence with 4 syllables
Name 4 color
Know age
Know family name
23. Yr old man went through hemicolectomy after 2 days he was confused his labs showed
low sodium low potassium low urea low creatinine osmolarity was normal upper
limit.diagnosis?
Siadh
Hyperaldosteronism
Hyponatremia
sampling from iv cannula site
24. A 28 year old girl came for CA breast screening since her paternal aunt has CA breast
and ovarian cancer diagnosed at 48 years old. What is your appropriate management?
A:Conduct genetic risk assessment
B:BRAC1 muatation screening
C:Reassurance
D:Early mammogram screening
E:Self breast awareness
25. Opportunistic screening in a 16year old:
Skin cancer
STD
Scoliosis
Depression
26. Child from daycare with post streptococcal Glomerulonephritis. When can she return to
school?
Immediately after discharge
After throat swab is done for all students
After prophylaxis for all students
After microscopic hematuria resolves
Low protein diet and minimal activity
27. Old man with back pain tender lumbar spine hb 10 esr 109 normal calcium
A thyroid test
B PsA
C mri
D BM Xn ( multiple myeloma )
28. Old man had urinary problem. 2 months back pain. On exam tenderness at L4-L5. DRE
shows hard, irregular large prostrate. Next?
a. CT lumber spine
b. PSA
c. PETscan
29. Girl with fever, sore throat, lymphnodes, what to confirm diagnosis?
A. General maculopapular rash
B. Diffuse pharyngitis
C. Knee pain
D. Spleenomegaly EBV
30. Child with fever, conjunctivitis, rash lymphadenopathy, abdominal pain. most appropriate
Investigation (Kawasaki disease)
a. Aso titre
b. ESR
c. Echo
D. Ana
31. A lady presents with some fine tremors of her hand at rest which disappears when she
watches it. She has no rigidity and bradykinesia. What will you give?
a. Benzhexol
b. Propranolol
c. Diazepam
32. Woman with symptoms of hypomania asking for treatment
a) Olanzapine
b) Lamotrigine
c) Sodium Valproate
33. lady overdose of Venalafaxine prescribed for her,she is irritable at work and
insomniac,she has low mood.wht u add for her after recovery from overdose
A.sodium valproate
B.olanzepine
C.risperidone
D.estrogen patch
34. Food to avoid in pregnancy
a) tinned salmon
b) soft cheese
c) coffee
35. Ecg of atrial fibrillation given. Management asked?
a. Warfarin
b. Digoxin
c. Beta blockers
36. ECG of NSTEMI given. Chest pain started 2 hours back. Now pain 4/10. taking
ramiprilatovastatinaspirin . No dypnoea, murmur. What should be given now?
a. Thrombolysis
b. aspirin,
c. GTN,
d. clopidogrel,
e. metoprolol
37. Aboroginal female has mitral stenosis. she presented with dyspnoea. C xray given. it had
bilateral
middle LOBE OPACITY. Whatsis the diagnosis.
A. Pulmonary Hypertension.
B. Pulmonary Fibrosis
C. LVF
38. Xray of a pt with history of chronic cough for 4 weeks smoker now rusty sputum asking
diagnosis
A. Bronchiectasis
B. Copd
C. Pul fibrosis
D. acute on chronic bronchitis
39. chronic bronchitis pt, H/O yellow green sputum expectoration for last 3 years. Now
developed fever cough and some copious amount of yellow green sputum. What to give?
a) roxithromycin
b) IV ticarcillin/clavulanate.
c) iv ampicillin
40. An old man presents with 6days history of surgery for esophageal achalasia. On
examination, there is dullness to percussion on the left lower lobe. Which of the following
will help you identify the diagnosois?
A-CTPa
B-non contrast CT
C-Chest X-ray
D-endoscopy ( contrast study ..related to HB ques 441 )
41. Most common reason for mothers often complaining of reduced milk supply?
a) Decreased frequency of breast feeding
b) Decreased time at each breast
c) Exhausted
d) Mother drinking Caffeine drink
e) Poor positioning
42. mother which brings her child for the 5th times during the 6th wks , Breast feeding
established and baby well growing.what imp. u ’ll ask her about ?
1. Premorbid personality
2. family conflicts
3.her attitude towards breastfeeding
4. Mood
43. A father came with her daughterfor haemochromatosis screening. What screening test
advice would you offer?
A. Screen daughter now
B. Screen only father but not the child
C. Screen daughter after 18 years of age
D. Do not screen daughter
44. History of orchidopexy, infertility, sperm count 1 million, what next?
1. Scrotal US
2. Testosterone
3. FSH, LH
45. An old lady above 60 has history of breast cancer treated surgically 3 years ago presents
for osteoporosis screening her bone density scan shows -2.7 how to treat it
A. raloxifen
B. strontium
C. vitamin d calcium
46. A man with history of limb claudication on 100 meters relieved by rest, on examinations
there was absent left femoral pulse and absent dorsalis pedis pulse, ABI was done and it
was 0.25. What is the most appropriate test leading you to the diagnosis?
a. Arteriography
b. Ct angiography
c. compression Doppler ultrasound
d. MR angiogram
e. X-ray
47. Investigation for venous ulcer
48. A plumber came with a history of chronic cough.he was a smoker. Chest x-ray was given
and there was clear pleural thickening.Asking next
investigation
Ct
Pleural biopsy
49. Farmer lost his wealth owing to draught. Recently he had severe weight loss and depressed
mood. He comes to see the doctor. When asked about his health condition, he did not admit
that his health is in bad condition and tried to justify his loss of weight with his recent
increase in workload. What kind of defense mechanism is it?
a. Rationalization
b. Denial
c. projection
50. Patient was on allopurinol 100 mg daily and had chronic renal impairment. What will you
do. Options were
A. Increase allopurinol,
B. Stop allopurinol,
C. Colchicine and some other
51. Gout scenario with chronic renal failure, now presents with acute symptoms, what to
give?
a) paracetamol
b) codeine
c) colchicine
d) allopurinol
e) NSAID
Upto this
52. A man with dvt of popliteal vein was prescribed warfarin and heparin. He developed a
hematoma. His INR 1.7. His hematoma size has not changed sinced then. What to do?
All options were to change enoxaparin to different LMWH,UFH, or increase warfarin dose
53. Man smoker presented with tiredness lethargy cough dyspnea blood stained sputum
Labs shows
Hb 104
Tlc 40
Plt 90
Blast cells 60%
lymphos n neutros also increased
Diagnosis
A.Acute leukemia
B.Bone marrow infiltration by CA lung
C.CML
D.CLL
54. SLE diagnosed(anti DtsDNA+ and ENA+)hving pain in joints.wht long term treatment?
a.Naproxen
b.Methotrexate
c.Hydroxychloroquine
d.paracetamol
55. SLE -Rash and Joint pain .BestNxt?
a.predisolone
b.Hydroxychloroquine
c.Methotrexate
56. Pic of female with SLE with malaria rash. Diagnostic investigation?
ANA
Anti Smith
ANCA
57. Patient is taking methotrexate, prednisolone and celecoxib for RA. Lab results show
pancytopenia with high esr (67). Management?
a. Increased prednisolone
b. HCQ
c. Folic acid
d. Folinic acid
e. Cease celecoxib
58. Pregnant HCV positive came in first trimester. Best advice to decrease vertical
transmission
a. c section at term
b. avoid scalp electrode
c. formula feeding of newborn
59. Autistic person now gambling, irritable. Asked how to manage now.
60. red eye pic recurrent oral and genital ulcer ask rx
Steroid bechet disease
61. Following a cataract surgery (picture of hypopyon given)
A- Hypopyn
B- Conjunctivitis
C- Uveitis
D- Keratitis
62. 4CT of brain with ring enhancement lesion, but no fever,h/o Clark I melanoma but there
was multiloculation: History os surgery of melanoma of leg 3 years ago.
a. Brain abscess
b. Melanoma metastasis
c. Glioma
d. Brain abscess
e. Tuberculosis Sclerosis
63. Man with dark brown pigmentation on face which had been there for many years and now
increasing in size recently. What’s the management ?
- local excision
- excision with 2cm margin
-reassure
64. Patient having delusion that he is listening microwave voice from his neighbours home all
the time.. Which is least effective
Resperidone
Olanzapin
Clozapine
Amisulpride
Quetiapine
65. Man brought to hospital after throwing brick at windows he said last thing he remembers
is leaving work from rural farm
Dissociative fugue
Factitious disorder
Malingering
66. Dental amalgam and dementia study
a.case control
b.cohort
c.RCT
67. a young girl 15 yr collapsed at excercise,her father had HOCM,after ecg and echo u find
no abnormality,next step
a)holter monitoring
b)stress echo
c)repeat echo after 3 year
68. A pregnant lady came to you with multiple bruises in her thigh as a result of husband
abuse.
What’s the next step in management?
A. Provide an immediate shelter
B. Confront the husband
C. Call the police
D. Take pictures for reporting
E. Arrange couple meeting
69. Fournier gangrene / Necrotising fasciitis photo :
Asking management initial :
Debridement
Hyperbaric o2
Antibiotics
70. Growth chart of child
71. 2 week old pregnant lady was brought to the emergency department with vomiting,
abdominal pain and severe bleeding for the past 4 hours. She also had 2 fainting attacks.
Her blood pressure is 80/40.She was normal till before this episode and USG at 11
weeks showed normal nuchal translucency. Which is the most likely diagnosis?
a. Incomplete abortion
b. Complete abortion
c. Abruption placenta
d. Ectopic pregnancy rupture
e. Hydatidiform mole
72. Obese Pt undergoing some surgery with BMI 32 and previous h/o of DVT...Asking which
will increase risk of DVT if he under goes surgery.?
a- Spider naevi on chest
b- Nicotine stains on fingers
c- Bilateral varicose veins
d- BMI
73. boy wearing girlfriend’s underwear. What is this?
a. transvestism
b. transsexual
74. 4 year old boy wants to b his fav cartoon character,momns concern he talks to his imaginary
friends,he sleeps with his fav panda, dry for the day but sometimes got wet his bed at
night.What it this?-
cca)tell mom this is normal in a 4 years old
b)autism
c)aspergus
75. old lady with stress incontinence=== treatment offered was pelvic floor exercise.she
came back again ,not satisfied as still experiencing stress incontinence, asking most
appropriate next.
a) vaginal cone
b)estrogen cream
c) surgery
76. 5 years old boy with lacerated wound. H/o 2 doses of dtpa at 2 and 4 months. Next
A.dtpa and ig
B.tt and antibiotics
C.tt and debridement
D.dtpa
77. Young male complaining of pain in groin after lifting a heavy object ,by exam no lump but US
revealed 1 cm defect in inguinal ring with fat herniation but no visible lump ,next ;
Open lapetotomy ,mesh
Laparoscopic repair
Check again if lump appears
78. Young female came with fatigue and tiredness ,Hb decreased from 11 to 9
Ca 1,9
INR 1.6
Na 130
K 5.5
Next best
a.blood transfusion
b.IV Ca
c. vit. K
d.Na
e.k
79. Sarcoidosis- skin biopsy and ACE
80. Viral pneumonitis/ TB question
81. 26. Chronic purulent ear discharge - Cipro ear drops
82. 21/4/2018 PERTH
83. Hi all,these are my recalls,as I can remember ,thank you so much all amedexmembers,plz
keep me in your prayers,thank you
84.
85. Exam started with 2 sample ques,
86. One is haemolytic anemia
87. Second one is melanoma eye,refer to plastic surgeon
88.
89. Main exam questions,
90.
91.
92. A 40 year old woman with pain n swelling for 8 years, now came with complaints
of morning stiffness for 2 hreveryday? What is your mx?
93. NSAID
94. Prednisolone
95. HCQ
96. Methotrexate
97.
98. 34 year old male with a history of fall on an outstretched hand with pain and
swelling.XRay was given, (showed scaphoid fracture. Very clear fracture line
through the middle)
99. What is the best treatment option for this patient
100. a. Crepe bandage
101. b. Plaster cast
102. c. Analgesics
103. d. Compression screw
104. e. Plate fixation
105. 9 yrs old with type 1 diabetes. on small n intermediate acting insulin. every
morning high glucose levels. what to do ?
106. a. check blood glucose levels at 3 am
107. b. check early morning insulin levels
108. c. increase the evening intermediate insulin
109. d. give insulin before breakfast
110. e. give another dose before sleep
111.
112. 12 year old girl at foster care.she's having difficult time in school in reading
n writing . she has a collection of her own toys and plays with them with her
imaginary friend. her foster parents/ carers are irritated bacons because she's
picky at food. what immediate danger to her? (not future in Question)
113. OCD
114. sexual abuse
115. drug abuse

116. Anorexia nervosa
117. schizophreniform psychosis
118.
119. apt came with loin pain and haematuria. what is the most app step ?
120. a. CT abdomen
121. b. IVU
122. c. USG
123. d. xray
124.
125. Case of community acquired pneumonia was treated and got well on day
three after giving 2 antibiotics (IV augmentin, oral roxithromycin) and given
enoxaprin. On day five patientstarted having fever 39F, chills and rigors associated
with crepitation in right lower zone. Foul smelling purulent white nasal discharge.
126. A. Empyema
127. B. Hospital acquired pneumonia
128. C. Drug hypersensitivity
129. D. Iv cannula bacteremia
130. E. Pulmonary embolism
131.
132. Patient with RA she takes ibuprofen and methotrexate to control her
disease, they mention the time it was years for both of drugs, patient complains of
upper abdominal pain, and her labs are given ALT, AST, GGT, even bilirubin all were
high, which of the following cause this condition?
133. A. Methotrexate induced hepatitis
134. B. Ibuprofen induced hepatitis
135. C. Autoimmune hepatitis
136. D. Viral hepatitis
137. E. Alcoholic hepatitis
138.
139. The patient has 8 months history of joint pain in the wrist and the ankle and
minimal pain in other joints but now the patient comes with morning stiffness of
both wrists for 1 to 2 hours.
140.
141. Her lab results as follow.
142. Hbà reduced
143. MCV à slightly reduced (nearly below lower margin level)
144. ESR à 70 (sure for this level)
145. A. NSAIDs
146. B. Prednisolone
147. C. Hydroxychloroquine
148. D. Methotrexate
149. E. Etarnarcept
150.
151. 50 years old male patient has sudden retrosternal chest pain and severe
vomiting proceed by pain. He has hypertension and controlled with thiazide. On
examination, dullness lower left lung and reduced breath sound on left lower zone
of lung. Which of the following investigations to reach diagnosis?
152. A. CT chest
153. B. Gastrografinswallow
154. C. Ultrasound
155. D. Echo
156. E. Chest Xray
157.
158. Woman with osteoporosis and taking HRT for 6 years, bone density score
for spine vertebra is -1.7 and for femoral head is - 1.2. What is next? Vit: D and
calcium level are given Normal level.
159. A. Cease HRT
160. B. Continue same
161. C. Change HRT to Alendronate
162. D. Add Alendronate
163. E. Vitamin D and Calcium supplement
164.
165. A patient with spiking fever, Shortness of breath and was inserted
prosthetic heart valves 3 months ago. And diagnosis as due to Staph aureus IE, and
given Flucloxacillin for treatment.The fever subsides after 2 weeks of treatment,
but SOB still persists, diastolic murmur at the left sternal edge. What will you do
for investigation?
166. A. Transthoracic Echo
167. B. Chest xray
168. C. Troponin
169. D. Blood culture for every week
170. E. CT scan
171.
172. A man presents with bradykinesia, rigidity, given risperidone for visual
hallucinations, he also complaints of fluctuating forgetfulness and signs of
parkinsonism. What is the diagnosis?
173. A. Lewy body dementia
174. B. Parkinson’s disease
175. C. Alzheimer’s disease
176. D. Drug induced psychosis
177. E. Frontal lobe lesions
178.
179. Patient with history of DVT, HTN with DM patient on warfarin, metformin,
statin and then add amiodarone, 2 days later suddenly got swelling at right thigh
and become painful, redness and Temp 37.8. Right thigh size is 4 cm larger than
left. Diagnosis?
180. A. Rhabdomylosis
181. B. Hematoma
182. C. DVT
183. D. Cellulitis
184. E. Drug reaction
185.
186. What is the most important indicator in finding health risk of obesity?
187. A. BMI
188. B. Waist Circumference
189. C. Waist hip ratio
190. D. High LDL
191. E. Low HDL
192.
193. A lady with BMI of 35. How will you manage her in addition to exercise for
long term management?
194. A. Low Carbohydrate food
195. B. Lipase inhibitor
196. C. Diuretics
197. D. Surgery
198. E. 4000 kJ/ day
199.
200.
201. 50 year old man with renal transplant. Neck stiffness and meningitis sign
start 3 days ago, nothing mentioned about respiratory symptoms, no fever. On
chest xray there is well defined round opacity in “middle right lung” (not given
chest xray, just mention). What is the cause?
202. A. Aspergillous
203. B. TB
204. C. Pneumocystis Pneumonia
205. D. Lymphoma
206. E. Nocardiosis
207.
208. Patient with weakness of the left upper limb, weakness of interosseous
muscles and right plantar response is equivocal, left is increased; reflexes are
normal. What investigation will you do to reach diagnosis?
209. MRI cervical spine
210. Ach receptor antibodies
211. EMG
212. CT brain
213.
214. Young aboriginal male presents to you with insomnia, fear of darkness and
seeing “mamu”. He has been having these symptoms after the death of his
mother. Which of the following should be next step in his treatment?
215. A-give him benzodiazepine
216. B-consult to aboriginal health worker
217. C-urine drug screen
218. D-antipsychotic
219. E-drug and alcohol abuse counseling
220.
221. Young man after a quarrel had a fracture of floor of eye what is the most
consistent symptom with that?
222. a-Conjunctival haemorrhage
223. b-loss of visual activity
224. c-anaesthesia around the cheek
225. d-Cant open the mouth completely
226. e- Epistaxis
227.
228.
229. Father with Huntington chorea, daughter carrier, she wants to check her 10-
year-old daughter?
230. Don’t do genetic testing
231. Counselling the 10-year-old daughter
232. Do the test for the 10-year-old daughter with her mother
233. Do genetic test for the daughter
234. Genetic counselling
235.
236. Alcoholic patient present with ascites, gynecomastia, spider nevi, parotid
gland enlargement and flapping tremors and confusion (not mention about
abdominal pain).Vitals given. What investigation to do?
237. Ammonia level
238. Full blood examination
239. Liver function test
240. Abdominal scan
241. Abdominal paracentesis
242.
243.
244. post operativept, oliguric with indwelling catheter, fever 38.5, what is ur
most appropriate management ?
245. 1.abd x ray
246. 2. blood culture
247. 3.serum electrolytes and Cr
248. 4-abd us
249.
250.
251.
252. young girl Came with menorrhagia. Absent from work because of menses
pain & heavy flow.
253. From history &Xm, cannot find cause. You ordered several lab
investigations but she insists to give medication. Her cycles were regular
254. A- OCP
255. B- Oestrogen patches
256. C- Mefenamic acid during period
257. D-tranexamic acid during period
258.
259. Couple come for infertility problem for the last 12 months. On testing
examination and tests of the female are unremarkable. Male has azospermia. And
bilateral absence of vas deferens. Which of the following is most important test
before the starting the treatment of infertility?
260. A. No testing required as they cannot have a child
261. B. Testing of both male and female for cystic fibrosis
262. C. Refer for IVF
263. D. Serum FSH and LH for male
264. E. Serum Testosterone level
265.
266. A man who is intubated and sedated in ICU after operation of
cholecystectomy. You are called to witness for the will he previous signed. What
will you do?
267. Refuse to witness
268. sign in front of lawyer
269. compare the signs from his driving license
270. inform your medical defense
271.
272. A 30 years old woman with increasing dyspnea, dry cough over few months
with painful lumpy skin lesions on legs and shins. On examination, there are red,
painful, lumpy lesions in leg and bilateral ankle swelling. ACE level increased.
Which of the following is the best next step to get diagnosis? (No X-ray given)
273. A. CT scan chest
274. B. Skin biopsy
275. C. Blood culture
276. D. Aspiration
277. E. ANA
278.
279.
280. A sudanese boy had sex with his girlfriend. after a few days he found
urethral discharge from his penis. He has other sexual partners. what specimen will
help you make a diagnosis of the STI
281. a.1st catch urine PCR
282. b. Mid stream urine pcr
283. c. urine culture
284. d. Urethral swab gram stain and culture
285.
286. patient with history of claudication, smoke 30 pack cigarette per day, drink
alcohol, obese with diabetic history. he refuse surgery although he was fit for it. He
asked you for the appropriate advice that will improve his symptoms of
claudication?
287. A-Reduce smoking
288. B-reduce alcohol drinking
289. C-supervised exercise
290. D-control his hypercholesterolemia
291.
292. h/o DVT HTN with DM patient on Warfarin, metformin, Statin suddenly got
swelling at right thigh for 2 days and become painful and got fever 38. Right thigh
size is 4 cm larger than left. Diagnosis?
293. A)Rhabdomylosis
294. B)Haematoma
295. C)DVT
296. D)Cellulitis
297. E)drug reaction
298.
299. 8yr old child brought by his mother , he complained from episode of staring
suddenly that occur along with fidgeting of right hand and movement of right arm
head twitching to right side, sometimes chewing and lip smacking each episode
last for 60-90 second then the child remain dizzy and confused for 1-2 minutes
after the episode . These occurs in 3-4 days and then the child back to his normal
activity and behaviour for several weeks. what is the most likely diagnosis?
300. A- Temporal lobe epilepsy
301. B- Juvenile myoclonic epilepsy
302. C- Absence seizure
303. D- other don’t remember
304.
305. Child (3-4 year) with fever 39C and history of cough unwell for 3 days. I
think he has wheeze but minimal lung signs on exam. CXR given. Asked which
investigation wud u do for further assisting in diagnosis?
306. pneumococcal PCR
307. pleural aspirate
308. blood culture
309. FBC
310. A woman comes to your clinic. She was prescribed trifluoperazine for her
condition. She was taking it for 3 years with improvement of her condition. She
says that she discontinued taking her medication for the last 3 weeks because her
doctor was not present for he was in trip, she also said that trifluoperazine makes
her hand or some muscle part stiffy, rigid, and restless. Now she presents with
voices in her head. What is the most appropriate initial choice in management?
311. A-trifluoperazine
312. B-resperidone
313. C-quetiapine
314. D-respa depot
315. E-stop trifluoperazine
316. A case of galactorrhea patient on amisulpride.Very long question. What
other drug to counteract the side effects?
317. A. Risperidone
318. B. Aripiprazole
319. C. Olanzapine
320. D. Clozapine
321. E. Mirtazapine
322. 55 year old patient, came with severe pain at L4 and L5. Have a history of
weight loss 6kg in 2 months. The pain travels down to the legs. No neurological
deficit present. Asking for the most important cause for investigating this patient?
323. Radicular pain
324. First ever episode
325. No history of trauma or strain
326. Weight loss
327.
328.
329. 58 year old woman after lifting some heavy thing, develops pain in the right
buttock and tenderness in her back at L4-5, has difficulty and restrictions of
movement in extension and flexion and rotations , what will you give beside
analgesic?
330. A- spinalxray
331. B- spinalmri
332. C- bed rest
333. D- referral to orthopedics
334. E- keep active
335.
336. 40yr old m gardening. Pain L4/L5 , pain radiation to buttock .. noneuro
deficit.
337. X-ray lumbosacral spine
338. Ct spine
339. MRI spine
340. Observation
341.
342. Woman, 42 yrs, comes complaining of back pain, pointing towards the
L4/L5 area. On exam the area is tender and she has difficulty in flexion and
extension. The woman has hx. of breast ca, 10 yrs back that went into remission
and now she is disease free. She describes she got this pain while she was doing
gardening. What should be you next investigation in management of this pt?
343. 1- MRI lumbosacral spine
344. 2- Xray lumbosacral spine
345. 3- Continue activity
346. 4- Physiotherapy
347.
348.
349. Middle age man WITH pain in buttock and thigh during 100 m walk on
ground. He can walk 20 m uphill but his femoral pulses are not palpable however
his dorsalispedis is palpable. which appropriate investigation will you request
350. Arterial Doppler
351. Digital subtraction arteriography
352. CT angiogram
353. d. Femoral ultrasound scan
354.
355. Old lady 58yrs with urinary incontinence, when she makes some stress such
as cough, urine will flow out. On examination she has only rectocele, no other
cystocele. Did urodynamic study and the urine avoids during low detrusor pressure
and advised to do pelvic floor exercise but still incontinence and embarrassed. She
is not satisfied. She had DM and well controlled with diet. What is the next most
appropriate treatment in this patient?
356. A. Weighted vaginal cone
357. B. Bladder neck suspension (exact words)
358. C. Imipramine
359. D. Oxybutyline
360. E. Surgery for rectocele
361. An old man has been taking many medications for some kind of cancer
pain. His son found him unconscious in the garden and call the ambulance. Along
the way, the patient was stabilized and the patient was given naloxone. During the
examination, no nuerological deficit but the patient had pinpoint pupils.His son
didn’t know what he is taking but brought all the medication containers. The
medications in it were methadone, oxycodone, nicotine patch, paracetamol and
another drug (I don’t remember). Which of the following drug causes this
presentation?
362. A-methadone
363. B-oxycodone
364. C-nicotine patch
365. d.paracetamol
366.
367. 74 years old man brought from resident home, he is masturbating near
nurses’ station, tried to cuddle & kiss the nurses while bathing him over 2mth.
History of Alzheimer's and taking donepezil & other medicines. What is the
diagnosis?
368. A. Worsening Alzheimer's
369. B. Side effect of donepezil
370. C. Side effect of mimentime
371. D. Frontal lobe SOL (NOT infarct)
372. Ct- fractured ribs
373.
374. Patient with weakness and lethargy. Following investigation shows T4
decreased, TSH normal. (that’s all scenario).
375. A. MRI brain
376. B. Thyroid scan
377. C. Give thyroxine 25 mcg
378. D. Thyroid autoantibodies
379. E. FNAC
380. A man is having a check up for prostate cancer last two years PSA was
1.5mg/ml, now PSA is 3.8mg/ml, then 12 slides of biopsy done and only one foci
showed adenocarcinoma. Gleason score is 4. How will you manage this patient?
381. A.TURP
382. B.radical prostatectomy
383. C.active surveillance
384. D.EBRT
385. E. Androgen therapy
386.
387. A child with left kidney smaller than the right, what is the best
measurement for renal function?
388. a. DMSA
389. b. DTPA
390. c. IV urogram
391. d. USG
392. A mother come with her 8 months old child complaint of continuous
bleeding after fall from coffee table. On examination there is bleeding frenulum
and some old bruise in forehead and leg. There was no petechiae,
lymphadenopathy, hepatomegaly. What is the cause?
393. A. ITP
394. B. Non accidental injury
395. C. VWD
396. D. Hemophilia A
397. E. Hemophilia B
398.
399. Restless leg syndrome scenario. Investigation asked:
400. a. iron studies
401. b. sleep studies
402. c. nerve conduction test
403.
404. . 24mth old child :
405. A- knows name of color
406. B- 2 pronouns
407. C- Full sentence
408. D- Family name
409.
410. Woman with a history of hysterectomy and DVT.Now complaining of hot
flushes and insomnia. What will you give her?
411. A. Oral low dose estradiol
412. B. Transdermal estradiol
413. C. Progesterone only pills
414. D. HRT
415. E. Clonidine
416. A 52 year old woman had her last menstruation 8 months back. She is
complaining of hot flushes and insomnia. What is the best treatment option for
her?
417. A. Continuous HRT
418. B. Cyclical estrogen with 12 days Progesterone
419. C. OCP
420. D. Estrogen patches
421. E. Continuous Progesterone for 30 day
422.
423. A young poor Indian girl going on a low-cost travelling agenda.Past history
of receiving polio, MMR vaccines at a young age. What vaccine are you going to
give her before she embark on her trip?
424. A. Polio
425. B. MMR
426. C. Polio, MMR, dTpa
427. D. Polio, MMR
428. E. dTpa
429.
430. 35 year old pregnant woman presented at 15 weeks gestation asking you
for a diagnostic screening of down, what is the diagnostic test you should do for
this woman?
431. A-nuchal translucency ultrasound
432. B-maternal serum and ultrasound
433. C-maternal serum
434. D-amniocentesis
435. E-chorionic villous sampling
436.
437. A 25 years old primigravida presents for results of antenatal screening
tests. She is a drug injector in the past. But has not get any drugs in the last 5 years.
Her results show Hep C Ab +ve, Hep C RNA –ve, HIV –ve, Hep B Ab +ve. Which of
the following will reduce the risk of transmission of Hep C to her baby?
438. A. LSCS at term
439. B. Formula feeding of newborn
440. C. Avoid pH monitoring of fetal scalp electrodes**
441. D. Anti-retro viral therapy
442.
443.
444. And other are one aspirin 100% increase in relative risk,
445. 2 min and 4 min stop synto,
446. One new questions about the 16 years teenage boy with negative
psychosis symptoms,
447. Fibula # scenario,
448. Ct brain infarction scenario,
449. I got 2 ecgs,one is atrial fibrillation and other is hyperkalemia,
450. Duputren contracture pic ,most common by alcohol
451. Anorexia nervosa ,feeling of inferiority and one borderline scenario ,mx
dialectal therapy,
452. Eye pic conjunctivitis,chloramphenicol eye drops,
453. One sarcoidosisscenario,skin biopsy for dx,
454. Venous ulcer mx
455. Hep c mother with baby screen at 18 months options other irrelevant,
456. Ankylosisspondylosis treatment med 10 cm sulfasalazine
457. This much I can remember now,I did last 6 months recalls and its really help
me in my exam ,time is enough so no worries for that ,just need to be calm down
and read whole scenario properly and then optios and choose right one,
458. All the best to all amccandidates,goodluck,thank you .

RECALLS APRIL 2018

#Trial Q

1. Melanoma eye pic --- Refer to Plastic Surgeon


Exactly this pic

2. baby after 6 hours of delivery develops jaundice. Asking Cause


A) Neonatal sepsis

B) Hemolytic anemia

C) Beast feeding

1. 2 year old child Respiratory rate 30. Heart rate 120 Fever 39. one day history of fever,
vomiting, abdominal pain. On examination mild abdominal tenderness but no rigidity.
Chest xray is given ( looks like pneumonia). Which of the following is the most appropriate
initial antibiotic?
a. Amoxicillin
b. Roxithromycin
c. Augmentin oral
d. IV penicillin
e. IV ceftriaxone

2. Recall of the young lady whose child is 10 days and she comes with him with complain
that he cries excessively and is difficult to handle. She has established breastfeeding. On
examination, child is well and on 90th percentile for weight and height. Which of the
following is essential to establish her current problem?
a. Her premorbid personality
b. Thoughts about harming the child
c. How she is managing her life with the child
3. Man on multiple drugs including ramipril which he was taking for 2 years. Thiazide
which he was taking for 3 years, developed a swelling in tounge with difficulty breathing.
The swelling started yesterday. He had started taking amoxicillin 3 days ago. Which of the
following is responsible for the man’s condition?
a. ACEI
b. Thiazides
c. Amoxicillin

4. Lady full term with adequate contraction was monitored on CTG. Everything was
progessing normally. Suddenly there was a deceleration of FHR dropping to 70bpm for 4
mins. What is the next most appropriate step?
a. Stop syntocinon
b. Continue synto at an increased dose
c. Immediate cessarean
d. Ventouse delivery

5. Lady with a term labour. Contractions are good and fetus is well on CTG. Suddenly
heart rate drops to 70bpm on CTG for 2 mins. What is the next most appropriate action?
a. Stop syntocinon
b. Continue synto at an increased dose
c. Immediate cessarean
d. Ventouse delivery

6. Lady primigravida HCV +ve comes to discuss risk to fetus during delivery. What is the
most important thing to tell her?
a. Vaginal delivery is contraindicated
b. She should not breastfeed her baby
c. Fetus will be checked for HCV at birth
d. Fetal scalp sampling should be avoided

7. Pic of Hutchison melanoma in an old Australian male. What is the most appropriate
action?

a. local excision
b. Excision with 2cm margin
c. Topical podophyllin
d. Topical imiquimod

8.Scenario of a girl with interpersonal conflicts, severe anger management issues, Fights
and argues with father. Which of the following maybe helpful?

a. Involve father to the therapy


b. Family therapy
c. Dialectical therapy
d. CBT

9.Young mother brings her 8 month old baby saying that he has been bleeding a lot after
he bumped his head on a table yesterday. On examination you notice the frenulum is torn.
You also notice multiple burises on the babys head and shins. There are no other findings
of hepatomegaly or petechias. Which of the following is the most likely cause of the babys
condition?

a. Non accidental injury


b. Von willibrand disease

10. Young woman 26 years of age comes to you complaining of heavy menstrual bleeding
with pain. She describes that clots of blood are lost in the first few days of menses with
severe back pain. You ask her for some investigations that need to be performed. She gets
irriated and says she just wants a treatment. Which of the following is the apporpraite next
treatment ?
a. Tranexemic acid from day 1 of menses
b. Mefanimic acid from day 1 of menses
c. Mirena
d. Oral contraceptive pills

11. 48 year old woman.complains of heavy menstrual bleeding for last 4 months. She
underwent a hysteroscopy and D&C but the symptoms did not subside. What is the most
appropriate management in this case. Her haemoglobin is 8.5 g/dl
a. Norethiesterone day 5 to 25 day of cycle.
b. Levonorgestel containing implant
c. OCPs
d. Tranexemic acid …..if IUcd then it will be

12.Man chronic smoker. He smokes 20 cigarettes a day and drinks 8-10 strandard drinks
of alcohol. Now presents with increasing dyspnea. On cardiac examination no notable
finding. Chest examination shows hyperresonant lung percussion and some crackles on
both lung bases. O2 saturation is 85%. Otherwise the patient is not in distress. What next to
do ? (chest xray is given – no notable findings)
a. Give him salbutamol
b. prescribe him beta blockers
c. Do ABGS
d. Do a ct chest

13. Woman 51 years of age presents after menopause for the last 8 months. She complains
of irritability, moodiness, low libido and hot flushes. Which of the following would be most
appropriate to give to this woman?
a. Combined continuous estrogen and progesterone throughout the cycle
b. Continuous estrogen throughout the cycle
c. Continuous estrogen with progesterone on day 12- 25 of the cycle
d. Continuous progesterone with testosterone

14. Couple come for infertility problem for the last 12 months. On testing examination and
tests of the female are unremarkable. Male has azospermia. And bilateral absence of vas
deferens. Which of the following is most appropriate before the starting the treatment of
infertility?
a. No testing required as they can not have a child
b. Testing of both male and female for cystic fibrosis (same wording , no option of testing
the male only)
c. Refer for IVF

15. Young boy after UTI presents for followup. On U/S one kidney is smaller than the
other. Which of the following is the best test to determine the function of kidney?
a. DMSA
b. DMPA
c. IVP
d. Cystourethrography
e. Micturating cystourethrogram

DTPA

16. Young woman presents with heavy menstrual bleeding. She is pale, otherwise in no
acute distress. On investigations her Hb is 8.5. Ca is 1.9 (normal is 2.1-2.5) and INR is 1.6.
Which of the following should be done before further investigations for cause?
a. Blood transfusion
b. IV calcium infusion
c. Vit K
d.haemolytic screen

17. Woman comes to you with vaccinations regarding a low self budget trip. She says she
had taken one oral polio , 1 dose of MMR , 2 doses of DPT. What will you consider giving
this patient?
a. MMR
b. Tetanus
c. MMR and diphtheria and tetanus
d. Polio, MMR, Diptheria and tetanus

18. Young child brought to you who missed her vaccination dose of 4 and 6 months due to
urti and fever. Now she has low fever and mild nasal discharge. Which of the following is
most appropriate management?
a. Delay the vaccination
b. Give the vaccines and follow a catch up schedule
c. Start the vaccines for this age group and drop the missed ones

19. Old man 75 years of age has 3 week history of constipation. He presents with mild
abdominal tenderness and on rectal examination there is no faeces in the rectum. On CT
there is cut off at the sigmoid colon and dilation of the ascending, transverse and
descending colon. Which of the following is the most likely diagnosis? (no ct given)
a. Sigmoid volvulus
b. Colon cancer
c. Caecal volvulus

20. 60 year old male presents to you with difficulty in micturation and nocturia. On DRE
you found the prostate to be irregular and hard. His PSA is ordered and Gleason 4
.prostatic cancer is diagnosed. Which of the following is the most appropriate
management?
a. TURP
b. External beam radiotherapy
c. Radical prostatectomy
D-brachytherapy
E-watchful waiting

New ques

21. A woman brings her husband after prostatectomy saying that he has been getting
extremely angry and agitated on most trivial things after the operation. Which of the
following questions will help you identify an immediate need to hospitalize this patient?
a. History of drug abuse
b. History of domestic violence
c. History of self harm

D. Worsening of symptoms at night


22. Recall of a woman whose son beats him and is a drug dealer. Which of the following
advice os helpful regarding this woman’s safety?
a. Tell her you are obliged to tell the police
b. Ask her to go to woman refuge center
c. tell her she must inform the police

23. A man comes to the ER with sudden severe chest pain which he developed after an
acute bout of vomiting. On examination there is dullness to percussion on the left lower
lobe. Which of the following will help you identify the diagnosois?
a. Gastrograffin swallow
b. CT scan non contrast
c. oesophagoscopy

24. A man in brought to the ER after he recived a kick to the side of his face. On
examination you have found an orbital floor fracture. Which of the following is the
accurate predictor of this diagnosis?
a. Inablity to open the mouth
b. Subconjunctival haemorrhage
c. Anesthesia on cheek

25. 1 week old infant is brought to you with bile stained vomiting. The child has a history
of passing meconium on day 4 of birth. What is the appropriate diagnosis?
a. Meconium ileus
b. Duodenal atresia
c. Hirschprung disease
d. Mid gut volvulus

26. An old man is brought to you with bradykinesia and akathesia. He is on respiridal for
his visual hallucinations. Now he is complaining of fluctuating forgetfulness. What is the
likely diagnosis?
a. Parkinson disease
b. Alzheimer disease
c. Acute delirium
d. Lewy body dementia

27. Young indigenous male presents to you with insomnia, fear of darkness and seeing
“mamu” . He has been having these symptoms after the death of his mother. Which of the
following should be next step in his treatment?
a. Give him benzodiazepine
b. Counselling with an indigenous councellor
c. antipsychotic agents
28. Biostat, out of 100 volunteers each group
Coronary event No coronary event
Aspirin 1 99
No aspirin 2 98
How much the relative risk increase in people not taking ASA to get coronary event?
A- 1%
B- 2%
C- 100%
D- 200%

29.X-ray of scaphoid fracture. Asking most appropriate management?


a. Apply crepe bandage and early mobilization
b. Screw fixation
c. slab and early mobilization
d. Plaster cast

30. Australian farmer with painless mass on the right groin. On biopsy it shows squamous
cell cancer. What is the likely site of primary cancer?
a. Leg
b. Penis
c. anus
d. Testes

31.A man presents following sudden onset of pain and tenderness of lower back following
lifting a heavy weight. There is tenderness in the L4/L5 region and numbness of the leg.
Which of the following is the next option in treating this patient?
a. MRI
b. Prescribe analgesia and continue daily activities
c. lumbar corset
d. Pelvic traction
e. Bed rest and analgesia

32. 24 month old child can do which of the following?


a. Can speak 2 pronouns
b. Can make 4 word sentences
c. knows family name
d. Name 4 colors

33. Elderly lady, presented with confusion, dizziness. She was on vacation for 3 months, her meds
got over so didn’t take it for last 2 weeks. Was on multiple drugs likeIndapamide , perindopril &
some more….. Labs given. Na: 127, K : 5.2, Glucose :Low Normal . What is the cause of her
presentation?(Question was the drug causing hyponatremia)

A. Dehydration
B. Indapamide ( diuretics )
C. Hypocalcemia
D. Hypoglycemia
34. 35 yrs old male pt, known case of Schizophrenia, well controlled on meds since last 15 yrs,
admitted due to some injury, needs operation. But pt refused. What is the appropriate step?

A. Take informed consent


B. Autonomy
C. Go ahead with the surgery

35. 81 yrs old female wants to change her will, MMSE was 20 or 24, but daughter
told her that she cant change due to her memory impairment. Dr. checked her
MME, showed mild memory impairment, same as before, No changes in MME.
What will you advise her?
A. You can change the will
B. you should speak to your lawyer about it
C. You can’t change the will
D. Refer to Neurophysiologist/ Neurologist for review

36. Young man about 30-34yrs old, presented with this condition as shown in the
pic, he is a smoker, drinks alcohol daily, on ranitidine for dyspepsia. O/E parotid
enlargement, Testicular swelling U/L – can’t get above the swelling. What is the
most likely diagnosis?

Similar pic
A. Teratoma
B. Leydig cell tumour
C. Alcoholic liver disease
D. Ranitidine


37. Lady working in a daycare center, 7 weeks pregnant. Exposed to rubella child.
She was fine, but after researching on internet, she is worried now. What to
advice her?
a. Do IgG Ab Rubella
b. Detailed h/o exposure to rubella
c. Gv Rubella Vaccine
( No option for reassure )
38. There was another case where young female was given rubella vaccine 1 week
before her menses .She found her missed menses and found pregnancy test
positive.What should be done:

a. Abort
b. Test for antidodies
c. There were other choices don’t remember others

39. 55 yrs old male, K/C/O Hep C, not using IV Drugs since last 5 yrs. Alcoholic. On
presentation, he is drowsy but answers /obeys commands. O/E gynecomastia,
flapping tremors, fever 38’C, mild ascites, spider nevi, parotid gland
enlargement. What is the most initial inv?
A. FBC
B. Ammonia
C. Abd. Paracentesis
D. LFT

40. 80 year old man presented with loin pain on the right side, urine examination showed
hematuria. What is the most appropriate next step?
A. X-Ray
B. Abd ultrasound
C. CT Abdomen
D. Right retrograde pyelography
E. Urine culture

41.Lady brought her husband, c/o pain in the right side of tongue since few hrs. He is on
Ramipril & other drugs. Recently started on Amox for some infection. O/E tongue sis
protruding out, swollen, when he tries to speak, makes noises ( not able to speak properly )
& points towards the tongue. What is the cause of his presentation?

a. Ramipril
b. Amox
c. Hydrochlorothiazide
d. Metformin
e. Vitamin C tabs

42. a man presents with c/o left leg pain, can walk upto 100 meters , due to pain has
to rest for sometime. relieved by rest, on examinations there was , right leg good
peripheral pulses, left leg weak pulses , ABI done and it was 0.25. what is the
most appropriate test leading you to the diagnosis?
A. Arteriography
B. Ct angiography
C. compression Doppler ultrasound
D. MRI
E. X-ray

43. 65 years woman after death of her husband due to prostate cancer 6 weeks back,
is brought to you. She lives alone , presented with loss of appetite, thinks he died
because of her infidelity. She sleeps badly and had similar episodes after her
miscarriage at age 23. What to give for Prompt relief.
A. Temazepam
B. Venlafaxine
C. ECT
D. Mirtazapine
E. Escitalopram

44. Young boy, post MVA, c/o pain, O2 sat 96%, other vitals normal, CT given,
showed mild Hemothorax. After giving O2, What next?

Similar pic
A. Morphine
B. Intubate
C. chest drain
D. Observe
E. needle thoracostomy


45. Patient with weakness and lethargy. Following investigation shows T4 decreased,
TSH normal.
1. MRI brain
2. Thyroid scan
3. Give thyroxine 25 microgram
4. Thyroid autoantibodies
5. FNAC

There were 4 to 5 questions on decreased urine output.Clearly do not remember


the scenarios and choices ,however they were only two concepts checked BPH
and post op oliguria.

46. Oliguria in pt post surgery, no mention of FC, IVF given =


Post op oliguria in a patient with cholecystectomy 12 hrs before. fever 37.9, BP
and pulse normal, fluid running at 80 ml/hr but urine output only 100 ml. no
mention of catheterization. what to do next:
a. bladder scan
b. Ct scan
c. Serum urea creatinine

47. Oliguria in pt post surgery with FC, IVF given = S/Urea, Creat

48. Oliguria in pt post surgery with FC, IVF given, asso with diffuse abdominal
tenderness = USG Abd
Post laparotomy opt pt oliguria with indwelling catheter.fever 38.5.There was
generalized abdominal pain. most appropriate mx?
A. And xray
B.blood culture
C. S electrolyte and creatinine
D. abd usg

49. Man with urinary retention mass two fingers above the pubic symphysis. DRE
showed enlarged prostate with smooth median sulcus palpable. Catheter was
inserted. What is the most appropriate next step?
A. Urine Cytology
B. PSA
C. bladder scan
D. CT
E. S/Creatinine

50. Man with urinary above retention mass two fingers the pubic symphysis. DRE
showed enlarged prostate with smooth median sulcus palpable. Catheter was
inserted. What is the most appropriate next step?
A. Urine Cytology
B. PSA
C. TRUS
D. CT
E. S/Creatinine( No option for Bladder scan )

51. 8 days old infant presented to u complained with poor feeding and bile stain
vomiting his birth was at 38 weeks gestation and weight 2600g he is breast
feeding with no immediate post natal complications. The child has history of
Meconium passing on day 4th of birth now he has mild jaundice and abdominal
distension and is irritable on examination abdomen distended and tender what is
appropriate diagnosis?
A) Meconium Ileus

B) Hirschsprungs disease

C) duodenal atresia
D) necrotizing enterocolitis
E) volvulus
52. 12-year-old girl at foster care. she's having difficult time in school in reading and
writing. She has a collection of her own toys and plays with them with her
imaginary friend. Her foster carers are irritated because she's picky at food. what
immediate danger for her?
A. OCD
B. Sexual abuse
C. Drug abuse
D. Anorexia nervosa
E. Schizophreniform psychosis

There were two questions on CTG ,one was to stop synto and other was to
start oxygen.

53. 37 weeks pregnant have labour induced , synto given , cervix 8cm dilated ,
fetal head at ischial spine level , CTG show HR drops to 70/min & came back to
145/min in 4 min. Asking next ?
A. Cease Syntocinon
B. Fetal scalp pH
C. CS
D. Ventous Delivery
E. O2 to mother

54. - 4mins Q- Both Q only scenario, no CTG

?weeks pregnant lady came with labour pain. she was put in left lateral position
having oxygen mask in place. iv fluids with syntocinon is running. Ctg was done
which shows heart rate of 140 which dropped to 70 and came back to 140 in 4
min. asking next appropriate treatment.
A. Fetal scalp sampling
B. Continuuous ctg monitoring
C. Stop syntocinon
D. C section

55. #Hernia pic from Xiphisternum to Umbilicus-


Huge epigastric hernia picture of a 56 year old man which extends from
umbilicus to xiphisternum. He looked morbidly obese and had a waist
circumference of 110 cm Asking for appropriate management for the patient.
A. Weight loss therapy
B. Abdominal binder
C. Herniorraphy with mesh repair
D. Hernioplasty
E. Observation and Reassurance

similar pic
56. Same question as above however patient was told to be overweight.
Hernia- 110 cms waist circumference, wat advice-.

A. Weight loss therapy


B. Abdominal binder
C. Herniorraphy with mesh repair
D. Hernioplasty
E. Observation and Reassurance

57. LBP 58yrs, localized pain- Continue activity


58. LBP 40 yrs with pain radiating to legs-


40 years lady with sudden severe pain while gardening. Pain is radiation to thigh
and leg with no neurological deficit what will you do as next?
A. Spine X-ray
B. MRI
C. Bed rest
D. Observation
E. Refer to orthopedic

59. Pt came with back pain & tenderness in L4 L5 region. On history it was found
that he used to be ivdrug abuser & there was history of hep c asked inv
A. cxr
B. Mri
C. Ct scan
D. Observe

60. #Lipase Inhibitor- BMI 35 Q


A lady with BMI of 35 how will u manage her in addition to exercise,weight
reduction and diet?
A. Involve in sports
B. Low carb food always
C. Lipase inhibitor
D. Diuretics
E. Surgery

61. A pharmaceutical company comes with some drugs introduced in market.


What u check the drug is efficacy and safe?
A. RCT
B. Double blind study
C. Cohort
D. Case Control
E. Case study

62. A patient with chronic lung disease develops cough and fever was admitted to
hospital. Ipratropium and salbutamol given 8 hourly. Oral prednisolone, oral
roxithromycin and IV amoxicillin given. ABGA done showing: pH 7.35, PaO2
80mmHg, PaCO2 50mmHg, HCO3- 35mmHg. What is your next management?
A. Give IV hydrocortisone
B. Increase bronchodilator to 4 hourly
C. CPAP
D. Admit to ICU
E. Change to IV Cefotaxime

63. 7 days old baby presented with jaundice since 4 days of life, his birth was at
term, not complicated and he is breast feeding, serum bilirubin level was 240
(normal <200) with 120 direct bilirubin, his liver is 1 cm palpable below costal
margin. What is the most appropriate next step? (Direct bilirubin for 120, that’s
sure)-
A. Thyroid function test
B. Abdominal ultrasound for liver and biliary tree
C. Coombs test
D. Stop breastfeeding and follow up
E. Phototherapy
A patient 23 years old who had a normal pap smear 6
64. months ago was advised to do a screening test for chlamydia. She wants to know
why or if the test is necessary?
A. Chlamydia causes infertility
B. Chlamydia is asymptomatic
C. Reassure her the test is not important because the PAP smear was negative
D. Give her a human papilloma vaccine

65. Young couple with infertility. Female with one child in previous marriage and
female investigations were normal, what in history you will ask the male patient
that will be most useful to guide u to cause of infertility?
A. how much alcohol do u consume?
B. Did u get chicken pox infection for once while u were young?
C. Do you use Marijuana?
D. ecstasy and other drug use
66. A woman comes to your clinic. She was prescribed trifluoperazine for her
condition. She was taking it for 3 years with improvement of her condition. She
says that she discontinued taking her medication for the last 3 weeks because her
doctor was not present for he was in trip, she also said that trifluoperazine makes
her hand or some muscle part stiffness, rigid, and restless.Now she presents with
voices in her head. What is the most appropriate initial choice in management?
A. Trifluoperazine
B. Risperidone
C. Quetiapine
D. Respa depot
E. Stop trifluperazine

67. 14 years old girl lives with family has now become sexually active comes to you
for contraception advice and tells you not to inform her parents. What will u do?
A. Give her prescription of contraception
B. Ask her about identity and age of her sex partner
C. Tell her she needs parents’ permission
D. Inform parents
E. give her prescription and Tell her to use condom to her partner

68. There was girl having social phobia prob reluctant to go to places came with
Friend with fear, trearful but happy at home doing painting and all other work
what in history will help us reach diagnosis-
a. School refusal
b. Night terror
c. family history
d. Alcohol abuse

69. Poorly controlled Diabetic pregnant woman with mild renal impairment asking
about the risk of effect of her diabetes on this baby?
A. IUGR
B. Macrosomia
C. Renal agenesis
D. Intrauterine fetal demise
E. low Birth weight

70. A child with fever and hx of sore throat. He developed pain in wrist later
developed arthralgia n swelling in ankle joint. skin rash present . what's the most
initial step of management ?No Murmur mentioned.
A. ESR
B. Full blood C. throat swab
D. USg
examination
E. ECG

71. #refuse to witness recall


You are intern. Patient wishes you to witness her will in the presence of lawyer.
What should you do?
A. Witness the will
B. Refuse to witness
C. Ask surgical register if he can do it
D. Seek legal advice

72. #Down synd diagnostic inv at 15 weeks asked.


15 wks pregnant comes for definitive diagnosis of Down syndrome
A. amniocentesis
B. CVS
C. nuchal fold thickness
D. serum screening

73. #check insulin at 3am (somogyi effect recall) –


girl with type 1Dm ,on intermediate insulin at night, short acting insulin in the
morning, her parents concerned about her high glucose levels of 15 at morning,
what next?
A. increase intermediate insulin at night
B. increase short acting insulin in the morning
C. prevent fruit in evening
D. measure glucose levels at 2am and 3am

74. #Ovarian Ca Scenario- No pic


Lady 50 years with constipation and back pain Mass extending from pubic
tubercle to the umbilicus it was free and present lateral to uterus not attached to
the uterus or others on examination. Diagnosis asked.
A. GSIT
B. Retroperitoneal sarcoma
C. Endometrial cancer
D. ovarian cancer

75. #Cluster Headache Q


Pt presented with left sided headache,Unilateral, in the periorbital, retro-orbital,
Suddenly started at night, peaking in 10-15 minutes lasting for 5 minutes. O/E
Ptosis. What is Imp to make diagnosis ? ( No mention of Unilateral Rhinorrhea &
Lacrimation )
A. Confirm with 100% oxygen by face mask-
B. CT Scan

76. 17yrs old girl has dyspareunia, she is worried about endometriosis because sister
has infertility because of this. Examination: nodule on ligament on vaginal
palpation and some other description, what is important to diagnose
endometriosis:
A. nodule on uterosacral ligament
B. dyspareunia
C. menorrhagia
D. family history

77. Lady comes on 10th postpartum day with pain due to perineal tear. Now on
examination there is perineal wound that is 2 cm long, 1cm wide and 1 mm deep.
It was clean just serous discharge was there. What to do next?
A. Apply local antibiotics
B. Put on oral antibiotics
C. Simply keep the wound clean
D. Suture the wound under local anaesthesia
E. Suture the wound after cutting the edges

78. A 42yo woman who smokes 20 cigarettes/d presents with complains of heavy
bleeding andprolonged menstrual period. Post D& C. What is the most
appropriate tx for her?
A. Tranexemic acid D. COCP
B. Mefenemic acid E. Levonorgestrel IUCD
C. Norethisterone

79. A 40yrs old woman on cocp for 8years. She is a smoker and her BP is high. She
doesn't want to get off pills. What is your advise?
A. Stop pills and rely on condom
B. Copper IUD
C. Implanon
D. POP
80. A man comes with clumsiness of hands and tripping over. Upper limb weakness
and fasciculation. Lower limb muscles also has weakness, increased tendon
reflexes. Ankle reflex is not increased in one side. No wasting. No sensory loss is
given. What will help to reach the diagnosis?
A. EMG
B. MRI Spine
C. CT scan
D. CSF examination
E. Cervical spine X-ray

81. A man comes with clumsiness of hands and tripping over. Upper limb weakness
and fasciculation. Lower limb muscles also has weakness, increased tendon
reflexes. Ankle reflex is not increased in one side. No wasting. No sensory loss is
given. What is the initial investigation?
A. EMG
B. MRI Spine
C. CT scan
D. CSF examination
E. Cervical spine X-ray

82. 3 years old child with fever 39 and history of cough unwell for 3 days. He has
wheeze but minimal lung signs on exam. Which investigation will you do for
further assisting in diagnosis? (Not given chest X-ray, not already give any
antibiotic in scenario.)
A. Pneumococcal PCR
B. Pleural Aspirate
C. Trans-pleural drainage
D. Blood culture
E. Full blood count

83. Patient complaining of soreness on lateral epicondyle at the end of work, the
patient attributes it due to computer mouse clicking. On examination, tenderness
on lateral epicondyle. What is the most appropriate treatment?
A. Lateral epicondylectomy
B. Finger immobilization splint
C. Lateral epicondyle immobilization splint
D. Bracing under lateral epicondyle
E. Analgesic

84. 18 year old male with bloody diarrhea, you made sigmoidoscopy which shows
10cm friable mucosa. What is the initial management?
A. Sulfasalazine
B. Rectal Corticosteroid
C. Injection Corticosteroid
D. Lopramide
E. Methotrexate

85. A patient with peptic ulcer with H. pylori positive, took triple therapy
(amoxicillin+metro+PPI) for two weeks. After 6 weeks, urea breath test still
positive after completion of course. What is the reason for this case?
A. Resistant to metronidazole
B. Resistant to amoxicillin
C. Unreliable urea Breath test
D. Improper timing of test after treatment

86. Old woman with GERD history and taking Proton pump inhibitor, T score given
-2.5 in vertebra and -2.5 in femur, Lab results were given, calcium and vitamin D
was normal. What is the best treatment?
A. Alendronate
B. HRT
C. Strontium ranelate
D. Parathyroid hormone
E. Vitamin D supplement

87. Old woman with GERD history and taking Proton pump inhibitor, T score given
-2.5 in vertebra and -2.5 in femur, Lab results were given, calcium and vitamin D
was normal. What is the best treatment?
A. Alendronate
B. HRT
C. Zolindronic Acid
D. Parathyroid hormone
E. Vitamin D supplement

88. A patient of CKD and hypertensive, patient taking ramipril, carvedilol with many
other drugs presents with confusion labs urea is 18 creatinine very elevated about
10x upper limit, raised calcium 3.1, phosphate also raised sodium 128 potassium
5.7. What is the cause of confusion in this patient?
A. Uremia
B. Hyponatremia
C. Hyperkalemia
D. Hypercalcemia
E. Hyperphosphatemia

89. #Copd with respiratory distress in ed. Sats 88% on oxygen.


Patient known case of chronic lung disease, Heavy smoker, drinks
alcohol,presents with chronic cough, dyspneic and speak in phrases. on
examination bilateral wheeze with basal crackles. what Initial investigation
should you perfrom?
A. Spirometry
B. Pefr
C. Ct chest
D. abgs
90. A young male travelled to Asia and upon arrival he developed a of cough and
mild fever as well as epigastric pain for a duration of 2 weeks, what is the likely
cause? X ray given
A. viral pneumonitis (exact words)
B. pneumonia pneumococcal
C. fungal infection
D. tuberculosis(No Mycoplasma pneumonia in options)

91. An 18 year old girl presents with epistaxis, malaise and tiredness for months. No
other symptoms. Hb is 8.5, serum ferritin is 35, INR is 1.5. Calcium is 1.9. Long
question. What to do next ?
A. Blood transfusion
B. IV calcium
C. Injection vitamin k
D. Fresh frozen plasma
E. Vitamin D tablets

92. #Restless leg syndrome scenario. Investigation asked?


A 55-year-old woman presented with discomfort in her both legs with an urge to
move for last 4 months. She remains awake at night due to creeping & crawling
sensation at night and sometimes electric current-like sensations. Relieves after
walking & massage of leg. Her current medication include paracetamol,
oxycodone and multivitamins. Blood tests show normal renal function, normal
electrolytes .What inv you do next?
A. Iron studies
B. Sleep studies
C. Nerve conduction test
D. Serum Calcium level
E. Cease oxycodone

93. Woman with osteoporosis and taking HRT for 6 years, bone density score for
spine vertebra is -1.7 and for femoral head is - 1.2. What is next? Vit: D and
calcium level are given Normal level.
A. Cease HRT
B. Continue same
C. Change HRT to Alendronate
D. Add Alendronate
E. Vitamin D and Calcium supplement

94. A patient with spiking fever, Shortness of breath and was inserted prosthetic
heart valves 3 months ago. And diagnosis as due to Staph aureus IE, and given
Flucloxacillin for treatment. The fever subsides after 2 weeks of treatment, but
SOB still persists, new diastolic murmur at the left sternal edge. What will you
do for investigation?
A. Transthoracic Echo
B. Chest xray
C. Troponin
D. Blood culture for every weekE. CT scan
95. Patient with history of DVT, HTN with DM patient on warfarin, metformin,
statin and then add amiodarone, 2 days later suddenly got swelling at right thigh
and become painful, redness and Temp 37.8. Right thigh size is 4 cm larger than
left. Diagnosis?
A. Rhabdomylosis
B. Hematoma
C. DVT
D. Cellulitis
E. Drug reaction

96. 38 years old man with renal transplant 8months back, presented with Neck
stiffness and meningitis sign start 3 days ago, nothing mentioned about
respiratory symptoms, no fever. On chest xray there is well defined round opacity
in “middle right lung” ( chest xray not given, just mentioned secenario ). What is
the cause?
A. Aspergillous
B. TB
C. Pneumocystis Pneumonia
D. Lymphoma
E. Nocardiosis

97. Recall of
You want to know the Hep C in newborn baby born to HepC positive mother.
How to investigate?
a. Antibody of baby at birth
b. Antibody of baby at 18 months
c. Antibody of baby at 12 months

98. Pt with active RA on methotrexate with chronic pyelonephritis , feels tired and
lethargic labs done showed the ff Hb 9.5 g/dl, Mch low normal, Serum iron
decreased , S. ferritin normal , Tibc decreased . What is treatment of choice
A. packed RBCs B. iv iron
C. low dose corticosteroid D. erythropoietin injection

99. 52 yr old woman whose son is a drug addict, beats her every day. She is afraid &
presents to you. What will u do for the safety of the woman?
A. tell her that she is obliged to inform the police
B. informs the police
C. urge her to go to the refugee center
D. send the son to correctional facility
E. advise female support group

100. Patient with RA she takes ibuprofen and methotrexate to control her disease,
they mention the time it was years for both of drugs, patient complains of upper
abdominal pain, and her labs are given ALT, AST, GGT, even bilirubin all were
high, which of the following cause this condition?
A. Methotrexate induced hepatitis
B. Ibuprofen induced hepatitis
C. Autoimmune hepatitis
D. Viral hepatitis
E. Alcoholic hepatitis

101. A patient with a painless neck swelling moves with swallowing. hoarseness
and difficulty in breathing at night…gradually increased over 3 weeks. O/E
smooth swelling in the anterior triangle of neck. dullness of percussion at upper
sternal border. most likely dx?
A. MND goitre
B. papillary carcinoma
C. anaplastic cancer.
D. haemorrhage in nodule of MND
E. follicular CA

102. An old woman taken many tablets, now unconscious, pinpoint pupil, his son
brought her to hospital, and also tablets she might have taken, methadone,
oxycodone, buspirone patch
a. methadone B. oxycodone C. buspirone

103. A lady presented with her baby 5 times in 2 weeks at 10 weeks following
delivery - all normal, established breastfeeding, baby growing well on 90th
percentile. What relevant info will you ask the mother?
A. Past obstetric history
B. premorbid personality
C. history of psychosis
D. Thoughts of harming her baby

104. #Not feeding frequently Q


Most Common cause of low milk production
a. Low fluid intake
b. Less adequate baby feeding
c. Baby is not put on breast feeding frequently

105. patient with history of claudication, smoke 30 pack cigarette per day, drink
alcohol , obese with diabetic history . he is not fit for surgery. He asked you for
the appropriate advice that will improve his symptoms of claudication?
A. Reduce smoking
B. reduce alcohol drinking
C. supervised exercise
D. control his hypercholesterolemia

106. #history of hysterectomy and dvt Q-


Woman with a history of hysterectomy 3yrs back, complicated with left sided
DVT. Now complaining of hot flushes and insomnia. What will you give her?
i. Oral low dose estradiol
ii. Transdermal estradiol
iii. Progestogen only pills
iv. Hrt
v. sertraline(paroxetine

107. #Lead level 0.72- educate mother


Mother living in mining town comes with her daughter for normal blood test. Her
daughter’s test show lead level 0.72 what will be your next appropriate step?
A. Assess IQ testing of child
B. Educate mother about environmental risks
C. Refer for chelation therapy
D. Move the family to another town
E. Urine test to confirm dx

108. patient taking amisulpride, complaints of gynaecomastia and sexual


dysfunction. What to give?
a. Aripiprazole
b. Olanzapine
c. Clozapine
d. Quetiapine

109. #Drug Rebound Headache Q


A woman works at a part-time job ( office cleaner ), and complaints of early
morning headache, frontal & bilateral, dull in character, varying in intensity, she
takes paracetamol & ibuprofen, which only cause relief for 2-3 hours. What is the
cause?
A. Drug rebound headache
B. Migraine
C. Tension headache
D. Cerebral tumor
E. Pre-menstrual headache

110. A 30 years old woman with increasing dyspnea, dry cough since 2 yrs with
painful lumpy red swelling/skin lesions on legs and shins. On examination, there
are red, painful, lumpy lesions in leg and bilateral ankle swelling. ACE level (
120 ) increased. Which inv for confirming diagnosis? (No given X-ray)
A. CT scan chest
B. Skin biopsy
C. Blood culture
D. Aspiration
E. ANA

111. Girl 16 years history of weight loss of 10 kgs, BMI 15. Her school
performance is good and recently fall in grade. What in history will help in
diagnosis?
A. Episodic fatigue
B. Sleep disturbances
C. Feeling of worthlessness
D. Feeling of inferiority
E. Idea of guilt
(No Suicidal idea in options )

112. Question on treatment of lichen planus of Vagina ?
Steroid topical as treatment choice given

113. 36 wk pregnant female with normal ANC presents with fetal bradycardia.Em
LSCS done .Neonate has still have HR of 60.Whcich antibodies contribure:

a)Anti Ro
b)Anti Sm
c)Anti cardiolipin ( anti Ro associated with neonatal lupus )

114. Young male has clear urethral discharge present.New sex partner a week back.
What test to do?
a) First urine collection for Chlamydia
b) Urethral discharge for microscopy and culture
c) Mid stream urine culture

115. Question about final treatment of angle closure glaucoma?
116. Adjustment disorder? Scenario
117. what is the best measurement for health risk in obesity?
a. BMI
b. Waist circumference
c. Waist : hip ratio
d) Body weight
e)Fat distribution

By the grace of Almighty, I sat my exam. Please keep me in your prayer. Please excuse me as I will
not be able to write all the options correctly. All the best

1. What is the probability of the affected child for Thalassaemia Minor in case of both parents ?
0.25

0.5

0.75

100

2. Need to calculate point prevalence in the year 2010 per 1000 population

Total population was 200

2010: 50

Ans : 250

3. Case scenario was there . asked to identify the personality disorder: Histrionic

4. Recall of Huntingtons disease for the 10 yrs girl who came with mother.


5. Ct scan of ischemic CVD, asked immediate next step: O2 inhalation

6. CT scan of Aortic aneurysm. Asked dx.



7. MALARIA: DX: one was rapid Diagnostic serology,

8. another one was failure of 3 serological dx, which one is next to confirm malaria? Thick and
Thin film
9. Chicken pox baby, when will she go for school? When vesicle completely dried up

10. Baby with 38 c temp with rt upper lobe consolidation with mild respiratory distress asked
drug treatment:

Inj penicillin, inj roxithromycin, oral amoxicillin for 5 days review and cxr, oral roxithromycin

11. Differenciate among meningococcus meningitis, pneumococcal meningitis, staph meningitis


( internet )

12. Differentiate between hydrocele and varicocele
13. case for ADHD., regarding drug noncompliant boy how to overcome

14. Scenario Conduct disorder.Dx


15. Accountant does not like to take medications but fails to do his daily work. Treatment..

16. OCD- nonpharmacological MX ans : CBT and exposure and response prevention for
compulsion


17. Intern admitted 10 burnt pt, but pt wants to get discharge what should be done? Ans
:discahrge

18. Mother got CA breast , daughter is not attentive for her study. During examination daughter
is fine, what need to check in history? Worries about mother’s illness.


19. Acute MI . 02 , aspirin given what is next? Streptokinase, heparin

20. Mild cystocele pt, with39 BMI does not like to do surgery, what is next appropriate advice:
weight reduction 10 kg



21. Ca sigmoid scenario.DX

22. What is the emergency case: hepatic injury, free fluid in the peritoneum, intra vascular
splenic injury


23. Femoral hernia: inv: USG, MRi, CT

24. Thyroid swelling in pregnant woman? MRI NECk, USG of neck

25. A well circumscribed mass in liver, dx: hepatoma,

26. Malignant melanoma of skin

27. Case IgA nephropathy, Good Pasteur syndrome

28. UTI, Treatment in case child: inj gentamycin, inj cephalothin, inj meropenem

29. Tonic clonic seizure treatment

30. Osteoporosis Treatment

31. DM, HTN, got warfarin and other drugs developed haematoma, what is next immediate inv:
INR

32. Triceps jerk root value,root value :c6 c7,

33. Inferior dislocation of shoulder:?loss of sensation of lateral part of upper arm

34. A 3 cm swelling in the thigh of 2 yr old child. INV: USG, FNAC, MRI jm 1227

35. Bleeding from nipple in58 yr old lady: Intraductal papilloma, intraductal carcinoma

36. A 19 yr girl having 3 sexual partner came for HPV vaccine. What to do? Give her, check HPV
DNA, Do PAP

37. A boy 2 yr strongly positive gamma interferon, mother is having TB. What to do?

38. 15% pneumothorax baby with mild res. Distress. What is next: Aspiration, Admit and review
within 24 hrs

39. A girl diabetic saying that she found HbA1C was 5 to 8 at her home by self checking. But at
hospital you found HbA1c-11. What is the cause?

40. Migraine girl cam for contraception: best for her OCP, POP, Condom
41. Menstruating woman are at risk of infection? Trichomonas, Group B streptococcus, candida

42. After 5 days of normal delivery sudden passage of clots? Cause: endometritis, retained
product, cervical tear

43. Woman 14 weeks of pregnancy was eagerly want to do check for Down Syndrome. INv:
Amniocentesis, maternal serum for triple test

44. patient has elective surgery he was on many drugs.


Amitryptilline,metformine ,iboprofine,statins .asked which drug have to stop it
before one week before operation
45. a.metformin
46. b.statin
47. c.amitriptiline
d.iboprofine
48.
49.
50. 2.photo given for chickenpox. asked when child can come back to school
51. a. after course of ab
52. b.after resolution of all blisters
53. c,after 2 days
54. d.after one week
55.
56.
57. 4. days old baby presented with jaundice since 4 days of life, his birth was at term, not
complicated and he is breast feeding, serum bilirubin level was 280 (normal <200) with 06
direct bilirubin.What is the most appropriate next step
58. A-thyroid function test
59. B-abdominal ultrasound for liver and biliary tree
60. C-coombs test
61. D-stop breast-feeding and follow up
62. E-phototherapy
63.
64. 5.a breast feeding woman presented complained that her baby remain hunger and her
breasts after feeding remain full, what is the relevant behind this
65. A-putting the baby on the breast insufficient frequently
66. B-putting the baby few time on breast each feeding
67. C-poor positioning
68.
69. 6
70. History of appendectomy & cholecystectomy presented with 3 week history of abdomen
pain, distension, Bowel Sound exaggerated, CT abdomen image given. Asking the cause
71. A- Sigmoid volvulous
72. B- Adhesive IO
73. C- CA Sigmoid
74. D- Pseudo obstruction
75.
76. 7.
77. A patient has taken lots of tablets of venlafaxine. Was on depression treatment. One year
back she was given psychotherapy as she had problem at work (shout at her coworkers). She
cant relax at home after work and insomnia. After stabilization of the patient, what should
be added?
78. A. Lorazepam
b.respiridone
79. C. Mirtazapine
80. D.Haloperidol
81. E. olanzapine
82.
83. 8.case with sigmoid carcinoma
84.
85. 9. 8year old girl came to emergency department with blood suger 2.1
86. what is the most appropriate next step rch
87. a. iv dextrose
88. b iv glucagone
89. c.im glucagone==home and ambulance
90. d.phention iv
91. e.diazepam




92.
93.
94. 10. a girl had some superficial wrist injuries. She is normal without suicidal thoughts .her
parents separated last 2 years and she stay with her mom.but also she wants to move with
her father to live with him.who you will inform first ?
95. a. mother
96. .b.father
97. c.grandmother
98. .both parents=

minor pt we should involve both parents
99.
100. 11. a woman her age 24 years old came to you after 12 days of postpartum
101. she is asking you about suitble cocp for her.as she is planning to get pregant again
next year .what is your advice for her
102. a. give her cocp
103. b. give low dose of pop
104. c. mirena
105. d.dont remember
should start pop from day 21
106.
107.
108. I got 5 questions about drugs interactions
109.
110.
111. got mammography for 65 years old woman. she had a trauma in her breast last 10
years . She has no discharge .no family hx for carcinoma of breast. what is your dx
112. a. calcinosis adenosis
113. b. adenocarcinoma
114. c.ductactesia
115.
116. 14 .64 years old man with PSA 1.5, after I think 2 months time PSA becomes 3.0
(n<4.0). On DRE, prostate is enlarged and regular. On USG-guided biopsy of 12 slides only 1
locus pathology showed adenocarcinoma of Gleason 4 stage. What is the management?
A Radical prostatectomy
B. Continue surveillance
C.Transurethral prostatectomy
D.Radiotherapy
E. EBRT
117.
118. 15 50 years old lady , otherwise well. Coming with presenting complaint of palpable
mass slight to right away from midline. Not attached to peritoneum .
a. GIST
b. Retroperitoneal Sarcoma
c. Ovarian Carcinoma
d. Endometrial Ca
119.
120. a 20 yeals old girl she has had three partners before .came to gp asking him about
hpv vacine . Her last smear was normal
121. .what is your next step
122. a.give hpv as she requested
123. b.do stds screen
124. c.repeat smear
125.
126. 17.patient came to ed. he has accident crush injury
127. asked what is the most appropriate indication for laparotomy
128. a.gas in retroperitoneal cavity
129. b.free blood in peritouneum cavity
130. c.10 c hematoma in the right hypochndrium
131. d.something was about spleen also
132. i got this question 3 times in my exam
133.
134.
135. 18. Old patient with ischiorectal abscess, has this problem recurrent many times,
what’s the cause?
136. A. Anal fistula
137. B. Diverticular disease
138. C. Crohn’s disease
139. D. Diabetic
140.
141.
142.
143.
144. 20. A man aged 68 years. his psa is 3.8ng now. last 2 year(sure) it was 1.5.he did 12
biopsies this time and one showed a focus of adenocarcinoma and Gleason score 4. he had
operation for cardiac stent placement 18 months back & on antiplatelet since then. What is
the management for this man?
145. A) radical prostatectomy
146. B) external beam radiotherapy
147. C) orchidectomy
148. continued surveillance
149.
150.
151. 21. Child 6 years old with history of asthma presented with upper respiratory tract
infection, urine examination showed very high serum glucose and ketone body of 2+.
152. What is the most appropriate test to follow up this child?
153. A. HbA1C
154. B. Serum creatinine and electrolyte
155. C. FBS
156. OGTT
157.
158. 22. 5 yr old child with lacerated wound. Receives DTPa at 2 month and 4 month after
that no booster. What to give him now ?
159. a. TT or TIG
160. b. DTPa now and booster after two months
161. c. TT + topical antibiotic ointment
162. d. DTPa and TIG
163.
164.
165. 23.You are intern. Patient wishes you to witness
166. her will in the presence of lawyer.
167. What should you do?
168. 1. Witness the will
169. 2. Refuse to witness
170. 3. Ask surgical register if he can do it
171. 4. Seek legal advice
172.
173. 24. 50 year old man present with hypertension with asthma and reflux
nephropathy. lab inv were given. There was high urea,high creatinine and proteinuria 900
mg/day. What is the choice of anti HTN?
174. 1.amlodipine
175. 2.losartan
176. 3.perindropil
177. 4.indapamide
178. 5.metoprolol
179.
180. 25. Intern at the hospital. He knows about all the cases of needle stick injury in the.
All cases in the hospital were manage correctly with no big problem. Now there is
accreditation and the intern was asked about this issue. What the intern should answer:
1. Give info about the protocol of management
2. Leave the answer to your supervisor
3. Tell a case that he knows
4. Say that there’s no problem with needle stick injuries in this hospital
181.
182.
183. 26. A man with dyspnoea and SOB , taking metformin and rosiglitazone for diabetes,
amiodarone for recurrent ventricular tachyarrhythmias, enalapril and metoprolol. Which
drug may have caused these symptoms?
A. Amiodarone
B. Metformin
C. Rosiglitazone
D. Enalapril
E. Metoprolol
184.
185.
186. 27.A pt had knee injury knee is swollen, & medial ROTATION PAINFUL. cause TEAR
OF??
A. ACL
B. PCL
C. Medial meniscus
D. Lateral meniscus
187.

188. 28. 10 days old child, mother bring 5th time and saying child is crying and
difficult to cuddle, on exam no problem with the child. growth is normal. what clues
give to aid the dx?
189. A. premorbid personality of the mother
190. B. parental divorce
191. C. weight loss of the mother
192. D. thought of harming baby.
193.
194.
195.
196. Immigrant lady with syphilis, was separated from her husband for 6 years during
which she was raped, beaten, comes to you with her husband, husband agrees to get tested,
is negative for syphilis, what advise do you give to them –
197. a. use condoms for six months
198. b. get tested for syphilis antibody after 3 months
199. c. treat the husband as well
200.
201. 30. 35 year old pregnant woman presented at 15 weeks’ gestation asking you for a
diagnostic screening of down, what is the diagnostic screening test you should do for this
woman?
202. A-nuchal translucency ultrasound
203. B-maternal serum and ultrasound
204. C-maternal serum
205. D-amniocentesis
206. E-chorionic villous sampling
207.
208. ctg question
209. 3 ecg question .one of them was hyperkalmia asked ttt
210. got about 6 xray was new
211. one question for ct scan
212.
213.
214.
215.

216. Hi guys
217. This is my recalls(21/4/18)
218. Shamini v rathinam
219. I got ony 5 % of recalls,rest were all new questions
220. I got many psychiatry as well as ethics
221. Please do pray for me
222. All the best
223.
224. 1- Depersonalization occur in

225. Depression
226. Schizo
227. Some religious thingy
228. Ptd
229. Or all above
230.

231. 2-Jealousy is the main component of


schizo
232. Depression
233. Mania
234. Ocd ( acc to DSM 5 CRITERIA )
235.
236.
237. 3-Psy treatment in women common in

238. First
239. Second
240. Third
241. Post partum 6 w
242. Purperium
243.
244. 4-Who criteria , s/sx of schizo

245. Flat affect


246. Lack insight
247. Sucide idea
248. Visual hallucinations
249. Auditory hallucinations
250.
251. 5-Stimuli and response prevention tx

252. Bipolar
253. Schizo
254. Depression
255. Ocd
256. Agrophobia
257.
258. 6- a Malaysian student, uni student, his gf went back to attend dad’s funeral now
this guy become restless ,irritable and cant sleep and complains that his gf taling behind his
back Dx?

259. Suspicious cannabis


260. Grief reaction
261. Panic psychosis
262. Reactive psychosis
263. Onset of schizo
264.
265. 7-A teenage girl,c/o abd pain -ultrasound done noted 4cm mass at the ovary ,what
is ur mx
266. ( no option for tumour markers) dx may be follicular cyst which reloves
spontaneously

267. Review back in 4 weeks


268. Surgery
269. Review in 1 year
270. Review 2 year
271.
272. 8-man presented with pain and erthyema with midline calf tenderness for 2 days
273. What is the best mx? Dx gastrocnemius rupture

274. Elevate and leg rest


275. Unfractionated heparin
276. Ice
277.

278. 9-a woman admitted for calf pain for 3 days on warfarin
279. Currently still having swelling and calf pain
280. Inr:1.8
281. What to do next?
282. -start antibiotics- increase warfarin dose
283.
284. 10—recall of burn pt 10% wanna discharge =discharge
285.
286. 11- huntington recall (mum brought child to do genetic test- child is 8 years old)
=don’t do
287. Father with Huntington chorea, daughter carrier, she wants to check her 10-year-
old daughter?
Don’t do genetic testing
Counselling the 10-year-old daughter
Do the test for the 10-year-old daughter with her mother
Do genetic test for the daughter
Genetic counselling
288. Huntington chorea is autosomal dominant. The question is not right. The daughter can either be
affected or disease free but never a carrier
289. No genetic testing till 18 years. Do counseling to a daughter is a right option
290.
291. 16 years old Girl came to clinic, Father diagnosed Huntington disease. She wants
to test without parents wish. What should you do?
Arrange Genetic counselling her alone
arrange counselling whole family
order genetic test for her
tell her to comeback 18 years of age
tell her to get permission from parents
292. 5. Grandfather had Huntington, now mother come with daughter for Huntington
(Autosomal Dominant):
a.Arrange Genetic counselling of daughter
b.Arrange counselling whole family
c.Genetic counseling of mother & daughter
293. Chances of daughter getting the disease where paternal grandfather, father and
a paternal aunt have Huntington disease – 0%, 25%,50%,100%
294.
295. 12- both parents are thalassemia minor carrier, wanted to know how many % their
child will be having thalassemia major 25% autosomal recessive
296.
297.
298.

1. An old lady above 60 has history of breast cancer treated surgically 3 years ago presents for
osteoporosis screening her bone density scan shows -2.7 how to treat it
A. raloxifen
B. alendronate
C. vitamin d calcium

2. Woman with previous fracture, taking Risedronate 35mg, has recurrent vertebral fracture in
last 3 months, what next:

A. Change to IVzolendronic acid

B. Change to alendronate

C. Continue Risedronate

D. Increase Risedronate

3. Picture of lesions on legs ( psoriasis)asks for treatment ?



A- calcipotriol cream
B- uvB radiation
C- oral corticosteroid

4. A Diabetic female,scratched by cat in upper left arm . Develooed rash n pain in arm , forearm
n hand along with numbness in hand. Diagnosis
A. Diabetic myotrophy
B. Cat scratch fever

Cat-scratch disease commonly presents as tender, swollen lymph nodes near the
site of the inoculating bite or scratch or on the neck, and is usually limited to one side.
This condition is referred to as regional lymphadenopathy and occurs 1–3 weeks
after inoculation.[5] Lymphadenopathy in CSD most commonly occurs in the arms,
neck, or jaw, but may also occur near the groin or around the ear.[2] A vesicle or
an erythematous papule may form at the site of initial infectionOther associated
complaints include headache, chills, muscular pains, joint pains, arthritis, backache,
and abdominal pain. It may take 7 to 14 days, n rare situations, CSD can lead to the
development of serious neurologic or cardiac sequelae such
as meningoencephalitis, encephalopathy, seizures, or endocardi

C. Cellulitis

5. A mother came with son. 6 years h/o social withdrawal. Goes to uni,good grades. But does
nt work, no hobby or social life,,watches TV at nigh alone .dx?
A.schizophrenia
B.depression
C.schizoaffective disorder
D. agarophobia (Jm 918)

6. 24year old man confined with a friend came mentioning " funny business is going around"
same in inverted comma) he had a feeling something is going to happen. Euphoric state. (it
was diff scenario,nothing about promotion was mentioned) . What is the example of his
comments?

a) prodormal schezo
b) passivity phenomenon
c) idea of reference
d) delusional disorder

7. Man with right upper qaud pain,settled within an hour. No other finding,phy exam normal.
On ct there was a left sided mass on kidney. Rt sided kidney was normal( Im looking for that
ct did nt find until now.ill post it in comments of same post if I find it later.)
A. GB stone
B. RCC
C. Adrenal hmg
D. Ca head of pancreas

8. Ecg SVT(HB). Bp 85/50 Mx?
A. Dc cardiiversiin
B. Atropine
C. Pacemaking

9. ECG afib. Irregular palpitations. Mx
A. Metoprolol
B. Pacemaker
C. Cardioversion

10. Colles fracture recall. xray at 6 week followup was normal and now pt having mild pain and
tingling sensations what to do?

A. Xray again
B splint for few days more
C occupational therapy to check on him
(No option for exercise or analgesia)

11. A woman. Pain rt upper quad,jaundiced. CBD 11mm,no abdominal rigidity or gaurding. On
US stone in CBD confirmed. Treatment.
A. Cholecystectomy
B. Ercp
C. Laproscooic drainage something

12. Diverticulitis scenario n Ct. Fluid resus given. What next?
A. Admit in ICU
B. Observe
C. Laproscopy depends on CT

13. . 24yr old male with dysphagia and history of asthma in childhood. Endoscopy showed
narrowing and inflammation of middle esophagus, biopsy showed eosinophils in mucosa.
What is the best treatment?


a. Oral Antibiotic
b. PPI (jm 548, Dx—eosinophilic oesophagitis)
c. Budesonide

14. Child,post viral ITP. Non blanchable petechial rashes. No other abn finding or bleeding. On
FBE
Hb n WBC normal. PLATS 35×10 9. Treatment?

A. Strict bed rest
B. Steroids ( OX 338)
C. Immunoglobulins

15. A man from thailand trip. All malarial precautions. Slept in net,bathed from lakewater. Now
with fever. Malarial tests neg?

A. Retest malaria
B. Tests for other parasites
C. Other irrelevant

16. A man from africa trip. Fever n eosinophilia.2 malarial tests negative.next?
A. RETEST malaria
B. Giardia ( JM 136)
C. Schisto

17. Female with some abdominal infection. Vomitting n pain abdomen . (not pelvic,not after
genital instrumentation). Treatment?
A. Cipro
B. Genta+metro+vanco
C. Metro
Did nt understand this qs at all. Too short scenario.

18. Croup old recall. child 4yrs. Treatment?
19. Pnemonia old child, 2times same scenario same options. Treatment?

20. Wife tests for std. Husband calls clinic for payment. But asks for more info before paying

A. tell

B.Explain the procedure of not giving more info

C.Tell him to talk to wife about payment


21. Man in coma. Urgent surgery to be done. Son is not allowing. He is not power of attorney.?
A. Ask power of attorney
B. Don’t do surgery
C. Do surgery

Couldn’t understand any xrays , answered on basis of scenario

22. Man with nephritis n SOB. Next?xray given. Serum Ca high. Next?

A. Ace
B. Anti basement membrane antibodies
C. Ct chest

23. Man with ankle,wrists n knees swelling(non migratory). No sore throat Hx. Rash with
erythema on margins. Next?
A. Throat Swab
B. ANA Ana is the first nd the most sensitive marker for mixed connective tissue
diseases diagnosis
C. Rf factor
D.
24. Man with lumbar pain. No urinary symptoms. Xray spine given,could nt appreciate.
On FBE, S. calcium was borderline high,ESR 80 something,HB low. Next?

A. Bence jones proteins (JM 256)

B. Psa

C.Bonescan

25. HB qs of wound dehiscence,paralytic ileus
26. SAH scenario. No Ct.
27. Epicondylitis scenario. Next?
A. Naproxen
B. Paracetamol n rest
C. No bracing in options
D.
28. Cervical tear. Post partum 10 days? Same old
A. Observe
B. Suture under LA
C. Suture after cuttung margins

D.keep wound clean

29. Woman after vaginal delivery. 2700gm baby. Previous c section. Ergometrine given,placenta
delivered through cord traction. Now PPH. Cause?
A. Uerine atony
B. Uterine inversion
C. Uterine rupture
D.
30. Psgn. when can child return to school. After discharge

31. Nephrotic synd. Child,next inv?
A. Serum elect
B. Bp
C. Urine culture
32. Prostate. Active surveillance 2 qs
33. Adrenal mass 5cm,incidental. Next?
A. Surgery
B. Observe )

C. Biopsy
34. Kidney upper pole stone. 2cm. Treatment?
A. Eswl
B. Percut litho
C. Open Surgery


That’s all I can remember for now . I got many recalls. Will post any other qs that I
remember, below this post on amedex.
Remember in prayers

You might also like